Você está na página 1de 302

1 INTRODUCTION TO RESERVOIR ENGINEERING

2 RESERVOIR PRESSURES AND TEMPERATURES


3 RESERVOIR FLUIDS COMPOSITION
4 PHASE BEHAVIOUR OF HYDROCARBON SYSTEMS
5 BEHAVIOUR OF GASES
6 PROPERTIES OF RESERVOIR LIQUIDS
7 FUNDAMENTAL PROPERTIES OF RESERVOIR ROCKS
8 ROCK PROPERTIES MEASUREMENT
9 PERMEABILITY-ITS VARIATIONS
10 FLUID FLOW IN POROUS MEDIA
11 DRIVE MECHANISMS
12 VAPOUR LIQUID EQILIBRIA
13 EQUILIBRIUM RATIO PREDICTION AND CALCULATION
14 PVT ANALYSIS
15 MATERIAL BALANCE EQUATION
16 MATERIAL BALANCE EQUATION APPLICATION
17 WATER INFLUX
18 IMMISCIBLE DISPLACEMENT
19 EXAMINATION AND MODEL SOLUTIONS
RESERVOIR ENGINEERING RE
This Reservoir Engineering module covers material presented in a range of reservoir engineering texts and a
number of the figures and examples are based on these texts and copyright is currently being sought. The student
may find the more detailed analysis in these texts supportive when going through these notes. The following
books are considered useful in building up a reservoir engineering library.
1.Fundamentals of Reservoir Engineering. L.P.Dake. Elsevier. 1978
ISBN:0-444-41667-6
2.The Practise of Reservoir Engineering. L.P.Dake. Elsevier. 1994.
ISBN: 0-444-82094-9
3.Principles of Petroleum Reservoir Engineering. G.H.Chierici. Springer-Verlag 1994.
ISBN:3-540-56037-8
4.Fundamental Principles of Petroleum Reservoir B.F. Towler. Society of Petroleum Engineers Inc
Engineering ISBN:55563-092-8
5.Applied Reservoir Engineering B.C.Craft & M.F.Hawkins. Prentice Hall.
1959.
6.The Properties of Petroleum Fluids 2nd Ed W.D.McCain Pennwell Books . 1990
ISBN:0-87814-335-1
7.Petroleum Engineering Principles and Practise. J.S.Archer & C.Wall.Graham & Trotman.
1986. ISBN:0-86910-715-9
8.Petroleum Reservoir Engineering. J.W.Amyx,D.M.Bass & R.L.Whiting.
McGraw-Hill. 1960. ISBN:07-001600-3
9.PVT and Phase Behaviour of Petroleum Reservoirs A. Danesh. Elsevier. ISBN: 0-444-82196-1
Adrian C Todd
All rights reserved no part of this publication may be reproduced, stored in a retrieval system or
transmitted in any form or by any means, electronic, mechanical, photocopying, recording or
otherwise without the prior permission of the Copyright owner.
2
Insitute of Petroleum Engineering, Heriot-Watt University 3
Reservoir Engineering notes cover an extensive amount of material. They are support
material for the examination in this topic but are also considered to be useful material
in subsequent career use. Not all the material in the text can be covered in a limited
time examination.
In the context of the examination a student should consider the learning objectives at
the front of each section which should help in the level of detail and analysis which
is required in relation to an examination covering the various topics.
Detailed below is a graded analysis of each section which should help the candidate
in examination preparation. These should be considered alongside the learning
objectives.
Grading structure:
5 - Core material for examination purposes
4 - Core material less analytical than 5 - examinable.
3 - Between 4 & 2
2 - General awareness. Not so examinable with respect to analysis of detail.
1 - Other information not examinable.
OM- Material covered in another module not for examination purposes in Reservoir
Engineering.
Equations It is not necessary to memorise complicated equations. Equations unless
asked to be derived will be given.
Clearly some basic equations one should know and would not be given e.g.
Darcys Law,
PV = nzRT
STOOIP equation
Equilibrium Ratio K=y/x
4
C
h
a
p
t
e
r

1

I
n
t
r
o
d
u
c
t
i
o
n
S
e
c
t
i
o
n


g
r
a
d
i
n
g

1
.
1

4

1
.
2

4

1
.
3

4

2

4

3


3
.
1

4

3
.
2

4

3
.
3

4

3
.
4

4

4

O
M

5

O
M

6

4

7

4

8

4

9

4
C
h
a
p
t
e
r

2


R
e
s
e
r
v
o
i
r

P
r
e
s
s
u
r
e
s

1

5

2

5

3

5

4

5

5

5

6

4
C
h
a
p
t
e
r

3

R
e
s
e
r
v
o
i
r

C
o
m
p
o
s
i
t
i
o
n

1

5

2

5

3

4

4

5

5
.
1

5

5
.
2

2

5
.
3

2





C
h
a
p
t
e
r

4

P
h
a
s
e

B
e
h
a
v
i
o
u
r
A
l
l

m
a
t
e
r
i
a
l

5
C
h
a
p
t
e
r

5

G
a
s
e
s

1
.
1

5

1
.
2

5

1
.
3

5

1
.
4

5

1
.
5

5

1
.
6

5

1
.
7

5

1
.
8

5

1
.
9

5

2
.
1

5

2
.
2

5

2
.
3

5

2
.
4

1

2
.
5

5

3

5

4

3

5

3

6

2
C
h
a
p
t
e
r

6


L
i
q
u
i
d
s

1

5

2

5

3

5

4

5

5

5

6

5

7

3



n
o
t
e

t
h
e
r
e

i
s

a
n

e
r
r
o
r

i
n

s
o
m
e

t
e
x
t
s

w
i
t
h

a
n
o
t
h
e
r

7

h
e
a
d
i
n
g

8
.
1

5

8
.
2

5

9

5

1
0

3

1
1

1

1
2

5





C
h
a
p
t
e
r

7


R
e
s
e
r
v
o
i
r

R
o
c
k
s

1

3

2

4

3

3

4
.
1

5

4
.
2

5

4
.
3

3

4
.
4

3

4
.
5

5

4
.
6

5

4
.
7

4

4
.
8

5

4
.
9

2

4
.
1
0

2

5

3

6

5

7
.
1

5

7
.
2

5

7
.
3

5

8
.
1

5

8
.
2

5

8
.
3

5
C
h
a
p
t
e
r

8

R
o
c
k

M
e
a
s
u
r
e
m
e
n
t

1
.
1

2

2
.
1

2

2
.
2

2

3
.
1

2

3
.
2

2

4
.
1

3

4
.
2

3

4
.
3

3

4
.
4

3

5

2

6
.
1

5

6
.
2

3

6
.
3

5

6
.
4

5

7









2
C
h
a
p
t
e
r

9

-

P
e
r
m
e
a
b
i
l
i
t
y

V
a
r
i
a
t
i
o
n
s

1

3

2

5

3

5
C
h
a
p
t
e
r

1
0

F
l
u
i
d

F
l
o
w




1

3

2

3

3
.
1

3

3
.
2

3

3
.
3
.
1

3

3
.
3
.
2

3

3
.
3
.
3

3

3
.
3
.
3
.
1

5

3
.
3
.
4

5

3
.
4

5

3
.
4
.
1

3

3
.
5

5

4

1

5

5

5
.
2

5

5
.
3

5

6

5
Insitute of Petroleum Engineering, Heriot-Watt University 5
C
h
a
p
t
e
r

1
1

D
r
i
v
e

M
e
c
h
a
n
i
s
m
s
A
l
l

s
e
c
t
i
o
n
s

5
C
h
a
p
t
e
r

1
2


V
a
p
o
u
r

L
i
q
u
i
d

E
q
u
i
l
i
b
r
i
u
m

1

2

2

2



E
q

1
1


5

3

5

4
.
1

5

4
.
2

5

4
.
3

5

5
.
1

3

5
.
2

3

5
.
3

3

5
.
4

3
C
h
a
p
t
e
r


1
3

E
q
u
i
l
i
b
r
i
u
m

R
a
t
i
o

1

3

2

3

3

2

4

2
C
h
a
p
t
e
r

1
4

P
V
T

1

4

2

2

3
.
1

5

3
.
2

5

3
.
3

5

3
.
4

2

3
.
5

2

4

5

5

2

6

5

7

3

8
.
1

2

8
.
2

2

9

5

1
0

5

1
1

5

1
2

5

1
3

3

1
4

3

1
5

1
C
h
a
p
t
e
r

1
5

M
a
t
e
r
i
a
l

B
a
l
a
n
c
e

1

5

2

3

3

5

4

5

5

5

6

5

7

5

8

5

9

5

1
0

5

1
1

5

1
2

5
C
h
a
p
t
e
r

1
6


M
B

A
p
p
l
i
c
a
t
i
o
n

1

5

2

5

3

5

4

4

5
.
1

5


(
5
.
1
.
2
.
2



E
q
4
6




-
1

)

5
.
2

4

5
.
3
.
1

4

5
.
3
.
2

2

5
.
3
.
3

5

5
.
3
.
4

5

5
.
3
.
5

2

5
.
4

2

5
.
5

1

5
.
6

3

6

1
C
h
a
p
t
e
r

1
7

W
a
t
e
r

I
n
f
l
u
x

1

5

2
.
1

3

2
.
2

3

2
.
3

3

2
.
4

3

2
.
5

5

3

5

4

4

5

3

6

2

7

2
C
h
a
p
t
e
r

1
8

I
m
m
i
s
c
i
b
l
e

D
i
s
p
l
a
c
e
m
e
n
t

1

5

2

5

3
.
1

3

3
.
2

5

3
.
3

3


(

E
q
n

1


5


-
s
h
o
u
l
d

b
e

e
x
p
e
c
t
e
d

t
o

k
n
o
w

)

3
.
4

4


(

p
o
s
t

e
q
u
a
t
i
o
n

1
4







5

)

4

5

5
.
1

2

5
.
2

5


(

f
r
o
m

e
q
u
a
t
i
o
n

7
2
+




-


2
)

6
.
1

3

6
.
2

3

6
.
3

3

6
.
4

5

6
.
5

5

6
.
6

1

6
.
7

5

7

5

8
.
1

2

8
.
2

2

8
.
3

2

8
.
4

1

8
.
5

1
Introduction To Reservoir Engineering
CONTENTS
1 INTRODUCTION
1.1 Reserves Estimation
1.2 Development Planning
1.3 Production Operations Optimsation
2 RESERVOIR ENGINEERING TECHNIQUES
3 RESERVE ESTIMATING
3.1 Defnitions
3.2 Proven Reserves
3.2.1 Exercises-ReserveDefnitions
3.3 Unproved Reserves
3.3.1 Probable Reserves
3.3.2 Possiible Reserves
3.4 Reserve Status Categories
3.4.1 Developed:
3.4.1.1 Producing
3.4.1.2 Non-producing:
3.4.2 Undeveloped Reserves:
4 PROBABILISTIC REPRESENTATION OF
RESERVES
5 VOLUME IN - PLACE CALCULATIONS
5.1 Volume of Oil and Gas in-Place
5.2 Evolution of Reserve Estimate
5.3 Reservoir Area
5.4 Reservoir Thickness
5.5 Reservoir Porosity
5.6 Water Saturation
5.7 Formation Volume Factors
5.8 Recovery Factors
5.9 Production Capacity
5.10 Hydrocarbon Pore Volume Map
6 OTHER APPRAISAL ROLES
7 DEVELOPMENT PLANNING
7.1 Reservoir Modelling
7.2 Technoconomics
7.3 Coping with Uncertainty
8. PRODUCTION OPERATIONS OPTIMISATION
8.1 Development Phase
8.2 History Matching
8.3 Phases of Development
9. THE UNIQUENESS OF THE RESERVOIR
10. CONCLUSIONS

LEARNING OBJECTIVES
Having worked through this chapter the Student will be able to:
Show using a block diagram the integration of reservoir engineering with other
petroleum engineering and other subjects.
DefnetheSPEdefnitionsofreserves;provenreserves,unprovedreserves;
probable reserves and possible reserves.
Calculategiventheprerequisitedataproved,probableandpossiblereserves.
Describe in general terms reserve estimation.
Sketchadiagramshowingtheprobabilityversusrecoverablereservesindicating,
proven,proven+probableandproven+probable+possiblereserves.
Present a simple equation for volumes of oil and gas in-place.
Describe in general terms the evolution of reserves through successive
exploration wells.
Describebriefywiththeaidofasketchthevariousmapsusedtorepresent
reservoir;area,thicknessporosity,saturation.
Describebriefytheuseoftheproduction(well0testtodeterminereservoir
fowabilityandproperties.
Describebriefythevariouselementsofdevelopmentplanning:reservoir
modeling technoeconomics and uncertainty.
Illustrate with a sketch the impact of different technical parameters on the
associated uncertainties on a project.
Describeingeneraltermsinthecontextofproductionoperations,optimization
in history matching.
Draw a sketch showing the various phases of production from build up to
economic limit.
Draw a sketch illustrating the various recovery scenarios from primary to
tertiary recovery.
Introduction To Reservoir Engineering
Insitute of Petroleum Engineering, Heriot-Watt University

1 INTRODUCTION
Withthepetroleumindustrysdesiretoconserveandproduceoilandgasmoreeffciently
afeldofspecialisationhasdevelopedcalledPetroleumReservoirEngineering.This
newsciencewhichcanbetracedbackonlytothemid1930shasbeenbuiltupona
wealthofscientifcandpracticalexperiencefromfeldandlaboratory.Inthe1959
text of Craft & Hawkins
1
on Applied Reservoir Engineering it is commented that as
early as 1928 petroleum engineers were giving serious consideration to gas-energy
relationships and recognised the need for more precise information concerning
physical conditions as they exist in wells and underground reservoirs. Early progress
in oil recovery methods made it obvious that computations made from wellhead or
surface data were generally misleading. Dake
2
,inhistext"ThePractiseofReservoir
Engineering", comments that Reservoir Engineering shares the distinction with
geologyinbeingoneoftheundergroundsciencesoftheoilindustry,attempting
to describe what occurs in the wide open spaces of the reservoir between the sparse
points of observation - the wells
The reservoir engineer in the multi-disciplinary perspective of modern oil and gas
feldmanagementislocatedattheheartofmanyoftheactivitiesactingasacentral
co-ordinating role in relation to receiving information processing it and passing it on
to others. This perspective presented by Dake
2
isshowninthefgurebelow.

Exploration
Geophysics/
Geology
Petrophysics
Reservoir Engineering
Economics
(Project viability)
General Engineering
Platform Topsides Design
Production
Process Egineering

Figure 1 ReservoirEngineeringinRelationtoOtherActivities(adaptedDake
2
)

Dake
2
hasusefullyspecifedthedistincttechnicalresponsibilitiesofreservoir
engineers as:
Contributing, with the geologists and petrophysicists , to the estimation of
hydrocarbons in place.
Determining the fraction of discovered hydrocarbons that can be recovered.
Attaching a time scale to the recovery.

Day-to-day operational reservoir engineering throughout the project lifetime.


Theresponsibilityofthefrstissharedwithotherdisciplineswhereasthesecondis
primarily the responsibility of the reservoir engineer. Attaching a time scale to recovery
isthedevelopmentofaproductionprofleandagainisnotanexclusiveactivity.The
day-to-day operational role is on going through the duration of the project.
A project can be conveniently divided into two stages and within these the above
activitiestakeplace,theappraisalstageandthedevelopmentphase.Theappraisal
phase is essentially a data collection and processing phase with the one objective of
determiningtheviabilityofaproject.Thedevelopmentphasecoverstheremaining
period if the project is considered viable from the time continuous production com-
mencestothetimethefeldisabandoned.Reservoirengineeringactivityinvarious
forms takes place during both of these stages.
The activities of reservoir engineering fall into the following three general catego-
ries:
(i) ReservesEstimation
(ii) DevelopmentPlanning
(iii) ProductionOperationsOptimisation
1.1 Reserves Estimation
Theundergroundreservesofoilandgasformtheoilcompanysmainassets.Quan-
tifying such reserves forms therefore a very important objective of the practising
reservoirengineerbutitisalsoaverycomplexproblem,forthebasicdataisusually
subjecttowidelyvaryinginterpretationsandontopofthat,reservesmaybeaffected
signifcantly by the feld developmentplan and operating practice.It is an on-go-
ingactivityduring,exploration,developmentplanningandduringproduction.Itis
clearly a key task of the appraisal phase for it is at the heart of determining project
viability.
Before any production has been obtained, the so-called volumetric estimate of
reservesisusuallymade.Geologicalandgeophysicaldataarecombinedtoobtain
a range of contour maps with the help of a planimeter and other tools the hydrocar-
bonbearingrockvolumescanbeestimated.Fromwelllogpetrophysicalanalysis,
estimates of an average porosity and water saturation can be made and when applied
tothehydrocarbonrockvolumeyieldanestimateofoilinplace(STOIIP).Since
itiswellknownthatonlyafractionofthisoilmayinfactberecoverable,labora-
tory tests on cores may be carried out to estimate movable oil. The reserve estimate
fnallyarrivedatislittlemorethananeducatedguessbutaveryimportantonefor
it determines company policy.
In 1987 the Society of Petroleum Engineers in collaboration with the World Petroleum
Congresspublisheddefnitionswithrespecttoreservesandthesearenowaccepted
world-wide
3
.Thesedefnitionshavebeenusedinthesummaryofreservedefni-
tions which follow.
Introduction To Reservoir Engineering
Insitute of Petroleum Engineering, Heriot-Watt University

1.2 Development Planning


Oilfelddevelopment,particularlyintheoffshoreenvironment,isafrontloaded
investment. Finance has to be committed far in advance not only of income guaran-
teedbytheinvestment,butfrequentlyalsoofgooddefnitivedataonthecharacter
ofthefeld.Muchoftheresponsibilityforthistypeofactivityfallsonthereservoir
engineersbecauseoftheirappreciationforthecomplexcharacterofsub-surfacefuid
behaviour under various proposed development schemes.
1.3 Production Operations Optimisation
Producingfeldswillseldombehaveasanticipatedand,ofcourse,bytheverynature
ofthissortofactivity,thebalanceofforcesinthereservoirrockgetsseverelyupsetby
oilandgasproduction.Thereservoirengineerisfrequentlycalledupontoexplain
acertainaspectofwellperformance,suchasincreasinggas-oilratio,sandand/or
water production and more importantly will be asked to propose a remedy. The actual
performance of the reservoir as compared to the various model predictions is another
ongoing perspective during this phase.
2 RESERVOIR ENGINEERING TECHNIQUES
In the past the traditionally available reservoir engineering tools were mainly
designed to give satisfactory results for a slide rule and graph paper approach. For
many problems encountered by reservoir engineers today this remains a perfectly
validapproachwherethesliderulehasbeenreplacedbythecalculator.Increasingly,
however, the advance of computing capability is enabling reservoir engineering
modellingmethods(simulations)tobecarriedoutattheengineersdesk,previously
considered impossible.
The basis of the development of the 'model' of the reservoir are the various data
sources. As the appraisal develops the uncertainty reduces in relation to the quality
oftheforecastspredictedbythemodel.Buildingupthisgeologicalmodelofthe
reservoirprogressesfromtheearlyinterpretationofthegeophysicalsurveys,through
variouswellderiveddatasets,whichincludedrillinginformation,indirectwireline
measurements,recoveredcoredata,recoveredfuidanalysis,pressuredepthsurveys,
to information generated during production.
3. RESERVE ESTIMATING
The Society of Petroleum Engineers SPE and World Petroleum Congress WPO1987
agreedclassifcationofreserves
3
providesavaluablestandardbywhichtodefne
reserves,thesectionbelowisbasedonthisclassifcationdocument.
3.1 Defnitions
Reserves are those quantities of petroleum which are anticipated to be commercially
recovered from known accumulations from a given date forward.
All reserve estimates involve some degree of uncertainty. The uncertainty depends
chiefyontheamountofreliablegeologicandengineeringdataavailableatthetime

of the estimate and the interpretation of these data. The relative degree of uncertainty
maybeconveyedbyplacingreservesintooneoftwoprincipalclassifcations,either
proved or unproved.
Unproved reserves are less certain to be recovered than proved reserves and may
befurthersub-classifedasprobable and possible reserves to denote progressively
increasing uncertainty in their recoverability.
Estimation of reserves is carried out under conditions of uncertainty. The method of
estimation is called deterministic if a single best estimate of reserves is made based
onknowngeological,engineering,andeconomicdata.Themethodofestimationis
called probabilisticwhentheknowngeological,engineering,andeconomicdataare
used to generate a range of estimates and their associated probabilities. Identifying
reservesasproved,probable,andpossiblehasbeenthemostfrequentclassifcation
method and gives an indication of the probability of recovery. Because of potential
differencesinuncertainty,cautionshouldbeexercisedwhenaggregatingreserves
ofdifferentclassifcations.
Reserves estimates will generally be revised as additional geologic or engineering
data becomes available or as economic conditions change. Reserves do not include
quantitiesofpetroleumbeingheldinaninventory,andmaybereducedforusageor
processinglossesifrequiredforfnancialreporting.

Reserves may be attributed to either natural energy or improved recovery methods.
Improved recovery methods include all methods for supplementing natural energy
or altering natural forces in the reservoir to increase ultimate recovery. Examples of
suchmethodsarepressuremaintenance,gascycling,waterfooding,thermalmethods,
chemicalfooding,andtheuseofmiscibleandimmiscibledisplacementfuids.Other
improved recovery methods may be developed in the future as petroleum technology
continues to evolve.
3.2 Proven Reserves
Proven reserves are those quantities of petroleum which, by analysis of geological
and engineering data, can be estimated with reasonable certainty to be commercially
recoverable, from a given date forward, from known reservoirs and under current
economic conditions, operating methods, and government regulations.
Proved reserves can be categorised as developed or undeveloped.
Ifdeterministicmethodsareused,thetermreasonablecertaintyisintendedtoexpress
ahighdegreeofconfdencethatthequantitieswillberecovered.Ifprobabilistic
methodsareused,thereshouldbeatleasta90%probabilitythatthequantitiesactu-
ally recovered will equal or exceed the estimate.
Establishment of current economic conditions should include relevant historical
petroleum prices and associated costs and may involve an averaging period that is
consistentwiththepurposeofthereserveestimate,appropriatecontractobligations,
corporate procedures, and government regulations involved in reporting these
reserves.Ingeneral,reservesareconsideredprovedifthecommercialproducibility
ofthereservoirissupportedbyactualproductionorformationtests.Inthiscontext,
Introduction To Reservoir Engineering
Insitute of Petroleum Engineering, Heriot-Watt University

the term proved refers to the actual quantities of petroleum reserves and not just
the productivity of the well or reservoir. In certain cases, proved reserves may
beassignedonthebasisofwelllogsand/orcoreanalysisthatindicatethesubject
reservoir is hydrocarbon bearing and is analogous to reservoirs in the same area that
are producing or have demonstrated the ability to produce on formation tests.
Theareaofthereservoirconsideredasprovedincludes(1)theareadelineatedby
drillinganddefnedbyfuidcontacts,ifany,and(2)theundrilledportionsofthe
reservoir that can reasonably be judged as commercially productive on the basis of
availablegeologicalandengineeringdata.Intheabsenceofdataonfuidcontacts,the
lowest known occurrence of hydrocarbons controls the proved limit unless otherwise
indicatedbydefnitivegeological,engineeringorperformancedata.Reservesmaybe
classifedasprovediffacilitiestoprocessandtransportthosereservestomarketare
operational at the time of the estimate or there is a reasonable expectation that such
facilitieswillbeinstalled.Reservesinundevelopedlocationsmaybeclassifedas
provedundevelopedprovided(1)thelocationsaredirectoffsetstowellsthathave
indicated commercial production in the objective formation, (2) it is reasonably
certain such locations are within the known proved productive limits of the objective
formation, (3) the locations conform to existing well spacing regulations where
applicable,and(4)itisreasonablycertainthelocationswillbedeveloped.Reserves
from other locations are categorised as proved undeveloped only where interpretations
of geological and engineering data from wells indicate with reasonable certainty that
the objective formation is laterally continuous and contains commercially recoverable
petroleum at locations beyond direct offsets.
Before looking at further detail we will carry out some tests to help emphasise the
abovedefnition.
3.2.1 Exercises - Reserve Defnitions
ThesectiononReserveDefnitionsasputtogetherbytheSPEandtheWorldPetro-
leumCongress,defnesthevariousaspectsofreservedefnitions.Thesedefnitions,
areimportantbothtocompaniesandcountries,andtheycanhaveverysignifcant
commercial impact. The following tests are presented to help understand the work-
ingoftheseearlierdefnitions.
Test 1
Thereare950MMstb(millionstocktankbarrels)ofoilinitiallyinplaceinares-
ervoir. It is estimated that 500 MM stb can be produced. Already 100 MM stb have
beenproduced.Intheboxesbelow,identifythecorrectanswer.

950
STOIIP is: MM stb
500 400
450 400 500
MM stb The Reserves are:
Turn to page 9 for answers

Test 2
Beforestartingproductionitwasestimatedthattherewasa90%chanceofproduc-
ingatleast100MMstb,50%chanceofproducing500MMstband10%chanceof
producing700MMstb.Thatiswearesurewecanproduceatleast100MMstb,and
wewillprobablyproduceasmuchas500MMstb,andwewillpossiblyproduceas
much as 700 MM stb.
Tick the correct answers.

500 400
400 500
400 500
200
200
200
100
100
100
600
600
600
700
700
700
Proved reserves (MM stb):
Probable reserves
Possible reserves
Turn to page 9 for answers
Test 3
Whatiswrongwiththefollowingdefnitions?
1. Reserves are those quantities of petroleum which are anticipated to be recovered
from a petroleum accumulation.
Test 4
1. We have a structure in our licence area which we intend to explore. We anticipate
ittocontainaSTOIIPof2000MMstb,andrecoveryfactorof65%usingprimary
methods(30%),secondary(25%)andtertiary(10%)recoverymethods.Whatare
thereserves?
Test 5
Areservoirhasbeendiscoveredbydrillingasuccessfulexplorationwell,anddrilling
a number of producing wells. We have even produced some 200 MM stb of oil.
STOIIP=2000MMstb Recoveryfactor=35%
Whatarethereserves?
Introduction To Reservoir Engineering
Insitute of Petroleum Engineering, Heriot-Watt University

Test 1 answer
There are 950 MM stock tank boards in place. It is estimated that 500 MM stb can
be produced and 100 MM stb have been produced then 400 recoverable reserves
remain.

950
STOIIP is: MM stb
500 400
450 400 500
MM stb The Reserves are:
X
X
X
X

Test 2 answer
Proved : 100 MM stb
Probable : 500 - 100 = 400 MM stb
Possible : 700 - 500 = 200 MM stb
Proved : 100 MM stb
Proved & Possible 500 MM stb
Proved & Probable & Possible : 700 MM stb
Test 3 answer
Reserves are those quantities of petroleum which are anticipated to be commercially
recovered from a petroleum accumulation.
Clearlyeconomicsisaveryimportantaspectofthedefnition.
Economic Variables
Whateconomicfactorsareusedinthecalculations?Whatoilandgaspricedowe
useforprovedreserveestimates?Isinfationtakenintoaccount?Dowepredict
futurepricetrends?Doweapplydiscountfactorstocalculatepresentvalueofthe
project?Arealltheseusedinprovedreservecalculations?Thecurrenteconomic
conditionsareusedforthecalculations,withrespecttoprices,costs,contractsand
government regulations.
Test 4 answer
1.AnsweriszerobySPC/WPCdefnition.
2. Intentions and anticipations are not the basis for reserves. In this case no well
has yet been drilled.
Note: Some companies allocate potential reserves for internal use but these cannot
beusedforpublicandgovernmentfgures.
Reserves are those quantities of petroleum which are anticipated to be commercially
recovered from a known accumulation.
Requirements for Proved include
Thefollowingsourcesarerequiredforprovedreserves.Maps(fromseismicand/
geological data). Petrophysical logs. Well test results and rock properties from core
analysis tests on recovered core.
10
Facilities
Animportantperspectivewhichmightbeforgottenbythereservoirengineer,isthat
forreservestobeclassifedasproven,allthenecessaryfacilitiesforprocessing
and the infrastructure for transport must either be in place or that such facilities will
beinstalledinthefuture,asbackedupbyaformalcommitment.
Contribution to the Proved Reservoir Area
Thiscomesfromdrilledandproducedhydrocarbons,thedefnitionofthegasandoil
and water contacts or the highest and lowest observed level of hydrocarbons. Also
the undrilled area adjacent to the drilled can be used.
Test 5 answer
Ultimate recovery = 2 000 x 0.35 = 700 MM stb
Minus production to date = 200
Reserves = 500 MM stb
Reserves are those quantities of petroleum which are anticipated to be commercially
recovered from known accumulations from a given date forward.
i.e. Reserves refer to what can be produced in the future.
Figure 2 gives a schematic of reserves showing the progression with time.

SPE / WPC Definitions Potential
P10
P50
P90
Time
Start of
Production
Abandonment
Start of Dev
Planning
Discovery of
Well
Seismic
Data
Before Drilling
Exploration Well
Prior and During
Appraisal
Delineation, Evaluation,
Development
Production PERIOD
Geophysical
and Geological
Geophysical,
Geological,
Petrophysical
and Well Test Data
Geophysical,
Geological,
Petrophysical
and Well Tests and Production Data
Reservoir Performance
and Production Data
TYPE OF
DATA
Mostly Probabilistic Deterministic and Probabilistic
METHOD
Possible
Probable
Provan
Possible
Probable
Provan
Cumulative Production
R
E
S
E
R
V
E

C
A
T
E
G
O
R
I
E
S
P
r
o
b
a
b
i
l
i
t
y

L
e
v
e
l
s
Figure 2 Variations of Reserves During Field Life
Whataretheamountstermedthatarenotrecoverable?Thequantityofhydrocar-
bonsthatremainsinthereservoirarecalledremaininghydrocarbonsinplace,NOT
remaining reserves!
Reserves which are to be produced through the application of established improved
recoverymethodsareincludedintheprovedclassifcationwhen:
Introduction To Reservoir Engineering
Insitute of Petroleum Engineering, Heriot-Watt University
11
(i)Successfultestingbyapilotprojectorfavourableresponseofaninstalled
programinthesameorananalogousreservoirwithsimilarrockandfuid
propertiesprovidessupportfortheanalysisonwhichtheprojectwasbased,
and,

(ii)Itisreasonablycertainthattheprojectwillproceed.Reservestoberecovered
by improved recovery methods that have yet to be established through
commerciallysuccessfulapplicationsareincludedintheprovedclassifcation
only:
(i)Afterafavourableproductionresponsefromthesubjectreservoirfromeither
(a)Arepresentativepilotor
(b)Aninstalledprogramwheretheresponseprovidessupportfortheanalysis
on which the project is based and

(ii)Itisreasonablycertaintheprojectwillproceed.
3.3 Unproved Reserves
Unproved reserves are based on geologic and/or engineering data similar to that
used in estimates of proved reserves; but technical, contractual, economic, or
regulatory uncertainties preclude such reserves being classifed as proved.
Unprovedreservesmaybefurtherclassifedasprobable reserves and possible re-
serves. Unproved reserves may be estimated assuming future economic conditions
different from those prevailing at the time of the estimate. The effect of possible
future improvements in economic conditions and technological developments can
be expressed by allocating appropriate quantities of reserves to the probable and
possibleclassifcations.
3.3.1. Probable Reserves
Probable reserves are those unproved reserves which analysis of geological and
engineering data suggests are more likely than not to be recoverable.Inthiscontext,
whenprobabilisticmethodsareused,thereshouldbeatleasta 50% probability that
the quantities actually recovered will equal or exceed the sum of estimated proved
plusprobablereserves.Ingeneral,probablereservesmayinclude:
(1)Reservesanticipatedtobeprovedbynormalstep-outdrillingwheresubsurface
controlisinadequatetoclassifythesereservesasproved,
(2) Reserves in formations that appear to be productive based on well log
characteristicsbutlackcoredataordefnitivetestsandwhicharenotanalogous
toproducingorprovedreservoirsinthearea,
(3)Incrementalreservesattributabletoinflldrillingthatcouldhavebeenclassifed
as proved if closer statutory spacing had been approved at the time of the
estimate,
1
(4)Reservesattributabletoimprovedrecoverymethodsthathavebeenestablished
byrepeatedcommerciallysuccessfulapplicationswhen;
(a)aprojectorpilotisplannedbutnotinoperationand
(b)rock,fuid,andreservoircharacteristicsappearfavourableforcommercial
application,
(5)Reserves in an area of the formation that appears to be separated from the
proved area by faulting and the geologic interpretation indicates the subject
areaisstructurallyhigherthantheprovedarea,
(6)Reservesattributabletoafutureworkover,treatment,re-treatment,changeof
equipment,orothermechanicalprocedures,wheresuchprocedurehasnotbeen
proved successful in wells which exhibit similar behaviour in analogous
reservoirs,and
(7)Incrementalreservesinprovedreservoirswhereanalternativeinterpretationof
performanceorvolumetricdataindicatesmorereservesthancanbeclassifed
as proved.
3.3.2. Possible Reserves
Possible reserves are those unproved reserves which analysis of geological and en-
gineering data suggests are less likely to be recoverable than probable reserves.
Inthiscontext,whenprobabilisticmethodsareused,thereshouldbeatleasta10%
probability that the quantities actually recovered will equal or exceed the sum of
estimatedprovedplusprobablepluspossiblereserves.Ingeneral,possiblereserves
may include:

(1)reserveswhich,basedongeologicalinterpretations,couldpossiblyexist
beyondareasclassifedasprobable,
(2)reservesinformationsthatappeartobepetroleumbearingbasedonlogand
coreanalysisbutmaynotbeproductiveatcommercialrates,
(3)incrementalreservesattributedtoinflldrillingthataresubjecttotechnical
uncertainty,
(4)reservesattributedtoimprovedrecoverymethodswhen
(a) aprojectorpilotisplannedbutnotinoperationand
(b)rock,fuid,andreservoircharacteristicsaresuchthatareasonabledoubt
existsthattheprojectwillbecommercial,and

(5)reservesinanareaoftheformationthatappearstobeseparatedfromthe
proved area by faulting and geological interpretation indicates the subject area
is structurally lower than the proved area.
Introduction To Reservoir Engineering
Insitute of Petroleum Engineering, Heriot-Watt University
1
3.4 Reserve Status Categories
Reservestatuscategoriesdefnethedevelopmentandproducingstatusofwellsand
reservoirs.
3.4.1. Developed:
Developed reserves are expected to be recovered from existing wells including reserves
behind pipe. Improved recovery reserves are considered developed only after the
necessaryequipmenthasbeeninstalled,orwhenthecoststodosoarerelativelyminor.
Developed reserves may be sub-categorised as producing or non-producing.

3.4.1.1 Producing:
Reserves subcategorised as producing are expected to be recovered from comple-
tion intervals which are open and producing at the time of the estimate. Improved
recovery reserves are considered producing only after the improved recovery project
is in operation.
3.4.1.2. Non-producing:
Reserves subcategorised as non-producing include shut-in and behind-pipe reserves.
Shut-inreservesareexpectedtoberecoveredfrom(1)completionintervalswhich
areopenatthetimeoftheestimatebutwhichhavenotstartedproducing,(2)wells
whichwereshut-informarketconditionsorpipelineconnections,or(3)wellsnot
capable of production for mechanical reasons. Behind-pipe reserves are expected to
berecoveredfromzonesinexistingwells,whichwillrequireadditionalcompletion
work or future recompletion prior to the start of production.
3.4.2. Undeveloped Reserves:
Undeveloped reserves are expected to be recovered:
(1) Fromnewwellsonundrilledacreage,
(2) Fromdeepeningexistingwellstoadifferentreservoir,or
(3) Wherearelativelylargeexpenditureisrequiredto
(a)Recompleteanexistingwellor
(b)Installproductionortransportationfacilitiesforprimaryorimproved
recovery projects.
4. PROBABILISTIC REPRESENTATION OF RESERVES
Whereas in the deterministic approach the volumes are determined by the calculation
ofvaluesdeterminedforthevariousparameters,withtheprobalisticstatisticalanalysis
isused,usingtoolslikeMonteCarlomethods.Thecurveasshowninthefgure3
below presents the probability that the reserves will have a volume greater or equal
to the chosen value.
1

'Proven'
'Proven + Probable'
P
r
o
b
a
b
i
l
i
t
y

t
h
a
t

t
h
e

r
e
s
e
r
v
e

i
s

a
t

l
e
a
s
t
a
s

l
a
r
g
e

a
s

i
n
d
i
c
a
t
e
d
.
'Proven + Proable
+ Possible'
1.0
0.9
0.5
0.1
0
Recoverable Reserve
Figure 3 Probabilistic Representation of Recoverable Reserves.
On this curve:
Theprovenreservesrepresentthereservesvolumecorrespondingto90%probability
on the distribution curve.
The probable reserves represent the reserves volume corresponding to the difference
between50and90%probabilityonthedistributioncurve.
The possible reserves represent the reserves volume corresponding to the difference
between10and50%probabilityonthedistributioncurve.
As with the deterministic approach there is also some measure of subjectivity in the
probalisticapproach.Foreachoftheelementsinthefollowingequation,thereisa
probabilityfunctionexpressioninlow,mediumandhighprobabilitiesfortheparticular
values. A schematic of a possible distribution scenario for each of the elements and
thefnalresultisgivenbelowinthefgure4.

Net rock Net rock Connate Formation Estimated


volume. average water volume recovery
porosity saturation factor factor

[ V
nr
x x (1 - S
wc
) / B ] x RF = Reserves
o
Uniform Triangular Gaussian Uniform
p90
p50
p10
=

P
Figure 4 Probablistic Reserve Estimates.
Introduction To Reservoir Engineering
Insitute of Petroleum Engineering, Heriot-Watt University
1
Theresultingcalculationsresultinaprobabilityfunctionforafeldasshownin
thefgure5below,wherethevaluesforthethreeelementsareshown
Proven=500MMstbtheP90fgure.
Probable=240MMstbwhichtogetherwiththeprovenmakesuptheP50fgure.
of 740MMstb
Possible = 120 MM stb which together with the proven and probable makes up the
P10 value of 860MMstb

Reserves distribution for a new field.
Reserves / MMstb
P
r
o
b
a
b
i
l
i
t
y

/

%
100
90
80
70
60
50
40
30
20
10
0
0 200 400 600 800 1000
P10 = 860 MMstb
P50 = 740 MMstb
P90 = 500 MMstb
Proven 500 MMstb
Probable 240 M
P+P+P = 860 MMstb
Proven Probable Possible
P90
P50
120 P10
Figure 5 Reserves Cummulative Probability Distribution.
Asafeldisdevelopedandthefuidsareproducedtheshapeoftheprobabilitycurve
changes.Probabilityfguresforreservesaregraduallyconvertedintorecoveryleav-
inglessuncertaintywithrespecttothereserves.Thisisillustratedinfgure6.
1

100
90
80
70
60
50
40
30
20
10
0
0 200 400 600 800 1000
Reserves / MMstb
P
r
o
b
a
b
i
l
i
t
y

/

%
Proved ultimate recovery.
Proved reserves Production
P90
P50
P10
Figure 6 Ultimate Recovery and Reserves Distribution For a Mature Field.
5. VOLUME IN-PLACE CALCULATIONS
5.1 The volume of oil and gas in-place depends on a number of parameters :
The aerial coverage of the reservoir. A
The thickness of the reservoir rock contributing to the hydrocarbon volume. h
n

Theporevolume,asexpressedbytheporosity,,thereservoirqualityrock.
Theproportionofporespaceoccupiedbythehydrocarbon(thesaturation).
1-S
w
Thesimpleequationusedincalculationofthevolumeoffuidsinthereservoir,V,
is
V=Ah
n
(1-S
w
): (1)
where:
A= average area
h
n
= nett thickness. nett thickness = gross thickness x nett: gross ratio
= average porosity
S
w
= average water saturation.
Whenexpressedasstocktankorstandardgasvolumes,equationaboveisdivided
by the formation volume factor B
o
or B
g
.

V Ah S B
n w o
( ) / 1
(2)
To convert volumes at reservoir conditions to stock tank conditions formation volume
factors are required where B
o
and B
g
are the oil and gas formation volume factors.
Thesearedefnedinsubsequentchapters.Theexpressionoforiginaloilinplaceis
termed the STOIIP.
Introduction To Reservoir Engineering
Insitute of Petroleum Engineering, Heriot-Watt University
1
Therecoveryfactor, R
F,
indicates the proportion of the in-place hydrocarbons ex-
pected to be recovered. To convert in place volumes to reserves we need to multiply
the STOIIP by the recovery factor so that:
Reserves = STOIIP x R
F
(3)
The line over the various terms indicates the average value for these spatial
parameters.
ThereservoirareaA,willvaryaccordingtothecategory;proven,probableorpos-
sible,thatisbeingusedtodefnethereserves.
Before examining the contributions of the various parameters it is worthwhile to
give consideration of the evolution of the reserve estimate during the exploration
and development stage.
5.2 Evolution of the Reserve Estimate
Figure 7 gives a cross section view of a reservoir structure as suggested from seismic
and geological data.

Oil
Suggested 0il and water contact
Figure 7 Cross Section Interpretation From Seismic and Geological Data.
Using this data and possible suggested structure we can carry out some oil in place
calculationsandestimatereserves.Thesefgureshoweverarenotadmissibleinpublic
reserve estimates. They are useful inside the company to justify project expenditure!
Thequestioniswheredowelocatethefrstexplorationwellandgetinvolvedinlarge
exploration expenditure costs. Figure 8 suggest three alternatives
1

Oil
Suggested oil and water contact
Suggest this location.
Figure 8 Alternative locations of Exploration Wells
Infgure9anexplorationwellhasbeendrilledandacorerecoveredandthestruc-
tureofthefeldwithrespecttoformationsandcontactsredefned.Theredefned
structure can now be used to provide an estimate of reserves according to the
three,proven,probableandpossibleperspectives.Figure10

Oil and water contact
Oil
Cored interval
Figure 9 Interpretation After Exploration Well Drilled and Cored.
Introduction To Reservoir Engineering
Insitute of Petroleum Engineering, Heriot-Watt University
1

Oil
P
o
s
s
i
b
l
e
P
r
o
b
a
b
l
e
P
r
o
b
a
b
l
e
P
o
s
s
i
b
l
e
Proved
Figure 10 After The Exploration Well Was Drilled.
Subsequentappraisalwellsarenowdrilledtogivebetterdefnitionofthereserves
ofthefeld.Well2aimedatdefningthefeldtotheleftidentifessomeadditional
isolatedhydrocarbonstructurewithitsownoilwatercontact.Figure11.Thewell,as
wellasincreasingtheprovenreserves,furtheridentifespreviousunknownreserves.
Thenextappraisalwellisaimedatdefningthereservesintheotherdirection.Dur-
ingwelltestingonwells1or2indicationsoffaultingarealsohelpingtodefnethe
fowingnatureoftheaccumulation.Figure12forthefurtherappraisalwellconfrms
theaccumulationtotherightandalsoidentifestheimpactofthefaultwithanew
oil water contact. Subsequent appraisal wells and early development give greater
defnitiontothefelddescription.Figure13

Oil
Proven
Well 2. Well 1. Proposed
delineation
well 3.
P
r
o
v
e
n
Initial appraisal stage.
Figure 11 Further Delineation Well.
0

Oil
Proven
Well 2. Well 1. Well 3.
P
r
o
v
e
n
New oil water contact.
Gas
Figure 12 After Further Appraisal.

Oil
Proven
Well 2. Well 1. Well 3.
P
r
o
v
e
n
New oil water contact.
Well 4.
Gas
Figure 13 Final Appraisal Well.
Fromadeterministicperspectivethevariousreserveestimates,thatis,proven,
probable and possible can be further determined. The indication of the various
elements based on the top structure map are shown. Figure 14
Introduction To Reservoir Engineering
Insitute of Petroleum Engineering, Heriot-Watt University
1

Possible
Probable
Proved
1
2
3
4
Figure 14 Reserves Uncertainties by Deterministic Method.
5.3 Reservoir Area
The reservoir area can be obtained by separately evaluating the individual units
making up the reservoir as obtained from various reservoir maps. These maps are
derived from the evidence given from seismic and subsequent drilled wells. The maps
generally indicate the upper and lower extent of the reservoir section or sections and
theaerialextentasdefnedbyfaultsorhydrocarboncontacts.Figure15showsan
aerialsectionwiththedefnedlimits.Thecontourlinesarelinesofconstantsubsea
depths. Figure 16 gives a cross section of a reservoir unit. The combination of the
tworepresentationsoftheunit(s)canbeusedtocalculatethegrossrockvolume.

Porosity
Boundary
F
a
u
lt B
o
u
n
d
a
ry
F
a
u
lt B
o
u
n
d
a
ry
Fluid
Contact
Figure 15 Structure Contour Map.
7


R
e
s
e
r
v
o
i
r
R
o
c
k

V
o
l
u
m
e
Hydrocarbon Water
Contact Elevation
Heighest Elevation
on Top Structure
Heighest Elevation
on Base Structure
C
o
n
t
o
u
r

E
l
e
v
a
t
i
o
n
(
u
n
i
t
s

s
s
)
Area Contained by Contour
T
o
p

S
t
r
u
c
t
u
r
e
o
B
a
s
e
S
t
r
u
c
t
u
r
e
Figure 16 Reservoir cross section.
7
Figures 17 & 18 show an example of a top structure map and cross section of the
RoughGasfeldintheNorthSea.

47/7 A4
A2
47/8-1
47/8-2
47/2 47/3
47/8
A3
A6
A5
x
x
x
G
w
C
G
w
C
9
5
5
0
9
5
0
0
9
5
0
0
9
5
0
0
9
6
0
0
9
4
5
0
9
4
0
0
9
3
5
0
9
3
0
0
9
2
5
0
9
2
0
0
9100
9
1
5
0
9
3
5
0
9
3
0
0
9
2
5
0
9
2
0
0
8
8
8
Platform A
Completed Producers
Proposed Well Locations
Abandoned Wells
C.I. = 50ft.
8
8
8
8
8
B
8
8
8
A
A
A
A
A
Figure 17 Top Sand Structure Map Rough Gas Field.
5
Introduction To Reservoir Engineering
Insitute of Petroleum Engineering, Heriot-Watt University


9000
9200
9400
9600
9800
A2
A3
A5
A1 A4
Depth (ft)
subsea
Carboniferous
Sands
Tentative
hydrocarbon/
water contact
F
a
u
l
t F
a
u
l
t
U
n
c
o
n
fo
r
m
ity
R
o
tlie
g
e
n
d
e
s U
n
c
o
n
fo
rm
ity
Figure 18 Schematic Cross Section of The Rough Field.
5
5.4 Reservoir Thickness
Another representation of the reservoir formations is the reservoir thickness map.
Where the areal contour maps show the thickness normal to the plane of the reservoir
the contours are called isopachs. When the thickness is mapped as a vertical thickness
then the contour is called an isochore. Not all the reservoir thickness will contrib-
utetofuidrecoveryandwillincludenon-productivestrata.Thosecontourswhich
include these non-productive material are called gross reservoir isopach and those
where non-productive material is excluded are called net reservoir isopach maps.
Thoseintervalscontributingtofowaretermedpay.Theratioofnettogross,h
n
/h
t
,
is an important aspect in reservoir evaluation. Figure 19 shows a net pay thickness
isopachandtheisopachmapfortheRoughfeldisshowninfgure20

0
150
125
100
75
Isopach C I
25 Units
Figure 19 Net Pay Thickness Isopach.
7


1
0
0
1
0
0
9
0
80
70
1
1
0
1
1
0
1
1
6
120
G
w
C
G
w
C
130
140
A4
A1
A2
47/8-1
47/8-2
47/2 47/3
47/7 47/8
A3
A6
A5
x
x
Figure 20 Rough Field Isopach.
5
Theisopachmapcanalsobeusedtocalculatereservoirvolume.Forexampleinfgure
21 the area under a plot of net pay thickness vs. area contained within the contour
provides a net pay volume. These plots can be generated for each section or rock
type. The thickness plots for each section are called isoliths.

OWC
Area Enclosed = Net Rock Volume
Area Contained by Contour
N
e
t

P
a
y

I
s
o
p
a
c
h

V
a
l
u
e
0
40
80
120
140
180
Figure 21 Hydrocarbon Volume From Net Pay Isopach.
7
5.5 Reservoir Porosity
Thevariationofporositycanalsoberepresented.Theaverageporosity,,inawell
can be calculated from the thickness-weighted mean of the porosities
4
.
Introduction To Reservoir Engineering
Insitute of Petroleum Engineering, Heriot-Watt University

w
k n k
k
m
n
h
h


,
1
(4)

where
k
is the average porosity derived from the log over a small thickness h
n,k

withinthenetpaythickness,h
n
.
These values of porosity can then be plotted to generate an isoporosity map as il-
lustrated in fgure 22. The example of an isoporosity map for the Rough Field is
showninfgure23.

5
10
15
20
25
Porosity C I
5%
Figure 22 Iso Porosity Map.
7

1
4
%
1
2
%
1
0
%
8
%
6
%
G
w
C
G
w
C
A4
A1
A2
A3
A6
A5
A
47/7
47/8-1
47/8-2
47/2 47/3
47/8
x
x
Figure 23 Rough Field Iso Porosity Map.
7

5.6 Water Saturation, S


w
Thewatersaturationinareservoirisinfuencedbythecharacteristicsofthereservoir
rock and the location with respect to the position above the free water level near
theoil-waterorgas-oilcontact(seesectionReservoirRockPropertiesChapter7).
The average water saturation S
w,w
,canbecalculatedinasimilarwaytoporosityby
calculating the volume weighted mean across the producing elements of the forma-
tion,thepay.

S
w,w

S
w, k

k
h
n,k
k 1
m

w
h
n
(5)
The values of S
w,w
canbeplottedandcontoursofconstantsaturation(isosaturation)
presented. Figure 24.

15 20
25
30
35
40
WOC
Shale
Figure 24 IsoSaturation(sw)Map.
4
A more detailed description together with exercises are given in the mapping section
of the geology module.
5.7 Formation Volume Factors Oil, B
o
and Gas, B
g
Thesepropertiesoftheoilandgaswhichconvertreservoirvolumestosurfacevolumes,
aregeneratedfrommeasurementsmadeonfuidsamplesfromthereservoir.Theydo
notvarysignifcantlyacrossthereservoirwhencomparedtotheotherrockrelated
parameters. These parameters are covered in the gas properties and oil properties
chapters. In some reservoirs where the formations are thick there is a compositional
gradient over the depth. This variation in composition from heavier (less volatile
components) to lighter components at the top results in a variation of the oil forma-
tionvolumefactor,B
o
over the thickness. In such cases an average value based on
values measured or calculated at depth would be a preferred value.
Introduction To Reservoir Engineering
Insitute of Petroleum Engineering, Heriot-Watt University

5.8 The Recovery Factor, E


R
The proportion of hydrocarbons recovered is called the recovery factor. This fac-
torisinfuencedbyawholerangeoffactorsincludingtherockandfuidproperties
andthedrivemechanisms.Thevariabilityoftheformationcharacteristics,thehet-
erogeneitycanhavealargeinfuenceonrecovery.Thedevelopmentprocessbeing
implemented and the geometries and location of wells again will also have a large
infuence.Calculatingrecoverythereforeintheearlystagesisnotfeasibleandmany
assumptions have to be included in such calculations. It is in this area that reservoir
simulationcangiveindicationsbutthequalityofthecalculatedfgureislimitedby
the sparse amount of quality data on which the simulation is based.

The American Petroleum Institute
6
hasanalysedtherecoveriesofdifferentfeldsand
correlations have been presented for different reservoir types and drive mechanisms.
Figures25and26givetheresidualsaturationsandoilrecoveryeffciencesfordif-
ferentdrivemechanisms.TheAPIalsopresentscorrelationsforrecoveries,E
R
,
For sandstone and carbonate reservoirs with solution gas drive
E
S
B
k
S
p
p
R o
w
ob ob
w
b
a
,
.
.
.
.
.

( )
,

,
,
]
]
]
]
j
(
,
\
,
(
( )
j
(
,
\
,
(
0 4185
1
0 1611
0 0979
0 3722
0 1741

(6)
For sandstone reservoirs with water drive
E
S
B
k
S
p
p
R o
w
oi
wi
oi
w
o
i
a
,
.
.
.
. .

( )
,

,
,
]
]
]
]
j
(
,
\
,
(
( )
j
(
,
\
,
(

0 54898
1
0 2159
0 0422
0 0770
1903

(7)
breferstobubblepointconditions,iistheinitialconditionanda,referstoabandonment
pressure.


1.00
0.50
0.10
0.05
0
2
1.00
0.50
0.10
0.05
5 10 20 30 40 50 60 70 80 95 98
2 5 10 20 30 40 50 60 70 80 95 98
0
M
E
D
I
A
N
+
S
o
r

(
O
R

S
g
r
)

a
s

F
r
a
c
t
i
o
n

o
f

T
o
t
a
l

P
o
r
e

S
p
a
c
e
RESIDUAL SATURATIONS
PERCENTAGE OF CASES LARGER THAN
S
or
In Water Drive
Reservoirs
S
gr
In Solution Gas Drive
Reservoirs

Figure 25 Log - Probability Residual Oil Saturation For Water Drive and Solution Gas
DriveReservoirs.(API
6
)

1.00
0.50
0.10
0.05
0
2
1.00
0.50
0.10
0.05
5 10 20 30 40 50 60 70 80 95 98
2 5 10 20 30 40 50 60 70 80 95 98
0
M
E
D
I
A
N
+
O
I
L

R
E
C
O
V
E
R
Y

E
F
F
I
C
I
E
N
C
Y

A
T

F
I
E
L
D

A
B
A
N
D
O
N
M
E
N
T
I
N

P
E
R
C
E
N
T

O
F

O
I
L

P
L
A
C
E
RESIDUAL SATURATIONS
PERCENTAGE OF CASES LARGER THAN
Water Drive
Gas Cap Drive
Solution Gas Drive
Gas Cap Drive +
Water Injection

Figure 26 Log-ProbabilityofOilRecoveryForVariousDriveMechanisms.(API
6
)
Introduction To Reservoir Engineering
Insitute of Petroleum Engineering, Heriot-Watt University

5.9 Production Capability


Anotherconcept,isocapacity,isusedtosignifyproductioncapability.Isocapacity
denotes equal values of permeability-net thickness product. This product can be
mappedinsteadofpermeability.Thefgure27showsanisocapacitymapwherethe
absolutepermeabilityhasbeenobtainedasanarithmeticaverageinthezone.

0.25
0.5
1
2 3 4 5 4
3 2
1
Figure 27 Isocapacity Map.
7
ThepermeabilitymapfortheRoughFieldisgiveninfgure28

A4
A3
A2
47/8-1
47/8-2
47/2
A6
A5
x
x
G
w
C
G
w
C
Platform B
80
100
120
60
40
0
Contour Intervals 20 millidarcies
47/7 47/8
Figure 28 Rough Field Permeability Map.
5
0
5.10 The Hydrocarbon Pore Volume Map
The hydrocarbon pore volume can be obtained by combining the net rock volume with
a mean porosity and a mean hydrocarbon saturation. An alternative is the mapping of
hydrocarbonthickness(HPT)ateachwell.HPTatawellinagivenzoneis:

HPT h S
n h

_ _
. .
(8)
where:

S
h
_
1 S
w
_
Figure29givesanHPTmapandtheRoughFieldHPTmapisgiveninfgure30

0
9
10
11
12
13
14
15
14
13
12
11
10
0
Figure 29
7
Hydrocarbon Pore Thickness Map.

A4
A2
A3
A1
A6
A5
9
10
0
8
7
6
5
4
Figure 30 Rough Field Hydrocarbon Pore Thickness.
5
Introduction To Reservoir Engineering
Insitute of Petroleum Engineering, Heriot-Watt University
1
6. OTHER APPRAISAL ROLES
In building up the picture to enable the reserves estimates and recoveries to be
determined the reservoir engineer will be involved in an number of aspects. One of
the most powerful tools is the production test.
In a well test an exploration or appraisal well is converted to a short term producing
well,withalltheassociatedfacilitiesputinplacetohandletheproducedfuidsand
monitorfuidrates.Adownholepressuremonitoringdeviceisalsolocatedinthe
well.Figure31.Thewellisfowedataconstantrate,andsometimestworatesas
illustratedinfgure32a,atworatetest.Thedownholepressuredevicerespondsto
the production and pressure declines. After a short or longer time period depending
onthenatureofthetest,thewellisshutin,i.e.thefowisstopped.Inthewellthe
pressurebuildsupandeventuallyasmonitoredbythedownholepressuredevice,
recovers to the original pressure. Figure 32b. It is in the analysis of the pressure
drawn down and build up curves and the rates that the reservoir engineer is able to
determinethefowabilityofthereservoir.Ifthefowingintervalthicknessisknown,
the permeability can be calculated. The presence of faults can also be detected.
A considerable amount of reservoir data can be obtained from these well tests
sometimescalledDSTs(drillstemtests).Ithasbeenthepractiseoverrecentyears
fortheproducedfuidstobefaredsincethereisunlikelytobeaninfrastructureto
collectthesefuids.Nowthatcompaniesaremovingtoazeroorreducedhydrocarbon
emission policy the nature and facilities required for these tests are changing. A
featureofthefaringapproachisapublicdemonstrationoftheproductivityofthe
well being tested.


Surface casing
Cement
Perforations
Production casing
Production tubing
Packer
Down hole
pressure monitor
Figure 31 Production Test Assembly.
Introduction To Reservoir Engineering
Insitute of Petroleum Engineering, Heriot-Watt University


q

b
b
l
s

/

d
a
y
P
f
.

p
s
i
g
Pressure build up
Well shut in Flow 1 Flow 2
Pressure draw down
Pi
t
t
Figure 32 Production Test Analysis. Two Rate Test.
Well test analysis is a powerful reservoir engineering tool and is treated in depth in
a subsequent module of the Petroleum Engineering course.
Thenatureofthefuidsiskeytoreservoirbehaviourandalsosubsequentprocessing
in any development. The collection and analysis of these fuids is an important
role and is at the focus of PVT analysis. This topic is covered in Chapter 14 PVT
Analysis. The pressure profle in a well is another important aspect of reservoir
characterisation and can be used to identify fuid contacts.When used during the
earlystagesofproductionitcanbeapowerfulmeansofrefningthestructureand
hydrodynamic continuity characteristics of the reservoir. This is covered in the next
chapter. Like PVT analysis where the information is based on samples removed
from the reservoir, core analysis is based on recovered core from the formation.
Varioustestsonthismaterialanditsreactiontovariousfuidsprovidesmanyofthe
reservoir engineering parameters important in determining the viability of a project.
Core analysis also provides a cross check for indirect measurements made downhole.
These core analysis perspectives are covered in chapters 7 and 8.
It is clear from what we have discussed that reservoir engineering is an important
function in the appraisal of the reservoir. The focus for this appraisal so far has con-
centratedondeterminingthecharacteristicsandpotentialfowbehaviourofareservoir
under development. Clearly there could be a whole range of possibilities with respect
totheplanthatcouldbeusedtodevelopthefeld.Thisdevelopment planning per-
spective is an important part of the reservoir engineers role. Again it is a team effort

involvingthegeologicalcommunitywhounderstandthereservoirandthevarious
engineers who have the responsibilities of designing and operating the hardware to
enable production. An important part of any future development are the facilities that
would be required for sustained production and its is therefore an important part of
the appraisal stage to provide data for those who would have responsibility for good
quality data predictions which will enable optimised facility design.
In any project new data is always being generated. Indeed for a reservoir, its
characteristics are unlocked over the whole lifetime of the project. The duration of
theappraisalstageclearlyisatechnoeconomicdecisionrelatedtotheconfdence
to go ahead based on a good foundation of quality data and forecasts. Fine tuning
can always be carried out but this is costly if this delays the development stage. It
is important to identify and fll the gaps for the largest uncertainties, and having
suffcient information to design a system which is safe and cost effective. The
diffculty is making the decision on the data under which a line is drawn which
defnesthebasisforfelddevelopmentdesign.Inreservoirdevelopmentthereservoir
isalwaysrevealingitsproperties,indeeditisintheproductionphasethatthetrue
characteristics are revealed.
7 DEVELOPMENT PLANNING
7.1 Reservoir Modelling
Givenappraisalwelldata,andtestresultsthereservoirengineercanconsidersome
alternativedevelopmentplans,relyingheavilyonexperienceandinsight.Sincethe
80scomputerbasedreservoirsimulationhasplayedamajorrole.
Thestartingpointwillinvariablybeareservoirmapusedtocalculatereserves,but
inadditionusewillbemadeofthematerialbalanceequation(chapter15),together
with some drive concepts (chapter 11), to predict reservoir behaviour. One of the
problems faced in making predictions is to adequately take into account knowledge
aboutgeologicaltrendsand,althoughindividualwellmodelscanbeadjustedtorefect
localconditions,thereisnopracticaldeskcalculatortechniqueforusingsay,the
material balance equation and well models to come up with a predictive reservoir
performance. Displacement models such as those derived by Buckley and Leverett
(chapter 18), mainly from observations in the laboratory, give some insight into
reservoirbehaviourbutagaindonotsignifcantlyassistinallowingtheengineerto
study the effect of alternative development plans on a heterogeneous reservoir.
With insight and ingenuity, the reservoir can be divided into a number of simple
units that can be analysed by the traditionally available techniques but such an
approach remains unsatisfactory. Over recent years the integration of geological and
geophysicalperspectivesiscontributingconsiderablytotheconfdenceinreservoir
modelling.
7.2 Technoeconomics
For hydrocarbon accumulations found on dry land the traditional reservoir engineering
techniquesavailableforfelddevelopmentplanningwere,infact,quiteadequate.This
is mainly so because land development operations offer a high degree of planning
Introduction To Reservoir Engineering
Insitute of Petroleum Engineering, Heriot-Watt University

fexibilitytooilcompaniesandhenceallowthemtomakeoptimaluseofthelatest
information.Inanoffshoreenvironmentthisisnotthecase;onceplatformshave
beenorderedmostdevelopmentoptionsareclosed.Itiswithrespecttooffshorefeld
development planning that reservoir simulation models have found their greatest
application potential.
7.3 Coping with Uncertainty
The challenge to the exploration & production business of the oil & gas industry is
considerable. The looking for the needle in the haystack scenario is not too far from
thetruth,whencomparedtootherindustrialsectors.Withthechallengeofreserves
being found in technically challenging areas and the oil price moving in response
topoliticalaswellasdemandscenarios,thereistheneedtodefnemoreaccurately
forecasts of production and recovery. Reducing uncertainty is the message of the
current decade and not least in reservoir engineering and its related disciplines.
It is clear from what we have overviewed in this chapter and the topics which will be
covered in the subsequent chapters that there are many parameters which contribute
to the viability of the various aspects of successful oil and gas production. It is also
clearthatthevariousformsofdatarequired,theconfdenceintheabsolutevalues
varyaccordingtothetype,andthereforethefnalimpactonthefnalresultwillvary
according to the particular parameter.
The following list summarises some of the principal uncertainties associated with
the performance of the overall reservoir model. The type of data can for example
be subdivided into two aspects static and dynamic data .
Static Properties
Reservoir structure
Reservoir properties
Reservoir sand connectivity
Impact of faults
thief sands
Dynamic Properties
Relative permeability etc
Fluid properties
Aquifer behaviour
Wellproductivity(fractures,welltype,condensatedropoutetc.)
Theimpactofeachoftheseparameterswillvaryaccordingtotheparticularfeldbut
it is important that the company is not ignorant of the magnitude of the contributing
uncertainties,sothatresourcescanbedirectedatcosteffectivelyreducingspecifc
uncertainties. Figure 33 illustrates an outcome which might arise from an analysis
ofvariousuncertaintiesforaparticularfeld.Itdemonstratesforthisparticularfeld
andatthetimeofanalysistheimpactofthevariousdatahasonthefnalprojectcost.
Clearly in this case the aquifer behaviour uncertainties has the least impact whereas
reservoirstructureandwellproductivityuncertaintieshadthemostsignifcant.An-
other feld would result in different impact perspectives, and therefore a different
strategy to reduce overall project uncertainty would be required.


Q
P
Project
Cost
Changes
- +
Well
production Reservoir
area
Reservoir
structure
Sand
conectives
Thief zones
Faults
Fluid properties
Relative
permeabilities etc.
Aquifer
behaviour
Figure 33 Impact on a Project of Different Uncertainties
8 PRODUCTION OPERATIONS OPTIMISATION
8.1 The Development Phase
The development phase covers the period from the time continuous production
startsuntiltheproductionfromthefeldstopsi.e.abandonment.Thedecisionwhen
to stop production clearly is a techno-economic decision based to a large extent on
the costs of the development. Low volume producers can be allowed to continue in
an onshore development where well operating costs might be low but the high costs
associated with for example in an expensive offshore operation sets a much higher
economiclimitforthedecisiontoabandonafeld.
During the development phase Dake
2
hasidentifedanumberofrolesfortheReservoir
Engineering which are targeted at optimising production. It is an irony that some
of the best data is generated during the production phase. Through production the
reservoirunveilsmoreofitssecrets.Someofthesemaycausemodifcationstothe
development,perhapsindefningnewwelllocations.Thenatureofthehydrodynamic
continuity of the reservoir is mainly revealed through pressure surveys run after a period
ofproduction.Thismaydefnezonesnotbeingdrainedandthereforemodifcations
to the well completions might result.
As production progresses fuid contacts rise and therefore these contacts need to
bemonitoredandtheresultsusedtodecide,forexample,torecompleteawellasa
resultof,forexampleexcessivewaterproduction.Asispointedoutinthechapteron
reservoirpressure,developmentwellsbeforetheyarecompletedprovideavaluable
resource to the reservoir engineer to enable surveys of pressure to be run to provide
adynamicpressure-depthprofle.
Introduction To Reservoir Engineering
Insitute of Petroleum Engineering, Heriot-Watt University

8.2 History Matching


Throughout the production phase the comparison of the actual performance with that
predicted during the appraisal stage and more recent predictions is made. It is during
this stage that the quality of the reservoir simulation model comes under examina-
tion. The production pressure decline is compared to that predicted and the reservoir
simulation model adjusted to match. This process is called history matching. Clearly
ifthesimulationcannotpredictwhathashappenedovertherecentpastitcannot
beusedwithmuchconfdencetoforecastthefuture!
More simple approaches not requiring the resources of a complex simulator can also
beusedtoupdateearlypredictions,forexamplematerialbalancestudies.
Onceproductionhasbeenobtained,theadditionaldatabecomesavailableandmakes
animportantcontributiontotherefningoftheinitialreservesestimates.Twotech-
niques historically used are decline curve analysis and material balance studies.

In material balance studies, the pressure-volume behaviour of the entire feld is
studiedassuminganinfnitepermeabilityforthereservoir.Byassuminganinitial
oil-in-placefromvolumetriccalculations,thepressureisallowedtodeclinefollowing
fuidwithdrawal.Thisdeclineismatchedagainsttheobservedpressurebehaviour
and,ifnecessary,theoriginaloil-in-placefgureismodifeduntilamatchisobtained.
Inthepresenceofawaterdrive,additionalvariablesareincludedbyallowingwater
infuxintothetank.Waterinfuxisgovernedbymathematicalrelationshipssuch
asvanEverdingenandHurst(TheseconceptsarecoveredinChapters11,12,and
13MB/MBApplicationsandWaterInfux).
Decline curves are plots of rate of withdrawal versus time or cumulative withdrawal
on a variety of co-ordinate scales. Usually a straight line is sought through these ob-
servations and extrapolated to give ultimate recovery and rates of recovery. Decline
curves only use rates of withdrawal and pay relatively little attention to the reservoir
andfowingpressures.Achangeinthemodeofoperationofthefeldcouldchangethe
slopeofthedeclinecurve;hence,thisisoneoftheweaknessesofthistechnique.
Anoteworthyfeatureofthesetwoapproachesisthattheengineerinfactftsasim-
ple model to observe data and uses this model to predict the future by extrapolation.
Asmoredatabecomesavailablethemodelgetsupdatedandpredictedresultsare
adjusted.Declinecurveanalysishasnotbeenusedtothesameextentasinthe60s
and70s.Withthepowerofcomputingandtheeffortsmadetointegrategeological
understanding,thephysicsofthefowandbehaviourofrockandfuidsystemsinto
reservoirsimulation,thefttingandtheuncertaintyofearliermethodsarebeing
superseded by integrated reservoir simulation modelling.
The routine company function will generate the need for on going production pro-
fleupdates.Thegenerationoftheseisgenerallytheresponsibilityofthereservoir
engineer,whomightchosesimpleanalyticalapproachestothemorecostlyreservoir
simulation methods.
8.3 Phases of Development
During the development there are a number of phases. Not all of these phases may
be part of the plan. There is the initial production build up to the capacity of the facil-

ity as wells are brought on stream. There is the plateau phase where the reservoir is
produced at a capacity limited by the associated production and processing facilities.
Different companies work with different lengths of the plateau phase and each project
will have its own duration. There comes a point when the reservoir is no longer able
todeliverfuidsatthiscapacityandthereservoirgoesintothedecline phase. The
declinephasecanbedelayedbyassistingthereservoirtoproducethefuidsbythe
useofforexampleliftingtechniquessuchasdown-holepumpsandgaslift.The
declinephaseisoftenadiffcultperiodtomodelandyetitcanrepresentasignifcant
amountofthereserves.Thesephasesareillustratedinfgure34

Build up phase
Plateau phase
Decline phase
Artificial lift
Time - years
P
r
o
d
u
c
t
i
o
n

r
a
t
e
Economic limit

Figure 34 Phases of Production.
The challenge facing the industry is the issue of the proportion of hydrocarbons left
behind.Theabilitytoextractagreaterproportionofthein-placefuidsisobviously
a target to be aimed at and over recent years recoveries have increased through the
application of innovative technology. Historically there have been three phases of
recovery considered. Primary recovery,whichisthatrecoveryobtainedthroughthe
natural energy of the reservoir.
Secondary recovery is considered when the energy is supplemented by injection of
fuids,forexamplegasorwater,tomaintainthepressureorpartiallymaintainthe
pressure.The injected fuid also acts as a displacing fuid sweeping the oil to the
producing wells. After sweeping the reservoir with water or gas there will still be
remainingoil;oilatahigh saturationwherethewaterforarangeofreasons,for
example; well spacing, viscosity, reservoir characteristics to name just a few, has
by-passedtheoil.Theoilwhichhasbeencontactedbytheinjectedfuidwillnotbe
completely displaced from the porous media. Because of characteristics of the rock
andthefuidsaresidual saturationoffuidisheldwithintherock.Bothofthese
unrecoveredamounts,theby-passedoilandtheresidualoilareatargetforenhanced
recovery methods, EOR.

Much effort was put into enhanced oil recovery (EOR) research up until the mid
seventies. Sometimes it is termed tertiary recovery. When the oil price has dropped
the economics of many of the proposed methods are not viable. Many are based on
Introduction To Reservoir Engineering
Insitute of Petroleum Engineering, Heriot-Watt University

the injection of chemicals which are often oil based. The subject of EOR has not been
forgotten and innovative methods are being investigated within the more volatile
oil price arena. Figure 35 gives a schematic representation of the various phases of
development and includes the various improved recovery methods. More recently
a new term has been introduced called Improved Oil Recovery (IOR). IOR is more
looselydefnedandcoversallapproacheswhichmightbeusedtoimprovetherecov-
eryofhydrocarbonsinplace.ClearlyitisnotasspecifcasEORbutprovidesmore
of an achievable target than perhaps some of the more sophisticated EOR methods.
As we have entered into the next millennium it is interesting to note that a number
of major improved recovery initiatives are being considered particularly with respect
to gas injection. One perspective which make a project more viable is that of the
disposalofgasforexamplewhichisanenvironmentalchallengeinonefeldcan
bethesourceofgasforanotherfeldrequiringgasforagasinjectionimprovedoil
recovery process.

Primary
Recovery
Artifical Lift
Pump gas lift etc.
Secondary
Recovery
Natural
Flow
Tertiary
Recovery
Pressure
Maintenance
Water, gas injection
Natural
Flow
Thermal Gas Chemical Microbial
Steam In-situ
combustion.
Hydrocarbon
miscible, CO2
N2 immiscible
gas
Polymer
surfactant/
polymer
E
O
R
C
O
N
V
E
N
T
I
O
N
A
L
Figure 35 Oil Recovery Mechanisms.
9. THE UNIQUENESS OF THE RESERVOIR
As we have discussed the role of the reservoir engineer in combination with other
disciplines is to predict the behaviour of the reservoir. Whereas in the early years of
oil exploration little attention was paid to understanding the detailed characteristics
ofthereservoir,itisnowrecognizedthatdetailedreservoirpropertiesassociatedwith
oftencomplexphysicalandchemicallawsdeterminefeldbehaviour.Theunlocking
of these characteristics and understanding the laws enable engineering plans to be
put in place to ensure optimised developments are implemented. This is schemati-
callyillustratedinfgure36.
0

Reservoir
Behaviour
Development
Plan
Reservoir Description
Unique
Dynamic and Static
Figure 36 RelationshipbetweenReservoirDescription,andReservoirBehaviour.
Atoneextremeforexampleinablow-outsituation,areservoirproducesinanun-
controlledmanneronlyrestrictedbythesizeofthewellthroughwhichisproducing.
Optmised development however based on a thorough understanding of the reservoir
enablesthereservoirtobeproducedinacontrolled,optimisedmanner.
In many other industries the effort expended on one project can be utilised in engi-
neeringaduplicateorasimilarsizeunitelsewhere.Suchopportunitiesarenotpos-
sible in the engineering of a reservoir. Reservoirs are unique in many aspects. The
compositionofthefuidsareunique,therockcharacteristicsandrelatedproperties
areunique,thesizeandshapeareuniqueandsoon.Fromourperspectivethisreser-
voir description is dynamic as the reservoir over a period of time gives up its secrets.
Fromthereservoirsperspectivehoweverthedescriptionisstatic,exceptwiththe
changesresultingfromtheimpactoffuidproductionorinjection.Thechallenge
tothoseinvolvedisreducingthetimeittakesforourdynamicdescriptiontomatch,
our static description known only to the reservoir or whoever was responsible for
its formation! The answer perhaps is more of a philosophical nature. The reality is
showninfgure37wherethetopstructuremapforaNorthSeagasfeldwithaten
year gap shows the impact of knowledge gained from a number of wells as against
that interpreted from the one well. Considerable faulting is shown not as a result of
major geological a activity over the ten years but knowledge gained from the data
associated with the new wells.
Introduction To Reservoir Engineering
Insitute of Petroleum Engineering, Heriot-Watt University
1

49/26.1
200
200
210 220
220
5310 5310
5305 5305
SHELL/ESSO 49/26 AMOCO 49/27
Gas /water contact
Depths in metres
scale 1 100,000
2
0
0
0
2
1
0
0
2
2
0
0
1
0
0
0
1
0
0
0
2
0
0
0
8
0
0
2
1
0
0
2
0
0
0
2
0
0
0
1
0
0
0
2
0
0
0
1
0
0
0
2000
2
0
0
0
1
0
0
0
2
1
0
0
1
2
0
0
1
0
0
0
2
1
0
0
Present interpretation of Leman Gas-field, showing contours on top of Rotliegendes in feet below sea-level
The Leman field as it appeared to be when the exploration well was drilled
Figure 37 (a) The Leman Field as it Appeared to be When The Exploration Well Was
Drilled.

Depth in feet
0 1
0 1 2
Miles
KMS
Gas /water contact
A permanent platform
5305 5305
5300
5310
5300
5310
200
200 210
210
220
220
230
230
6
9
0
0
6400
SHELL/ESSO 49/26 AMOCO 49/27
Present interpretation of Leman Gas-field, showing contours on top of Rotliegendes in feet below sea level.
Leman field ten years after discovery
7
0
0
0
6
9
0
0
6900
6
3
0
0
6
3
0
0
7
0
0
0
6
9
0
0
6
9
0
0
6300
6400
6300
6300
6
2
0
0
6
1
0
0
6
4
0
0
6
9
0
0
Figure 37b Leman Field Ten Years After Discovery
The coverage of the reservoir has also changed effecting the equity associated with
theblocks.Thisillustratestheearlybeneftstobegainedfromdrillinganumberof
explorationwells.Theseequityagreements,arecalledunitisationagreementsandsuch
agreements are shortened when good quality and comprehensive reservoir descrip-
tiondataisavailable.Clearlytherecanneverbesuffcientdescription,howeverthe

economics of project management will determine when decisions have to be taken


based on description to date. The value of extra information has to be balanced by
the cost of delay in going ahead with a project.
10. CONCLUSION
In order to accomplish these objectives the Petroleum Reservoir Engineer should
have a broad fundamental background both theoretically and practically in the basic
sciences and engineering. The basic areas are:

(i) Thepropertiesofpetroleumreservoirrocks
(ii) Thepropertiesofpetroleumreservoirfuids
(iii) Thefowofreservoirfuidsthroughreservoirrock
(iv) Petroleumreservoirdrivemechanisms
It is also important that the Petroleum Reservoir Engineer has a thorough basic
understandingingeneral,historicalandpetroleumgeology.Theinfuenceofgeological
history on the structural conditions existing in a reservoir should be known and
considered in making a reservoir engineering study. Such a study may also help to
identifyandcharacterisethereservoirastoitsaerialextent,thicknessandstratifcation
andthechemicalcomposition,sizedistributionandtextureoftherockmaterials.
Inhislatesttext,Dake
2
comments on some of the philosophy of approach to reser-
voirengineering,andidentifestheimportanceofpinningdowninterpretationand
prediction of reservoir behaviour to well grounded laws of physics.
Reservoir forecasting has moved on considerably since wells were drilled with little
interest and concern into the production and forecasting of what was happening in
the reservoirs thousands of feet below. The approach to coping with uncertainty as
jokinglyrefectedinthecartoonbelow,(Figure38)isnolongerthecaseassophisti-
catedcomputationaltoolsenablepredictionstobemadewithconfdenceandwhere
uncertaintyexiststhedegreeofuncertaintycanbedefned.
Introduction To Reservoir Engineering
Insitute of Petroleum Engineering, Heriot-Watt University


"We feed the geological data for the area, the computer produces a schematic topological
overview designating high probability key points, then we stick the printout on the wall and
Lever throws darts at it."
Figure 38 A Past Approach to Uncertainty!
REFERENCES
1. Craft,B.C.andHawkins,M.F.AppliedReservoirEngineering,Prentice-Hall
Inc. 1959
2. Dake,L.P.,ThePractiseofReservoirEngineering.Elsevier.1994
3. SocietyOfPetroleumEngineers.ReservesDefnitions1995.
4. Chierici,G.L. Principles of Petroleum Reservoir Engineering. Vol 1 Springer
Verlag 1994
5. Hollois,A.P.Somepetroleumengineeringconsiderationsinthechangeoverof
theRoughGasfeldtothestoragemode.PaperEUR295ProcEuropec.1982,
pg 175
6. API.AStatisticalStudyoftheRecoveryEffciency.AmericanPetroleumInstitute.
BullD14,1stEdition,1967
7. Archer,J.S.andWall,C.G.PetroleumEngineeringPrinciplesandPractise,Graham
andTrotman,1986.
Reservoir Pressures and Temperatures
CONTENTS
1 INTRODUCTION
2 ABNORMALPRESSURES
3 FLUIDPRESSURESINHYDROCARBON
SYSTEMS
4PRESSUREGRADIENTSAROUNDWATER-
OILCONTACT
5.TECHNIQUESFORPRESSURE
MEASUREMENT
6.RESERVOIRTEMPERATURE

LEARNING OBJECTIVES
Having worked through this chapter the Student will be able to:
Havingworkedthroughthischapterthestudentwillbeableto:
Defnetheterms;lithostaticpressure,hydrostaticpressureandhydrodynamic
pressure.
Drawthenormalhydrostaticpressuregradientforwatersystems.
Defnenormalpressuredreservoirs,overpressuredreservoirsandunderpressured
reservoirs
Describebriefyandsketchthepressuregradientsassociatedwithoverpressured
andunderpressuredreservoirs.
Describe briefy , sketch and present equations for the pressures in a water
supportedoilandgasbearingformation.
Illustratehowadownholeformationpressuredevicecanbeusedtodiscriminate
permeabilitylayersafterproductionhascommenced.
Commentbriefywhatgeothermalgradientisinareservoirwherefow
processesoccuratconstantreservoirtemperature.
Reservoir Pressures and Temperatures
Institute of Petroleum Engineering, Heriot-Watt University

1. INTRODUCTION
Determiningthemagnitudeandvariationofpressuresinareservoirisanimportant
aspectinunderstandingvariousaspectsofthereservoir,bothduringtheexploration
phasebutalsoonceproductionhascommenced.
Oilandgasaccumulationsarefoundatarangeofsub-surfacedepths.Atthesedepths
pressureexistsasaresultofthedepositionalprocessandfromthefuidscontained
withintheprousmedia.Thesepressuresarecalledlithostatic pressures and fuid
pressures. Thesepressuresareillustratedinfgure1.
Thelithostatic pressureiscausedbythepressureofrockwhichistransmittedthrough
thesub-surfacebygrain-tograincontacts.Thislithostaticorsometimescalledgeostatic
oroverburden pressureisoftheorderof1psi/ft.Thelithostaticpressuregradient
variesaccordingtodepth,thedensityoftheoverburden,andtheextenttowhichthe
rocksaresupportedbywaterpressure.Ifweusethisgeostaticpressuregradientof
1psi/ft.thenthegeostaticpressureP
ov
,inpsigatadepthofDfeetis
p
ov
=1.0D (1)
Thegeostaticpressureisbalancedinpartbythepressureofthefuidwithinthepore
space,thepore pressure,andalsobythegrainsofrockundercompaction.Inun-
consolidatedsands,loosesands,theoverburdenpressureistotallysupportedbythe
fuidandthefuidpressureP
f
isequaltotheoverburdenpressureP
ov
.Indeposited
formationslikereservoirrocksthefuidpressureisnotsupportingtherocksabove
butarisesfromthecontinuityoftheaqueousphasefromthesurfacetothedepthDin
thereservoir.Thisfuidpressureiscalledthehydrostatic pressure.Thehydrostatic
pressureisimposedbyacolumnoffuidatrest.Itsvaluedependsonthedensityof
thewater
w
,whichisaffectedbysalinity.Inasedimentarybasin,wheresediment
hassettledinaregionofwaterandhydrocarbonshavebeengeneratedandtrapped,
wecanexpectahydrostaticpressure.Foracolumnoffreshwaterthehydrostatic
pressureis0.433psi/ft.Forwaterwith55,000ppmofdissolvedsaltsthegradientis
0.45psi/ft;for88,000ppmofdissolvedsaltsthegradientisabout0.465psi/ft.
Itsvariationwithdepthisgivenbytheequation.
P
f
=
w
Dg (2)
wheregistheaccelerationduetogravity.
Thereisanotherfuidpressurewhicharisesasaresultoffuidmovementandthat
iscalledthehydrodynamic pressure.Thisisthefuidpotentialpressuregradient
whichiscausedbyfuidfow.Thishoweverdoesnotcontributetoin-situpressures
atrest.

Pressure (psia)
D
e
p
t
h

(
F
t
.
)
FP GP
Overburden
Pressure (OP)
Underpressure
Overpressure
Normal
14.7
(FP = Fluid Pressure, GP = Grain Pressure)
0
Figure 1 Givestherelationshipbetweenthelithostaticpressureandthehydrostatic
pressure.
1
Fluid pressure inhydrocarbonaccumulationsaredictatedbythe prevailingwater
pressureinthevicinityofthereservoir.Inanormalsituationthewaterpressureat
anydepthis:

P
dP
dD
xD 14.7psia
w
water
=

+
(3)
wheredP/dDisthehydrostaticpressuregradient
Thisequationassumescontinuityofwaterpressurefromthesurfaceandconstant
salinity. In most cases even though the water bearing sands are divided between
impermeableshales,anybreakofsuchsealingsystemswillleadtohydrostaticpres-
surecontinuity,butthesalinitycanvarywithdepth.
Reservoirswhosewaterpressuregradientwhenextrapolatedtozerodepthgivean
absolutepressureequivalenttoatmosphericpressurearecallednormal pressured
reservoirs.
EXERCISE 1
If the average pressure gradient in a region is 0.7 psi/ft, calculate the pore
pressure in a normally pressurised formation at 700ft. Convert the pressure from
psi to KPa, then express the pressure in MPa. What is the pressure gradient in
KPa/m?

Reservoir Pressures and Temperatures


Institute of Petroleum Engineering, Heriot-Watt University

2. ABNORMAL PRESSURE
Undercertainconditions,fuidpressuresmaydepartsubstantiallyfromthenormal
pressure.Overpressured reservoirsarethosewherethehydrostaticpressureisgreater
thanthenormalpressureandunderpressured reservoirsarebelownormalpressure.
Figure1.Theyarecalledabnormal pressured reservoirsandcanbedefnedbythe
equation:

P =
dP
dD
xD + 14.7psia + C
w
water

(4)
whereCisaconstant,beingpositiveforoverpressuredandnegativeforanunder-
pressuredsystem.
Forabnormallypressuredreservoirs,thesandissealedofffromthesurroundingstrata
sothatthereisnothydrostaticpressurecontinuitytothesurface.
Conditionswhichcauseabnormalfuidpressureinwaterbearingsandshavebeen
identifedbyBradley
2
andinclude(Figure2):

Original Deposition
Dense Shale
Shale deposited too
quickly to allow
fluid equilbrium
FP-Too High
Upthrust
Reservoir
North Sea
Glacier
Greenland 3 km thick
1300 psi/1000 m ice
Normal Surface
(a)
(b)
(c)
Figure 2 Causesofoverpressurring
Thermaleffects,causingexpansionorcontractionofwaterwhichisunable
toescape;anincreaseintemperatureof1Fcancauseanincreaseof125psi
inasealedfreshwatersystem.

Rapidburialofsedimentsconsistingoflayersofsandandclay.Speedofburial
doesnotallowfuidstoescapefromporespace.
Geologicalchangessuchasupliftingofthereservoir,orsurfaceerosionboth
ofwhichresultinthewaterpressurebeingtoohighforthedepthoftheburial.
Theoppositeoccursinadownthrownreservoir.
Osmosisbetweenwatershavingdifferentsalinity,thesealingshaleactingasa
semi-permeablemembrane.Ifthewaterwithinthesealismoresalinethanthe
surroundingwater,theosmosiswillcauseahighpressureandviceversa.
OverpressuredreservoirsarecommoninTertiarydeltaicdepositssuchastheNorth
Sea,NigerdeltaandtheGulfCoastofTexas.IntheNorthSeaonemechanismfor
overpressure is the inability to expel water from a system of rapidly compacted
shales.
Withabnormallypressuredreservoirsapermeabilitybarriermustexist,whichinhibit
pressure release. These may be lithological or structural. Common lithological
barriersareevaporatesandshales.Lesscommonaretheimpermeablecarbonates
and sandstones. Structure permeability barriers may result from faults which, in
somecases,seal.The subject on of abnormal pressures is covered more fully in
the Geology Module
Ifreservoirsareallnormalpressuredsystemsthenthepressuregradientforthese
reservoirswouldbevirtuallyallthesame,otherthanfromtheinfuenceofsalinity.
Thefgurebelowshowsthewaterpressuregradientsforanumberofreservoirsin
theNorthSeaandindicatesthesignifcantoverpressuringinthisregion.Oftenthese
overpressuring show regional trends. For example the felds depicted in fgure 3
showanincreaseinabnormalpressureinthesoutheastdirection.Clearlyifallthese
reservoirswerenormallypressuredthenthepressuredepthsvalueswouldlieonthe
samegradientlinewithazerodepthpressurevalueofatmosphericpressure.
Reservoir Pressures and Temperatures
Institute of Petroleum Engineering, Heriot-Watt University
7

4
2
1
3
5
Note: Water gradient lines drawn
through known or projected
oil/water contacts
Alwyn
Lyell
Ninian
OWC
Heather
OWC
Cormorant
OWC
S.W> Ninian
N.W. Alwyn
Thistle OWC
Brent OWC
Statfjord OWC
5000 6000 7000 8000 9000 10,000
13,000
12,000
11,000
10,000
9,000
8,000
Pressure, psig
S
u
b
s
e
a

D
e
p
t
h

(
F
e
e
t
)
Figure 3 ExamplesofoverpressuredreservoirsintheNorthSea
3
3. FLUID PRESSURES IN HYDROCARBON SYSTEMS
Pressuregradientsinhydrocarbonsystemsaredifferentfromthoseofwatersystems
andaredeterminedbytheoilandgasphasein-situspecifcgravities,
o
and
g
of
eachfuid.
Thepressuregradientsareafunctionofgasandoilcompositionbuttypicallyare:

dP
dD
= (0.45psi / ft)
water


(5)

dP
dD
= (0.35psi / ft)
oil


(6)

dP
dD
= (0.08psi / ft)
gas

(7)

Forareservoircontainingbothoilandafreegascapapressuredistributionresults,
asintheFigure4Ascanbeseen,thecompositionoftherespectivefuidsgivesrise
todifferentpressuregradientsindicatedabove.Thesegradientswillbedeterminedby
thedensityofthefuidswhichresultfromthespecifccompositionofthefuids.

Depth (Ft.)
Formation Pressure (PSI)
Gas-Oil Contact
0.17 psi/ft

f
= 0.39 gm/cc
0.29 psi/ft

f
= 0.67 gm/cc
0.47 psi/ft

f
= 1.09 gm/cc
Oil-Water Contact
1
4000
8800
8700
8600
8500
4050 4100 4150
2
3
4
5
6
7
8
9
10
11
12
13
D
e
p
t
h

(
F
t
.
)
Figure 4 Pressuredistributionforanoilreservoirwithagascapandanoil-watercontact.
Thenatureofthepressureregimeandthepositionandrecognitionoffuidcontacts
areveryimportanttothereservoirengineerinevaluatingreserves,anddetermining
depletionpolicy.
Thedatausedforthesefuidcontactscomesfrom:
(i) Pressuresurveys
(ii) Equilibriumpressuresfromwelltests
(iii) Flowoffuidfromparticularminimumandmaximumdepth
(iv) Fluiddensitiesfromreservoirsamples
(v) Saturationdatafromwirelinelogs
(vi) Capillarypressuredatafromcores
(vii) Fluidsaturationfromcores
EXERCISE
If the pressure in a reservoir at the OWC is psi, calculate the pressure at the top
if there is a 00ft continuous oil column. If a normal pressure gradient exists outwith
the reservoir, calculate the pressure differential at the top of the reservoir. Redo the
calculations for a similar field, but this time containing gas.
Reservoir Pressures and Temperatures
Institute of Petroleum Engineering, Heriot-Watt University

4. PRESSURE GRADIENTS AROUND THE WATER-OIL CON-


TACT
WaterisalwayspresentinreservoirrocksandthepressureinthewaterphasePw
andthepressureinthehyrocarbonphaseP
o
aredifferent.IfPisthepressureatthe
oil/watercontactwherethewatersaturationis100%,thenthepressureabovethis
contactforthehydrocarbonandwaterare:
P
o
=P-
o
gh (8)
P
w
=P-
w
gh (9)
ThedifferencebetweenthesetwopressuresisthecapillarypressureP
c
:seeChapter
8.Inahomogenouswater-wetreservoirwithanoil-watercontactthevariationof
saturationandphasepressurefromthewaterzonethroughthecapillarytransition
zoneintotheoilis showninFigure5). In thetransitionzonethe phasepressure
differenceisgivenbythecapillarypressurewhichisafunctionofthewettingphase
saturation.(Chapter8).

Oil Phase Pressure


Oil Zone
Capilliary
Transition
Zone
Water Zone
Water Saturation, S
w
Pressure, P
Vertical
Depth
D
WOC
Oil Gradient
Water Gradient
Water Phase Pressure
p
o
= p
FWL
-
o
gh
p
w
= p
FWL
-
w
gh
(p
c
= o)
S
wc
S
w
p
c
FWL
p
FWL
0 1
p
c
(S
w
)
g
h =
Figure 5 PressureGradientsaroundtheWater-OilContact
P
c
=P
o
-P
w
(10)
athydrostaticequilibrium
P
c(Sw)
=gh
=
w
-
o

h=heightabovefreewaterlevel
10
Thefreewaterlevel,FWL,isnotcoincidentwiththeoil-watercontactOWC.The
watercontactcorrespondstothedepthatwhichtheoilsaturationstartstoincrease
fromwaterzone.Thefreewaterlevelisthedepthatwhichthecapillarypressure
iszero.
Thedifferenceindepthbetweentheoil-watercontactandthefreewaterleveldepends
onthecapillarypressurewhichinturnisafunctionofpermeability,grainsizeetc.
Providingthephaseiscontinuousthepressuresintherespectivephasesare:
P
o
=P
FWL
-
o
gh (11)
P
w
=P
FWL
-
w
gh (12)
Onthedepth-pressurediagramtheintersectionofthecontinuousphasepressureline
occursatthefreewaterlevel.
5. TECHNIQUES FOR PRESSURE MEASUREMENT
Earliertestsforverticalpressurelogginghavebeenreplacedbyopen-holetesting
devicesthatmeasuretheverticalpressuredistributioninthewell,andrecoverfor-
mationsamples.
Onesuchdevicewhichwasintroducedinthemidseventieswhichhasestablished
itselfinreservoirevaluationistherepeatformationtesterRFT(Schlumbergertrade
name).Itwasinitiallydevelopedasadevicetotakesamples.Overtheyearshowever
itsmainapplicationistoprovidepressure-depthproflesoverreservoirintervals.The
deviceplacesaprobethroughthewellmudcakeandallowssmallvolumesoffuid
tobetakenandpressuremeasurementstobemade(Figure6).Itcanonlybeoperated
thereforeinanopenholeenvironment.Theunitcanbesetatdifferentlocationsin
thewellandthepressuregradienttherebyobtained.Thisdevicehasbeensuperseded
bydifferenttoolsprovidedbyanumberofwirelineserviceproviders.Theprinciple
isthesameofmeasuringwithaprobeinopenholethepressuredepthprofle.
Reservoir Pressures and Temperatures
Institute of Petroleum Engineering, Heriot-Watt University
11

Pressure Guage
Seal Valve
to Upper Chamber
Seal Valve
to Upper Chamber
Piston
Filter
Flow Line
Formation
Probe Closed
Probe Open and
Sampling
Packer
Chamber 1
Chamber 2
Flow Line
Equalising Valve
(To Mud Column)
Packer
Mud Cake
Figure 6 OriginalSchematicoftheRFTTool
Theseopenholepressuremeasurementshaveprovedvaluableatboththeappraisal
stageandcanbeusedtoestablishfuidcontacts.Ithasalsoprovedparticularlyvalu-
ableduringthedevelopmentstageinaccessingsomeofthedynamiccharacteristics
ofthereservoir.Thepressurechangesindifferentreservoirlayersresultingfrom
productionrevealtheamountofinterlayercommunicationandthesepressuremeas-
urementscanbeapowerfultoolinunderstandingthecharacteristicsofthereservoir
formation.
Bycomparingcurrentpressureinformationwiththoseobtainedpriortoproduction,
importantreservoirdescriptioncanbeobtainedwhichwillaidreservoirdepletion,
completiondecisionsandreservoirsimulation.
In1980Amoco
3
publishedapaperwithrespecttotheMontroseFieldinTheNorth
Seawhichillustratestheapplicationofpressure-depthsurveys.Figure7showsthe
pressuredepthsurveyin1978ofawellafterproductionsincemid1976.Onlythe
top45ftofthe75ftoilcolumnhadbeenperforated.Theinitialpressuregradientin-
dicatestheoilandwatergradientsattheconditionofhydrostaticequilibrium.The
secondsurveyshowsasurveyafteraperiodofhighproductionrate,andrevealsthe
reservoirbehaviourunderdynamicconditions.Thevariouschangesinslopeinthe
pressureproflerevealthepartialrestrictedfowincertainlayers.Similarsurveys
ineachnewdevelopmentwells(Figure8)showthesimilarproflesandenablethe
detailedlayeredstructureofthereservoirtobecharacterisedwhichisimportantfor
reservoirsimulationpurposes.
1

4000
8100
8200
8300
8400
8500
8600
8700
8800
2500
2550
2600
2650
Layer 4
Layer 1
Layer 2
Layer 3
Layer 5
2500 3000 3500
14 16 18 20 22 24 26
T
r
u
e

v
e
r
t
i
c
a
l

s
u
b
s
e
a

d
e
p
t
h

-

m
e
t
r
e
s
T
r
u
e

v
e
r
t
i
c
a
l

s
u
b
s
e
a

d
e
p
t
h

-

f
e
e
t
Reservoir pressure - MPa
Reservoir pressure - psig
Perforations
Original
pressure
gradient
Top paleocene
Gr%
0 100
Sw%
100 0
%
0 50
Figure 7 RFTPressureSurveyinDevelopmentWellofMontroseField
3
.

Reservoir pressure - psig


8000
8100
8200
8300
8400
8500
8600
8700
8800
8900
9000
2450
2500
2550
2600
2650
2700
3000 3200 3400
18 20 22 24 26 28
Reservoir pressure - MPa
T
r
u
e

v
e
r
t
i
c
a
l

s
u
b
s
e
a

d
e
p
t
h

-

f
e
e
t
T
r
u
e

v
e
r
t
i
c
a
l

s
u
b
s
e
a

d
e
p
t
h

-

m
e
t
r
e
s
symbol ?Well number Date
22/17-A6 05/04/77
A8 27/01/78
A11 20/12/77
A15 15/08/78
A17 02/11/78
A18 28/03/79
A6
A8
A11
A15
A17
A18
Original
pressure
gradient
Figure 8 RFTPressureSyrveysonanumberofMontroseWells
3
.
Reservoir Pressures and Temperatures
Institute of Petroleum Engineering, Heriot-Watt University
1
6. RESERVOIR TEMPERATURE
Thetemperatureoftheearthincreasesfromthesurfacetocentre.Theheatfowout-
wardsthroughtheEarthscrustgeneratesageothermal gradient,g
c
.Thistemperature
variationconformstobothalocalandregionalgeothermalgradient,resultingfrom
thethermalcharacteristicsofthelithologyandmoremassivephenomenonassociated
withthethicknessoftheearthscrustalongridges,riftsandplateboundaries.
In most petroleum basins the geothermal gradient is of the order of 1.6F/100 ft.
(0.029K/m)Thethermalcharacteristicsofthereservoirrockandoverburdengive
risetolargethermalcapacityandwithalargesurfaceareaintheporousreservoir
onecanassumethatfowprocessesinareservoiroccuratconstant reservoir tem-
perature.Thelocalgeothermalgradientwillbeinfuencedbyassociatedgeological
featureslikevolcanicintrusionsetc.Thelocalgeothermalgradientcanbededuced
fromwellboretemperaturesurveys.Howevertheyhavetobemadeunderstabilised
conditions sincetheycanbeinfuenced bytransient coolingeffectsofcirculating
andinjectedfuids.
Duringdrillingthelocalthermalgradientcanbedisturbedandbyanalysisofthe
variationoftemperaturewithtimeusingabottomholetemperature(BHT)gauge
thelocalundisturbedtemperaturecanbeobtained.
Without temperature surveys the temperature at a vertical depth can be estimated
usingasurfacetemperatureof15
o
C(60
o
F)atadepthD.
T
(D)
=288.2+g
c
D(K)
Solutions to Exercises
EXERCISE 1
Iftheaveragepressuregradientinaregionis0.47psi/ft,calculatetheporepressure
inanormallypressurisedformationat7400ft.ConvertthepressurefrompsitoKPa,
thenexpressthepressureinMPa.WhatisthepressuregradientinKPa/m?
MultiplyKPaby0.145togetpsi.
1USfoot=0.3048m.
SOLUTION
Pressureinformation=0.47*7400=3478psi
ConvertingtoKPa=3478/0.145=23986Kpa
ConvertingtoMPa=23986/1000=23.99MPa
Pressuregradient =0.47psi/ft=(0.47/0.145)KPa/ft=3.2414KPa/ft
=(3.2414/0.3048)KPa/m
=10.63KPa/M
1
EXERCISE 2
IfthepressureinareservoirattheOWCis3625psi,calculatethepressureatthetop
ifthereisa600ftcontinuousoilcolumn.Ifanormalpressuregradientexistsoutwith
thereservoir,calculatethepressuredifferentialatthetopofthereservoir.Redothe
calculationsforasimilarfeld,butthistimecontaininggas.
SOLUTION
Typicalpressuregradientsare(psi/ft):
Water 0.45
Oil 0.35
Gas 0.08
Pressureatseal=3625-(600*0.35)=3415psi
Tocalculatethepressuredifferentialacrossseal,lookatfuidgradientdifferential
fromOWCtoseal600ftabove
Differential=(0.45-0.35)*600=60psi
Ifthereservoirisgasthenthedifferentialbecomes
(0.450.08)*600=222psihigherinthereservoirthansurroundingarea
REFERENCES
1. Dake,L.P.FundamentalsofReservoirEngineering.Elsevier1986
2. Bradley,J.S.AbnormalFormationPressure.TheAmericanAssociationof
PetroleumGeologistsBulletin.Vol59,No6,June1975
3. Bishlawi,MandMoore,RL:MontroseFieldReservoirManagement.SPE
EuropecConference,London,(EUR166)Oct.1980
Reservoir Fluids Composition
CONTENTS
1 INTRODUCTION
2 HYDROCARBONS
2.1 ChemistryofHydrocarbons
2.2 AlkanesorParaffnicHydrocarbons
2.3 Isomerism
2.4 UnsaturatedHydrocarbons
2.5 NaptheneSeries
2.6 Aromatics
2.7 Asphalts
3 NON-HYDROCARBONCOMPOUNDS
4 COMPOSITIONALDESCRIPTIONFOR
RESERVOIRENGINEERING
4.1 DefnitionsofCompositioninReservoir
Engineering
5 GENERALANALYSIS
5.1 SurfaceConditionCharacterisation
5.2 RefractiveIndex
5.3 FluorescenceofOil

LEARNING OBJECTIVES
Having worked through this chapter the Student will be able to:
Describebriefytheorigin,natureandappearanceofpetroleumfuids
Be aware that the principal components of petroleum fluids to be
hydrocarbons.

Drawadiagramillustratingtheclassifcationofhydrocarbonsandtoidentify;
paraffns(alkanes),aromaticsandcyclicaliphatics(napthas).
Listthenon-hydrocarboncompoundswhichmightbepresentinsmallqualities
inreservoirfuids.
Defnetheblackoilmodeldescriptionofthecompositionofareservoirfuid.
ExplainbriefywhatPNAanalysisisanditsapplication.
Describe briefy the concept of pseudo components in fuid composition
characterization.
Beawareofgeneralanalysisdescriptorsforpetroleumfuidse.g.
o
API,refractive
indexandfourescence.
Beabletocalculatethe
A
PIgravitygiventhespecifcgravity
Calculategiventheprerequisitedataproved,probableandpossiblereserves.
Describeingeneraltermsreserveestimation.
Reservoir Fluids Composition
Institute of Petroleum Engineering, Heriot-Watt University

1 INTRODUCTION
Petroleumdepositsvarywidelyinchemicalcompositionanddependingonlocation
have entirely different physical and chemical properties. The very complex
characteristicsareevidentfromthemanyproductswhichcanbeproducedfromoil
andgas.
What is petroleum? Petroleum is a mixture of naturally occurring hydrocarbons
whichmayexistinthesolid,liquidorgaseousstates,dependingontheconditions
oftemperatureandpressuretowhichitissubjected.
1
Petroleumdepositsoccurringasagaseousstatearetermednaturalgas,intheliquid
stateaspetroleumoilorcrudeoilandinthesolidstateastars,asphaltsandwaxes.
Foramixturewithsmallmoleculesitwillbeagasatnormaltemperatureandpressure
(NTP). Mixtures containing larger molecules will be a liquid at NTP and larger
moleculesasasolidstate,forexample,tarsandasphalts.
The exact origin of these deposits is not clear but is considered to be from plant,
animalandmarinelifethroughthermalandbacterialbreakdown.
The composition of crude oil consists mainly of organic compounds, principally
hydrocarbonswithsmallpercentagesofinorganicnon-hydrocarboncompounds.such
ascarbondioxide,sulphur,nitrogenandmetalcompounds.Thehydrocarbonsmay
includethelightest(C
1
methane)tonapthenesandpolycyclicswithhighmolecular
weights.
The appearance varies from gases, through very clear liquids, yellow liquids to a
dark,oftenblack,highlyviscousmaterial,thevarietyobviouslybeingafunctionof
composition.Althoughtheprincipalelementsarecarbon(84-87%),andhydrogen
(11-14%),crudeoilcanvaryfromaverylightbrownliquidwithaviscositysimilar
towatertoaveryviscoustarlikematerial.
Waterisalwayspresentintheporespaceofareservoir,sincetheoriginaldepositional
environmentfortherockswaswater.Thiswaterhassubsequentlybeendisplaced
bytheinfuxofhydrocarbonsbutnottotallysincesurfacetensionforcesactingin
therockporespacecausesomeofthewatertoberetained.
Forreservoirengineeringpurposesthedescriptionofthecompositionisanimportant
characterisationparameterforthedeterminationofarangeofphysicalparameters
importantinvariousreservoirvolumetricandfowcalculations.Itisnottheconcern
ofthereservoirengineertodeterminethecompositionwithrespecttounderstanding
thepotentialtoseparatethematerialtoarangeofsaleableproducts.Forthisreason
thereforesimplisticcharacterisationapproachesareused.
Thetwocompositionalcharacterisationapproachesusedarethecompositional model
and the black oil model. Thebasisofthecompositionalmodelisamulticomponent
descriptionintermsofhydrocarbonsandtheblackoilmodelisatwo component
descriptionintermsofproducedoil,stock tank oilandproducedgas,solution gas.
Thecompositionalmodelisthetopiccoveredinthischapterandtheblackoilmodel
iscoveredintheliquidpropertieschapter.

2 HYDROCARBONS
2.1 Chemistry of Hydrocarbons
Thecompositionalmodeluseshydrocarbonsasthedescriptorsincehydrocarbons
representthelargestproportioninpetroleumfuids.Itisimportanttoreviewbriefy
thechemistryofhydrocarbons.
Thehydrocarbonseriesisrepresentedinfgure1below

Alkenes Alkynes Cyclic Aliphatics Alkanes


(Paraffins) (Napthenes)
Hydrocarbons
Aliphatic Aromatics
Figure 1 ClassifcationofHydrocarbon.
Thehydrocarbonsdivideintotwogroupingswithrespecttothearrangementofthe
carbonmoleculesandthebondsbetweenthecarbonmolecules.Thearrangementof
themoleculesareopenchainorcyclicandthebondsbetweenthecarbonaresaturated
(single)bondsorunsaturatedor(multiple)bonds.
2.2 Alkanes or Paraffnic Hydrocarbons
Thelargestseriesisthealkanesorparaffnswhichareopenchainmoleculeswith
saturatedbonds.Carbonhasavalanceoffourandthereforetheformulaforthese
compounds is C
n
H
2n+2
. These saturated hydrocarbons include all the paraffns in
whichthevalenceofthecarbonatomsissatisfedbysinglecovalentbonds.Thistype
ofstructureisverystable.Unsaturatedhydrocarbonsarethosewherethevalence
ofsomeofthecarbonatomsisnotsatisfedwithsinglecovalentbondssotheyare
connectedbytwoormorebondswhichmakethemlessstableandmoreproneto
chemicalchange.
The paraffn series begins with methane (CH
4
), and its basic formula is C
n
H
2n+2
.
Pentanetopentadecaneareliquidsandthechiefconstituentsofuncrackedgasoline.
Itshighermembersarewaxysolids.Inagivenboreholethewaxmayclogthepore
spacenexttotheholeasgasexpandsandcools.
Theparaffnsarethelargestconstituentofcrudeoilandarecharacterisedbytheir
chemicalinertness.Clearlytheywouldnothaveremainedastheyareifthiswerenot
so.
2.3 Isomerism
Frommethanetopropanethereisonlyonewaytoarrangethebranchedchainshowever
abovepropanetherearealternativearrangementsandthesearecalledisomers.
Reservoir Fluids Composition
Institute of Petroleum Engineering, Heriot-Watt University

Structuralformulaedonotrepresenttheactualstructureofthemolecules.Isomers
aresubstancesofthesamecompositionthathavedifferentmolecularstructureand
thereforedifferentproperties,forexample,normalbutaneandisobutane.
normalbutane CH
3
CH
2
CH
2
CH
3
- B.Pt.31.1F
isobutane CH
3
CHCH
3
- B.Pt.10.9F

CH
3
Pentanehasthreestructures(isomers).Clearlythenumberofisomersincreaseasthe
numberofcarbonatomsincreases.Hexanehas5isomersandheptane9.
Table 1 below gives some of the basic physical properties of the more common
hydrocarbons of the paraffn series andTable 2 lists the state of the various pure
components demonstrating that components which might be solid on their own
contributetoliquidstateswhenpartofamixture.Figure2givessomestructural
formulaforthreeparaffncompounds.

Name Chemical Molecular Boiling Point Critical Gas Liquid


Formula Weight (C) at normal Temp C (air = 1) (water = 1)
conditions
sp.gr.

Methane CH
4
16.04 -161.4 -82.4 0.554 0.415 (-614)
Ethane C
2
H
6
30.07 -89.0 32.3 1.038 0.54 (-88)
Propane C
3
H
8
44.09 -42.1 96.8 1.522 0.585 (-44.5)
n-butane C
4
H
10
58.12 0.55 153.1 2.006 0.601 (0)
Isobutane C
4
H
10
58.12 -11.72 134.0 2.006 0.557
n-pentane C
5
H
12
72.15 36.0 197.2 2.491 0.626
Isopentane C
5
H
12
72.15 27.89 187.8 2.491 0.6197
n-hexane C
6
H
14
86.17 60.30 228.0 2.975 0.6536
Density
Table 1 Physicalpropertiesofcommonhydrocarbons.



1 Methane Gas
2 Ethane Gas
3 Propane Gas
4 Butane Gas
5 Pentane Liquid
6 Hexane Liquid
7 Heptane Liquid
8 Octane Liquid
9 Nonane Liquid
10 Decane Liquid
C5-C17 Liquid
C18+ Solid
ALKANES or PARAFFIN HYDROCARBONS
Cn H 2n+2
No of carbon Name State (ntp)
atoms
Table 2 AlkanesorParaffnHydrocarbonsCnH2n+2

PARAFFINS
Methane Iso-butane n-octane
H
C H H
H
H
C H H
C C C
H H H
H H
H H
H
C H H
H
H
C
H
H
C
H
H
C
H
H
C
H
H
C
H
H
C
H
H
C
H
Figure 2 Givessomestandardformulaforsaturatedhydrocarbons
2.4 Unsaturated Hydrocarbons
Thesearehydrocarbonswhichhavedoubleortriplebondsbetweencarbonatoms.
Theyhavethepotentialtoaddmorehydrogenorotherelementsandaretherefore
termedunsaturated.Therearetermedtheolefns,andtherearetwotypes,alkenes,
for example ethylene, CH
2
=CH
2
, which have a carbon-carbon double bond and
alkynes, for example acetylene,CH=CH which have a carbon carbon triple bond.
Bothcompoundtypesbeingunsaturatedaregenerallyveryreactiveandhenceare
notfoundinreservoirfuids.
2.5 Napthene Series
Thenaptheneseries(C
n
H
2n
)sometimescalledcycloparaffnsoralicyclichydrocarbons
areidentifedbyhavingsinglecovalentbondsbutthecarbonchainisclosedandis
saturated.Theyareverystableandareimportantconstituentsofcrudeoil.Their
chemicalpropertiesaresimilartothoseoftheparaffns.Acrudeoilwithahighnapthene
contentisreferredtoasannapthenicbasedcrudeoil.Anexampleiscyclohexane
C
6
H
12
.Figure3givesthestructuralformulafortwonaptheniccompounds.

Reservoir Fluids Composition


Institute of Petroleum Engineering, Heriot-Watt University


H
C H
H
H
H
H
H
H H
H H
H
C
C
C
C
C
NAPHTHENES
Methyl
Cyclopentane Cyclohexane
C
H H
H
H
H
H
H H
H H
C
C
C
C
C
H H
Figure 3 Structuralformulafortwonapheniccompounds.
2.6 Aromatics
Thearomaticseries(C
n
H
2n-6
)isanunsaturatedclosed-ringseries,basedonthebenzene
compoundandthecompoundsarecharacterisedbyastrongaromaticodour.Various
aromaticcompoundsarefoundincrudeoils.Theclosedringstructuregivesthem
agreaterstabilitythanopencompoundswheredoubleortriplebondsoccur.Figure
4givesthestructuralformulafortwoaromaticcompounds.

H
H
H
C
C
C
C
C
C
H
AROMATICS
Benzene
H
H
H
H
H
C
C
C
C
C
C
H
Naphthalene
H
C
C
C
C
H
H
H
Figure 4 Structuralformulafortwoaromticcompounds.
Thearomatic-napthenebasedcrudesareusuallyassociatedwithlimestoneanddolomite
reservoirssuchasthosefoundinIran,theArabianGulfandBorneo.
Somecrudeoilsusedtobedescribed,morefromarefningperspective,according
totherelativeamountofthesenonparaffncompounds.Crudeoilswouldbecalled
paraffnic, napthenic or aromatic. It is not a classifcation of value in reservoir
engineering.

Physical Properties of some Common Petroleum


Reservoir Fluid Constituents
Component Formula Melting Point Normal Boiling Point Density (g/cm3)
(C) (C) at 1 atm and 15C
Paraffins
Methane CH4 -184 -161.5 -
Ethane C2H6 -172 -88.3 -
Propane C3H8 -189.9 -42.2 -
n-Butane C4H10 -135 -0.6 -
Iso-Butane C4H10 -145 -10.2 -
n-Pentane C5H12 -131.5 36.2 0.626
n-Hexane C6H14 -94.3 69.0 0.659
Iso-octane C8H18 -107.4 99.3 0.692
n-Decane C10H22 030 174.0 0.730
Naphthenes
Cyclopentane C5H10 -93.3 49.5 0.745
Methyl cyclo-pentane C6H12 -142.4 71.8 0.754
Cyclohexane C6H12 6.5 81.4 0.779
Aromatics
Benzene C6H6 5.51 80.1 0.885
Toluene C7H8 -95 110.6 0.867
Xylene C8H10 -29 144.4 0.880
Naphthalene C10H8 80.2 217.9 0.971
Table 3 Physicalpropertiesofsomecommonpetroleumreservoirfuidconstituents
2.7 Asphalts
Asphaltisnotaseriesbyitself.Asphaltsarehighlyviscoustosemi-solid,brown-
black hydrocarbons of high molecular weight usually containing a lot of sulphur
andnitrogen,whichareundesirablecomponents,andoxygen.Asphaltsareclosely
relatedtothenaptheneseriesandbecauseoftheirhighnitrogenandoxygencontent
theymaybeconsideredjuvenileoil,notfullydeveloped.
3 NON-HYDROCARBON COMPOUNDS
Althoughsmallinvolume,generallylessthan1%,non-hydrocarboncompoundshave
asignifcantinfuenceonthenatureoftheproducedfuidswithrespecttoprocessing
andthequalityoftheproducts.

Themorecommonnon-hydrocarbonconstituentswhichmayoccurare:
sulphur,oxygen,nitrogencompounds,carbondioxideandwater.
Sulphur and its associated compounds represent 0.04% - 5% by weight. These
corrosivecompoundsincludesulphur,hydrogensulphide(H
2
S),whichisverytoxic,
andmercaptansoflowmolecularweight(theseareproducedduringdistillationand
requirespecialmetalstoavoidcorrosion).Non-corrosivesulphurmaterialsinclude
sulphides. Sulphur compounds have a bad smell and both the corrosive and non-
corrosiveformsareundesirable.OncombustiontheseproductsproduceS0
2
andS0
3
whichareundesirablefromanenvironmentalperspective.
Reservoir Fluids Composition
Institute of Petroleum Engineering, Heriot-Watt University

Oxygencompounds,upto0.5%wt.,arepresentinsomecrudesanddecomposeto
formnapthenicacidsondistillation,whichmaybeverycorrosive.
Nitrogen contentisgenerallylessthan0.1%wt.,butcanbeasmuchas2%.Nitrogen
compoundsarecomplex.Gaseousnitrogenreducesthethermalqualityofnatural
gas andneedstobeblendedwithhighqualitynaturalgas ifpresentatthehigher
levels.
Carbon Dioxideisaverycommonconstituentofreservoirfuids,especiallyingases
andgascondensates.Likeoxygenitisasourceofcorrosion.Itreactswithwaterto
formcarbonicacidandirontoformironcarbonate.Carbondioxidelikemethanehas
asignifcantimpactonthephysicalpropertiesofthereservoirfuids.
Other compounds.Metalsmaybefoundincrudeoilsatlowconcentrationandare
oflittlesignifcance.Metalssuchascopper,iron,nickel,vanadiumandzincmaybe
present.Producednaturalgasmaycontainhelium,hydrogenandmercury.
Inorganic compounds Thenon-oilproducedfuidslikewaterwillclearlycontain
compoundsarisingfromthemineralspresentintherock,theirconcentrationwill
therefore vary according to the reservoir. Their composition however can have a
verysignifcanteffectonthereservoirbehaviourwithrespecttotheircompatibility
withinjectedfuids.Theprecipitationofsalts,scale,isaseriousissueinreservoir
management.
ManyofthesesaltsneedtoberemovedonrefningassomegenerateHC1when
heatedwithwater.
4. COMPOSITIONAL DESCRIPTION FOR RESERVOIR ENGI-
NEERING
4.1 Defnitions of Composition in Reservoir Engineering
Inpetroleumengineering,andspecifcallyinreservoirengineering,themainissue
is one of the physical behaviour and characteristics of the petroleum fuids. The
composition of the fuid clearly has a signifcant impact on the behaviour and
properties.Inpetroleumengineeringthereforethedescriptionofthecompositionis
akeytodeterminethephysicalpropertiesandbehaviour.
Fortheoilrefnerorchemicalmanufacturerthecompositionofthefuidisthekeyto
determinewhatchemicalproductscanbeextractedorprocessedfromthematerial.
Thepetroleumengineerisnotconcernedwiththefactthattheoilmightcontain,albeit
insmallconcentrations,hundredsofdifferentcomponents.Thepetroleumengineer
wantsassimpleadescriptionaspossiblewhichstillenablesthedeterminationofthe
physicalpropertiesandbehaviourunderdifferenttemperatureandpressureconditions.
Twomodelsareusedinthisindustrytodescribethecompositionforphysicalproperty
predictionpurposes,the black-oil model andthe compositional model.
The black-oil model isa2componentdescriptionofthefuidwherethetwocomponents
are,thefuidsproducedatsurface,stocktankoilandsolutiongas.Associatedwith
thismodelareblack-oilparameterslikesolutiongas-oilratioandtheoilformation
volumefactor.Theseparametersarediscussedinthechapteronliquidproperties.
10
Thecompositional model isacompositionaldescriptionbasedontheparaffnseries
C
n
H
2n+2.
Thefuidisdescribedwithindividualcompositionsofnormalparaffnsup
toalimitingCnumber.HistoricallyC
6
,morecommonnowtogouptoC
9
,oreven
higher. Components greater than the limiting C number are lumped together and
defnedasaC+component.
Isomers, normal and iso are usually identifed up to pentane. Non paraffnic
compoundsareassignedtothenexthigherparaffnaccordingtoitsvolatility.The
materialrepresentingallcompoundsabovethelimitingcarbonnumberarecalledthe
C
+
fraction,soC
7+
foralimitingvalueofC
6
andC
10+
foralimitingvalueofC
9
.
The physical properties of paraffns up to the limiting C number are well known
anddocumented.TheC+componentishoweveruniquetothefuidandtherefore
twopropertiesareusedtocharacteriseit,apparent molecular weight and specifc
gravity.
Thebehaviourofsomefuidsarecomplexandtheparaffnbaseddescriptionmay
havediffcultyinpredictingpropertiesundercertainconditions.Considerationmay
berequiredtoalsoidentifynapthenic and aromatic compounds,(PNAanalysis),
whichcouldbecontributingtocomplexbehaviour.Thisisparticularlythecasefor
gascondensatesexistingathighpressuresandhightemperatures.
Figure4illustratesthecompositionalmodelanditsapplicationasreservoirfuidsare
producedtosurface.Althoughtheindividualcomponentscontributetoasingleliquid
reservoirphaseforanoil,whenthefuidsareproducedtosurfacetheyproduceagas
phase,solutiongas,andaliquidphase,stocktankoil.Thedistributioncharacteristics
oftheindividualcomponentsiscomplexandnotjustafunctionoftemperatureand
pressure.Forreservoirfuidsthecompositionisalsoaninfuenceonthedistribution.
Thismakesitadiffculttasktopredictthisdistributionperspectivesincereservoir
fuid compositions are unique. This topic is further dealt with in the chapter on
vapourliquidequilibrium.Improvedmethodsofchemicalanalysismakeitpossible
todescribetheoiluptoaCvalueofC
29
.Althoughsuchdefnitionsprovideavery
accuratedescription,theassociatedcomputereffortinusingsuchacomprehensive
description does lead to the use of pseudo components. Pseudo components are
obtained by grouping the various C number compositions, thereby reducing the
descriptionto4or5"pseudocomponents".Anumberofmethodsexisttogroupthe
variousCvaluesandothercomponents.
Reservoir Fluids Composition
Institute of Petroleum Engineering, Heriot-Watt University
11

Reservoir Fluid Gas at Surface Conditions


Oil at Surface Conditions
C1 C2 C3 C4 C5 C6 C7+
The relative amounts of C1 - C7+ are a
function of :
Temperature, Pressure, Composition (particularly at high temperature)
Figure 5 CompositionalModel
5. GENERAL ANALYSIS
5.1 Surface condition characterisation
Reservoirsaswellashavinguniquecompositionsalsoexistatspecifcpressuresand
temperatures.Itisimportantthereforetoprovideacommonbasisfordescribingthe
quantitiesoffuidsinthereservoirandthroughouttheproductionprocess.
Thebasischosenisthefuidsatsurfaceconditions,thesurfaceconditionsbeing14.7
psiaor101.3kPaand60
o
For298K.Theseconditionsarecalledstandardconditions.
ForgasthereforethisyieldsstandardcubicfeetSCForstandardcubicmetersSCM.
Itisusefultoconsidertheseexpressionnotasvolumesbutasmass,thevolumeof
whichwillvaryaccordingtodensity.Forliquidsweexpresssurfaceconditionsas
stocktankvolumeseitherstocktankbarrelsSTBorstocktankcubicmetersSTM
3
.
Therelativeamountofgastooilisexpressedbythegas-oilratioGORSCF/STB.
Sincetherearesomanytypesofoil,eachwithawiderangeofspecifcgravity,an
arbitrarynon-linearrelationshipwasdevelopedbytheAmericanPetroleumInstitute
(API)toclassifycrudeoilsbyweightonalinear-scaledhydrometer.Theobserved
readingsarealwayscorrectedfortemperatureto60
o
F,byusingapreparedtableof
standardvalues.
1

DegreesAPI=
141.5
-131.5
Sp.Gr.at60F
(1)
Sp.Gr=specifcgravityrelativetowaterar60
o
F.
TheAPIgravityofwateris10.AlightcrudeoilwouldhaveanAPIgravityof40,
whileaheavycrudewouldhaveanAPIgravityoflessthan20.Inthefeld,theAPI
gravityisreadilymeasuredusingacalibratedhydrometer.
Therearenodefnitionsforcategorisingreservoirfuids,butthefollowingtable5
indicates typical GOR,API and gas and oil gravities for the fve main types.The
compositions show that the dry gases contain mostly paraffns, with the fraction
of longer chain components increasing as the GOR andAPI gravity of the fuids
decrease.
Inchapter4wegiveaclassifcationforthevariousreservoirfuidtypesinthecontext
ofphasebehaviour.

Type Dry Gas WetGas Gas Condensate Volatile Oil Black Oil
Appearance Colourless Colourless Colourless Brown liquid Black
at surface Gas Gas + + significant Some Viscous
clear liquid clear/straw Red/Green Liquid
Colour Liquid
Initial GOR No Liquids >15000 3000-15000 2500-3000 100-2500
(scf/stb)
API - 60-70 50-70 40-50 <40
Gas S.G. 0.60-0.65 0.65-0.85 0.65-0.85 0.65-0.85 0.65-0.85
(air=1)
Composition (mol %)
C
1
96.3 88.7 72.7 66.7 52.6
C
2
3.0 6.0 10.0 9.0 5.0
C
3
0.4 3.0 6.0 6.0 3.5
C
4
0.17 1.3 2.5 3.3 1.8
C
5
0.04 0.6 1.8 2.0 0.8
C
6
0.02 0.2 2.0 2.0 0.9
C
7+
0.0 0.2 5.0 11.0 27.9
Table 5 Typicalvaluesfordifferentreservoirfuids
5.2 Refractive index
Therefractiveindexprovidesanotherindicatorofthedensityofproducedoils.The
general refractive index range for oil is 1.39 to 1.49. The heavier the crude, the
highertherefractiveindexandthelowertheAPIgravity.Thiscanbemeasuredwith
arefractometerorbythesamemethodsusedinopticalmineralogywithreference
gravityoils.
Reservoir Fluids Composition
Institute of Petroleum Engineering, Heriot-Watt University
1
5.3 Fluorescence of oil
Thefuorescenceofoilwhichismeasuredbyitscolourunderultravioletlightprovides
another indicator, and is often used by those analysing the cuttings as the well is
drilled.Therocksampleshouldbeplacedasquicklyaspossibleunderultraviolet
lightsincefuorescenceofoilsubsideswithevaporationandtheactivityofliveoil
decreases.Ifwholecoreisbeingexaminedthenthewholecoreshouldbepassed
underUVlighttodeterminethefuorescentcolourandthepatternofoil-in-placein
thecoredinterval.
Whenpossible,picturesshouldbetakenofthecoreshowingthefuorescence.These
areveryusefulwhenaccompanyingreportstotheheadoffcewhichmaybehundreds
ifnotafewthousandmilesaway.
Thedegreeoffuorescenceisindicatedbelowfordifferentcompositionsasrefected
intheAPIgravity.
2 -10 API non-fuorescenttodullbrown
10 -18 API yellowbrowntogold
18 -45 API goldtopaleyellow
45 -aboveAPI blue-whitetowhite
ItshouldbepointedoutthatmostoilsincreaseinAPIgravitywithdepthinagiven
lithologiccolumnwiththereasonbeingthatyoungerjuvenileoils,heavierwitha
lowerAPIgravity,havenotyetbeentransformedfromtheinitialformationconditions
tohigherpetroleummembers.Twowell-knownexceptionstothispatternarefound
intheBurgansandsofKuwaitandtheshallowsandsoftheBibiEibatfeldinthe
USSRwherethehigh-gravitymembersarefoundhigherupinthestratifedcolumn
thanthelow-gravitymembers.
1
EXERCISE 1
Calculate the Specific Gravity (SG) of a
o
API oil. What is its density in lbs/cu.ft?
(. lbs/cu.ft equals an SG of 1.0 and . API)
Now convert an oil with an SG of 0. to Degrees API.

EXERCISE
A reservoir oil is quoted as having a Gas Oil Ratio (GOR) of 0 scf/bbl. Convert
this to Standard Cubic Meters (SCM)gas per Stock Tank Cubic Meters (SM

)
1 Foot = 0.0m
1 barrel = .1 cu ft.
1 barrel = 0.1 M

EXERCISE
A reservoir is said to contain an initial GOR of 11,000scf/bbl. What type of
reservoir is described, and what API oil could be typically expected from such a
field?
EXERCISE
Define the Black Oil Model and the Compositional Model
Reservoir Fluids Composition
Institute of Petroleum Engineering, Heriot-Watt University
1
Solutions to Exercises
EXERCISE 1
Calculate the Specifc Gravity (SG) of a 38
o
API oil. What is its density in lbs/
cu.ft?
(62.32lbs/cu.ftequalsanSGof1.0and43.28API)
NowconvertanoilwithanSGof0.744toDegreesAPI.
SOLUTION
Convertusingtheequation1:
API=(141.5/SG)-131.5
38=(141.5/SG)-131.5
Sg=141.5/(131.5+38)
SG=0.835
Similarly,toconvertSGintoAPI:
API=(141.5/0.744)-131.5
API=58.7
o
EXERCISE 2
AreservoiroilisquotedashavingaGasOilRatio(GOR)of604scf/bbl.Convert
thistoStandardCubicMeters(SCM)gasperStockTankCubicMeters(SM
3
)
1Foot=0.3048m
1barrel=5.615cuft.
1barrel=0.159M
3
SOLUTION
604scf/bbl=604*0.3048
3
STM/bbl=17.09SCM/bbl=107.48SCM/STM
3

EXERCISE 3
AreservoirissaidtocontainaninitialGORof11,000scf/bbl.Whattypeofreservoir
isdescribed,andwhatAPIoilcouldbetypicallyexpectedfromsuchafeld?
SOLUTION
AreservoirwithaGORof11,000scf/bblwouldbetypicallytermedaGasCondensate
Reservoir.TheAPIgravitywouldprobablybeinthelow50s.
1
EXERCISE 4
DefnetheBlackOilModelandtheCompositionalModel
SOLUTION
BlackOilModel.
Two component description of the reservoir fuid consisting of stock tank oil and
solution gas. Compositional changes with varying pressure and temperature are
ignored.Terms such as Gas Oil Ratio and FormationVolume Factor are black
oilmodelterms.
CompositionalModel.
ThecompositionalmodelisbasedontheparaffnseriesC
n
H
2n+2
.Tokeepthenumber
ofcomponentsinthemodelmanageable,longchainmembersaregroupedtogether
andgivenanaverageproperty.ThesecompoundsaretermedcollectivelyastheC+
fraction. Typically this covers the hydrocarbons above Heptane and therefore is
calledtheC
7
+fraction,whichischaracterisedusingthetermsApparentMolecular
WeightandSpecifcGravity.
REFERENCES.
1. Amyx,J.W.,Bass,D.M.,andWhiting,R.L."PetroleumReservoir
Engineering",McGraw-HillBookCompany,NewYork1960

Phase Behaviour of Hydrocarbon Systems


CONTENTS
1 DEFINITIONS
2 PHASEBEHAVIOUROFPURESUBSTANCES
2.1 ThePhaseDiagram
3 TWOCOMPONENTSYSTEMS
3.1 Pressure-TemperatureDiagrams
3.2 PressureVolumeDiagram
4 MULTI-COMPONENTHYDROCARBON
4.1 PressureVolumeDiagram
4.2 PressureTemperatureDiagram
4.3 CriticalPoint
4.4 RetrogradeCondensation

5 MULTI-COMPONENTHYDROCARBON
5.1 OilSystems(BlackOilsandVolatileOils)
5.2 RetrogradeCondensateGas
5.3 WetGas
5.4 DryGas
6 COMPARISONOFTHEPHASEDIAGRAMSOF
RESERVOIRFLUIDS
7 RESERVOIRSWITHAGASCAP
8 CRITICALPOINTDRYING

LEARNING OBJECTIVES
Having worked through this chapter the Student will be able to:
General
Defne;system,components,phases,equilibrium,intensiveandextensive
properties.
PureComponents
Sketchapressure-temperature(PT)diagramforapurecomponentandillustrate
onit;thevapour-pressureline,criticalpoint,triplepoint,sublimation-pressure
line,themeltingpointline,theliquid,gasandsolidphasezones.
Defnethecriticalpressureandcriticaltemperatureforapurecomponent.
DescribebriefywiththeaidofaPTdiagramthebehaviorofapurecomponent
systembelow(left|)andabove(right)ofthecriticalpoint.
Sketchthepressure-volume(PV)diagramforapurecomponentillustratingthe
behaviorabovethebubblepoint,betweenthebubbleanddewpointandbelow
thedewpoint.
SketchaseriesofPVlinesforapurecomponentwithatemperaturebelow,at
andabovethecriticaltemperature.
Sketchthethreedimensionalphasediagramforpurecomponentsystems.

TwoComponents
PlotaPVdiagramfora2componentsystemandidentifykeyparameters.
PlotaPVdiagramfora2componentsystemandidentifykeyparametersand
therelationshiptothevapourpressurelinesforthetwopurecomponents.
Sketchthecriticalpointlociforaseriesofbinarymixturesincludingmethane
andindicatehowamixtureamixtureofmethaneandanothercomponentcan
existas2phasesatpressuresmuchgreaterthanthe2phaselimitforthetwo
contributingcomponents.
DrawaPTdiagramforatwocomponentsystem,toillustratethecricondentherm,
cricondenbarandtheregionofretrogradecondensation.
Defnethetermscricondenthermandcricindenbar.
Explainbriefywhatretrogradecondensationis.

MulticomponentSystems
SketchaPTandPVdiagramstoillustratethebehaviouratconstanttemperature
forafuidinaPVTcell.Identifykeyfeatures.
Draw a PT diagram for a heavy oil, volatile oil, retrograde condensate gas,
wetgasanddrygas.Illustrateandexplainthebehaviourofdepletionfromthe
undersaturatedconditiontotheconditionwithinthephasediagram.
Describebriefywiththeaidofasketch,thereasonsforandtheprocessofgas
cycling,forretrogradegascondensatereservoirs.
PlotaPTdiagramforareservoirwithagascaptoillustratethegasatdewpoint
andoilatbubblepoint.
Miscellaneous
Withtheaidofsketchexplaintheprocessofcriticalpointdrying.
Phase Behaviour of Hydrocarbon Systems
Institute of Petroleum Engineering, Heriot-Watt University

Oilandgasreservoirfuidsaremixturesofalargenumberofcomponentswhichwhen
subjectedtodifferentpressureandtemperaturesenvironmentsmayexistindifferent
forms,whichwecallphases.Phasebehaviourisakeyaspectinunderstandingthe
natureandbehaviourofthesefuidsbothinrelationtotheirstateinthereservoirand
thechangeswhichtheyexperienceduringvariousaspectsoftheproductionprocess.
Inthischapterwewillreviewthequalitativeaspectsofthebehaviourofreservoir
fuidswhensubjectedtochangesinpressureandtemperature.

1 DEFINITIONS
Beforeweconsidertheeffectoftemperatureandpressureonhydrocarbonsystems
wewilldefnesometerms:
System-amountofsubstanceswithingivenboundariesunderspecifcconditions
composedofanumberofcomponents.Everythingwithintheseboundariesare
partofthesystemandthatexistingoutsideoftheboundariesarenotpartofthe
system.Ifanythingmovesacrosstheseboundariesthenthesystemwillhave
changed.
Components - those pure substances which produce the system under all
conditions.
Forexample,inthecontextofreservoirengineering,methane,ethane,carbondioxide
andwaterareexamplesofpurecomponents.
Phases-Thistermdescribesseparate,physicallyhomogenouspartswhichare
separatedbydefniteboundaries.
1
Examplesinthecontextofwaterarethethree
phases,ice,liquidwaterandwatervapour.
Equilibrium-Whenasystemisinequilibriumthennochangestakeplacewith
respecttotimeinthemeasurablephysicalpropertiesoftheseparatephases.
Intensive and extensive properties - physical properties are termed either
intensive or extensive. Intensive properties are independent of the quantity
ofmaterialpresent.Forexampledensity,specifcvolumeandcompressibility
factorareintensivepropertieswhereaspropertiessuchasvolumeandmassare
termedextensive properties;theirvaluesbeingdeterminedbythetotalquantity
ofmatterpresent.
The physical behaviour of hydrocarbons when pressure and temperature changes
canbeexplainedinrelationtothebehaviouroftheindividualmoleculesmakingup
thesystem.Temperature,pressureandintermolecularforcesareimportantaspects
ofphysicalbehaviour.
The temperatureisanindicationofthekineticenergyofthemolecules.Itisaphysical
measureoftheaveragekineticenergyofthemolecules.Thekineticenergyincreases
asheatisadded.Thisincreaseinkineticenergycausesanincreaseinthemotionof
themoleculeswhichalsoresultsinthemoleculesmovingfurtherapart.

Thepressurerefectsthefrequencyofthecollisionofthemoleculesonthewallsof
itscontainer.Asmoremoleculesareforcedclosertogetherthepressureincreases.
Intramolecularforcesaretheattractiveandrepulsiveforcesbetweenmolecules.They
areaffectedbythedistancebetweenthemolecules.Theattractiveforcesincreases
asthedistancebetweenthemoleculesdecreasesuntilhowevertheelectronicfeldof
themoleculesoverlapandthenfurtherdecreaseindistancecausesarepulsiveforce,
whichincreasesasthemoleculesareforcedclosertogether.
The molecules in gases are widely spaced and attractive forces exist between the
molecules whereas for liquids where the molecules are closer together there is a
repellingforcewhichcausestheliquidtoresistfurthercompression.
Thehydrocarbonfuidsofinterestinreservoirsystemsarecomposedofmanycompo-
nents howeverinunderstandingthephasebehaviourofthesesystemsitisconvenient
torefectonthebehaviourofsingleandtwocomponentsystems.
2 PHASE BEHAVIOUR OF PURE SUBSTANCES
2.1 The Phase Diagram
Itisbenefcialtostudythebehaviourofapurehydrocarbonundervaryingpressure
andtemperaturetogainaninsightintothebehaviourofmorecomplexhydrocarbon
systems.
Phasediagramsareusefulwaysofpresentingthebehaviourofsystems.Theyare
generallyplotsofpressureversustemperatureandshowthephasesthatexistunder
thesevaryingconditions.
Figure1givesapressure-temperaturephasediagramforasingle-componentsystem
onapressuretemperaturediagramandthefollowingpointsaretobenoted.
Phase Behaviour of Hydrocarbon Systems
Institute of Petroleum Engineering, Heriot-Watt University

P
r
e
s
s
u
r
e
Temperature
M
e
l
t
i
n
g

P
o
i
n
t

S
u
b
lim
a
tio
n

V
a
p
o
u
r

P
r
e
s
s
u
r
e

Triple Point
Critical Point
C
1
2
3
Vapour
Liquid
Solid
Gas
Figure 1 Pressuretemperaturediagramforasinglecomponentsystem
Defnetheblackoilmodeldescriptionofthecompositionofareservoirfuid.
ExplainbriefywhatPNAanalysisisanditsapplication.
Vapour Pressure Line
The vapour pressure line divides regions where the substance is a liquid, 2, from
regionswhereitisagas,3.Abovethelineindicatesconditionsforwhichasubstance
isaliquid,whereasbelowthelinerepresentconditionsunderwhichitisagas.Con-
ditionsonthelineindicatewherebothliquidandgasphasescoexist.
Critical Point
ThecriticalpointC.isthelimitofthevapourpressurelineanddefnesthecritical
temperature, Tc and critical pressure, Pcofthepuresubstance.Forapuresubstance
thecriticaltemperatureandcriticalpressurerepresentsthelimitingstateforliquidand
gastocoexist.Amoregeneraldefnitionofthecriticalpointwhichisbothapplicable
tomulticomponentaswellassinglecomponentsystemsis;thecriticalpointisthe
pointatwhichalltheintensivepropertiesofthegasandliquidareequal.
Triple Point
Thetriplepointrepresentsthepressureandtemperatureatwhichsolid,liquidand
vapour co-exist under equilibrium conditions. Petroleum engineers seldom deal
withhydrocarbonsinthesolidstate,however,morerecentlysolidstateissuesarea
concernwithrespecttowax,asphaltenesandhydrates.
Sublimitation-Pressure Line
Theextensionofthevapour-pressurelinebelowthetriplepointrepresentsthecon-
ditionswhichdividestheareawheresolidexistsfromtheareawherevapourexists
andisalsocalledthesublimation-pressureline.

Melting Point Line


Themeltinglinedividessolidfromliquid.Forpurehydrocarbonsthemeltingpoint
generallyincreaseswithpressuresotheslopeofthelineispositive.(Waterisex-
ceptionalinthatitsmeltingpointdecreaseswithpressure).
3 USE OF PHASE DIAGRAMS
3.1 Pressure -Temperature Diagrams (PT)
Considerthebehaviourofacellchargedwithapuresubstanceandthevolumevaried
bythefrictionlessdisplacementofapistonasshowninfgure2,below.

P1 Pb P Pd P2
Liquid
Gas
Figure 2 PhaseChangesWithPressureatConstantTemperature
Forexample,followingthepath1-2infgure3onthepressure-temperaturediagram,
ieholdingtemperatureconstantandvaryingpressurebyexpansionofthecylinder.
Phase Behaviour of Hydrocarbon Systems
Institute of Petroleum Engineering, Heriot-Watt University

c P
c
T
c
P
r
e
s
s
u
r
e
Temperature
Solid
Liquid
M
e
l
t
i
n
g

-

P
o
i
n
t

L
i
n
e
V
a
p
o
u
r
-
p
r
e
s
s
u
r
e
lin
e
T
Gas
E
A
B
G
F
1
2
3
4
Figure 3 Pressure-TemperatureDiagramforaSingle-ComponentSystem
Asthepressureisreduced,thepressurefallsrapidlyuntilapressureisreachedlying
onthevapourpressureline.Agasphasewillbegintoformandmoleculesleavethe
liquid.Atfurtherattemptstoreducethepressurethevolumeofgasphaseincreases,
whileliquidphasevolumedecreasesbutthepressureremainsconstant.Oncethe
liquidphasedisappearsfurtherattemptstoreducepressurewillbesuccessfulasthe
gasexpands.
Abovethecriticaltemperature,followingthepath3-4,adecreaseinpressurewill
causeasteadychangeinthephysicalproperties,forexampleadecreaseindensitybut
therewillnotbeanabruptdensitychangeasthevapourpressurelineisnotcrossed.
Nophasechangetakesplace.
Considerthebehaviourofthesystemaroundthecriticalpoint.Ifwegofrompoint
AtopointB,byincreasingthetemperature,wegothoughadistinctivephasechange
onthevapourpressurelinewheretwophases,liquidandgasco-exist.Ifwenowgo
adifferentroutetoB,startingwiththeliquidstateatAincreasethepressureiso-
thermally(constanttemperature)toavaluegreaterthanP
c
atE.Thenkeepingthe
pressureconstantincreasethetemperaturetoavaluegreaterthanT
c
atpointF.Now
decreasethepressuretoitsoriginalvalueatG.Finally,decreasethetemperature
keepingthepressureconstantuntilBisreached.Thesystemisnowinthevapour
stateandthisstatehasbeenachieved withoutanabruptphasechange.Thevapour
statesareonlymeaningfulinthetwophaseregions.Inareasfarremovedfromthe
twophaseregionparticularlywherepressureandtemperatureareabovethecritical
values,defnitionoftheliquidorgaseousstateisimpossibleandthesystemisbest
describedasinthefuidstate.
Thepressure-temperaturediagramforethaneisgiveninFigure4.

400
500
600
700
800
40 60 80 100 120
Liquid
Vapor
c
Temperature - F
P
r
e
s
s
u
r
e

-

P
S
I
A
Figure 4 Pressure-TemperaturediagramofEthane
3.2 Pressure Volume Diagram (PV)
Theprocessjustdescribedin3.1canalsoberepresentedonapressure-volumedia-
gramatconstanttemperature(Figure5).Asthepressureisreducedfrom1,alarge
change in pressure occurs with small change in volume due to the relatively low
compressibility of the liquid. When the vapour pressure is reached gas begins to
form.Thispointiscalledthebubblepoint,iethepointatwhichthefrstfewmol-
eculesleavetheliquidandformsmallbubblesofgas.Asthesystemexpandsmore
liquidisvaporisedatconstantpressure.Thepointatwhichonlyaminutedropof
liquidremainsiscalledthedewpoint.Sharpbreaksinthelinedenotethebubble
pointanddewpoint.
Phase Behaviour of Hydrocarbon Systems
Institute of Petroleum Engineering, Heriot-Watt University

4
PVT CELL PV DIAGRAM
All Liquid
All Gas
First Gas Bubble
Last Drop of Liquid
1
2
Liquid state-rapid change of
pressure with small volume change
Pressure remains constant while
both gas and liquid are present
Dew Point
Gas
Bubble Point
Volume
P
r
e
s
s
u
r
e
TWO PHASE REGION
SINGLE PHASE T > T
c
T < T
c
T
2
> T
c
Figure 5 Pressure-VolumediagramforaSingle-ComponentSystem
Forapuresubstancevapourpressuresatbubblepointanddewpointareequaltothe
vapourpressureofthesubstanceatthattemperature.Abovethecriticalpoint,ie3
-4,thePVbehaviourlineshowsnoabruptchangeandsimplyshowsanexpansion
ofthesubstanceandnophasechange.Thisfuidiscalledasupercriticalfuid.
A series of expansions can be performed at various constant temperatures and a
pressurevolumediagrambuiltupandthelocusofthebubblepointanddewpoint
valuesgivesthebubblepointanddewpointlineswhichmeetatthecriticalpoint.
Conditions under the bubble point and dew point lines represent the conditions
wheretwophasescoexistwhereasthoseabovethesecurvesrepresenttheconditions
whereonlyonephaseexists.AtthecriticaltemperaturetheP,Tcurvegoesthrough
thecriticalpoint.Figure6
10

B
u
b
b
l
e

P
o
i
n
t

C
u
r
v
e

D
e
w

P
o
i
n
t

C
u
r
v
e

4
3
Liquid state rapid
change of temperature
with small volume change
Critical Point
1
2
Volume
P
r
e
s
s
u
r
e
TWO PHASE REGION
SINGLE PHASE
T = T
c
T < T
c
T > T
c
Pressure remains constant while
both gas and liquid are present
Figure 6 SeriesofPVlinesforapurecomponent
Thepressurevolumecurveforpurecomponentethaneisgiveninfgure7
The locus of the bubble points and dew points form a three-dimensional diagram
whenprojectedintoaP-Tdiagramgivethevapourpressureline(Figure8).

400
500
600
700
800
900
0 0.05 0.10 0.15 0.20 0.25
Liquid
Vapor
C
D B A
Specific Volume - Cu. Ft. per lb.
P
r
e
s
s
u
r
e

-

P
S
I
A
Two Phase Region
1
1
0


F 9
0


F
6
0
F
Figure 7 Pressure-VolumeDiagramofEthane
Phase Behaviour of Hydrocarbon Systems
Institute of Petroleum Engineering, Heriot-Watt University
11

V
o
l
u
m
e
T
e
m
p
e
r
a
t
u
r
e
T
e
m
p
e
r
a
t
u
r
e
L
i
q
u
i
d
G
a
s
L
i
q
u
i
d
G
a
s

a
n
d

L
i
q
u
i
d
G
a
s
Critical Point
Critical Point
Vapor Pressure Curve
Dew Point Line
Bubble Point Line
P
r
e
s
s
u
r
e
P
r
e
s
s
u
r
e
Figure 8 ThreeDimensionalPhaseDiagramforaPureComponentSystem
4 TWO COMPONENT SYSTEMS
Reservoirfuidscontainmanycomponentsbutwewillfrstconsiderasystemcon-
tainingtwocomponents,suchasystemiscalledabinary.
4.1 Pressure Volume Diagram
Thebehaviourofamixtureoftwocomponentsisnotassimpleasforapuresub-
stance.Figure9showstheP-Vdiagramofatwo-componentmixtureforaconstant
temperaturesystem.

P
r
e
s
s
u
r
e
Volume
L
i
q
u
i
d
G
a
s
L
iq
u
id
a
n
d
G
a
s
Bubble Point
Dew Point
Figure 9 Pressure-VolumeLineforaTwo-ComponentSystematConstantTemperature
1
Theisothermisverysimilartothepurecomponentbutthepressureincreasesasthe
systempassesfromthedewpointtothebubblepoint.Thisisbecausethecomposi-
tionoftheliquidandvapourchangesasitpassesthroughthetwo-phaseregion.At
thebubble pointthecompositionoftheliquidisessentiallyequaltothecomposi-
tionofthemixturebuttheinfnitesimalamountofgasisricherinthemorevolatile
component.Atthedew pointthecompositionofvapourisessentiallythemixture
compositionwhereastheinfnitesimalamountofliquidisricherinthelessvolatile
component.Breaksinthelinearenotassharpasforpuresubstances.
Thepressure-volumediagramforaspecifcn-pentaneandn-heptanemixtureisgiven
inFigure10.Clearlyadifferentcompositionofthetwocomponentswouldresult
inadifferentshapeofthediagram.

100
200
300
400
500
600
0 0.1 0.2 0.3 0.4 0.5
Critical point
Specific Volume - Cu. Ft. per lb.
P
r
e
s
s
u
r
e

-

P
S
I
A
4
5
4


F
4
5
0


4
2
5

4
0
0

3
5
0

3
0
0

Dew Point Line
B
u
b
b
l
e

P
o
i
n
t

L
i
n
e
Figure 10 Pressure-VolumeDiagramforN-PentaneandN-Heptane(52.4mole%
Heptane)ref.4
4.2 Pressure Temperature Diagram
Comparedtothesinglelinerepresentingthevapourpressurecurveforpuresubstances
thereisabroadregioninwhichthetwophasesco-exist.Thetwo-phaseregionof
thediagramisboundedbythebubble point lineandthedew point line,andthetwo
lines meetatthe critical point. Points within a loop represent two-phase systems
(Figure11).
Considertheconstanttemperatureexpansionofaparticularmixturecomposition.At
1thesubstanceisliquidandaspressureisreducedliquidexpandsuntilthebubble
pointisreached.Thepressureatwhichthefrstbubblesofgasappearistermedthe
bubblepointpressure.Aspressureisdecreasedliquidandgasco-existuntilaminute
amountofliquidremainsatthedewpointpressure.Furtherreductionofpressure
causesexpansionofthegas.
Phase Behaviour of Hydrocarbon Systems
Institute of Petroleum Engineering, Heriot-Watt University
1
Bycarryingoutaseriesofconstanttemperatureexpansionsthephaseenvelopeis
defnedandwithintheenvelopecontoursofliquidtogasratiosobtained.Theseare
calledqualitylinesanddescribethepressureandtemperatureconditionsforequal
volumesofliquid.Thequalitylinesconvergeatthecriticalpoint.
4.3 Critical Point
In the same way as pure components, when more than one component is present
liquidandgasescannotcoexist,atpressuresandtemperatureshigherthanthecriti-
calpoint.Thecriticalpointforamorethanonecomponentmixtureisdefnedasa
pointatwhichthebubblepointlineanddewpointlinejoin,ie.itisalsothepoint
atwhichalltheintensivepropertiesoftheliquidareidentical.Thisaspectisavery
severetestforphysicalpropertypredictionmethods.
IfthevapourpressurelinesforthepurecomponentsaredrawnontheP-Tdiagram
then the two-phase region for the mixture lies between the vapour pressure lines.
Inthefgure11thecriticaltemperatureofthemixtureT
cAB
liesbetweenT
cA
andT
cB

whereasthecriticalpressureP
cAB
liesaboveP
cA
andP
cB
.Itisimportanttonotethat
theP
cAB
andT
cAB
ofthemixturedoesnotnecessarilyliebetweentheP
c
&T
c
ofthe
twopurecomponents.

C
A
C
B
P
CAB
T
CA
P
r
e
s
s
u
r
e
Temperature
Liquid
Gas
1
2
T
CAB
T
CB
P
CA
P
CB
B
u
b
b
le
- P
o
in
t L
in
e

D
ew
P
oint
% Liq.
100
75
50
25
0
Critical Point
Figure 11 Pressure-TemperatureDiagramforaTwoComponentSystem
Aspecifcmixturecompositionwillgiveaspecifcphaseenvelopelyingbetweenthe
vapourpressurelines.Amixturewithdifferentproportionsofthesamecomponents
willgiveadifferentphasediagram.Thelocusofthecriticalpointofdifferentmix-
turecompositionsisshowninFigure12fortheethaneandn-heptanesystem,andin
Figure13foraseriesofbinaryhydrocarbonmixtures.Figure13demonstratesthat
forbinarymixturee.g.Methaneandn-decanetwophasescancoexistatconditions
ofpressureconsiderablygreaterthanthetwophaselimit,criticalconditionsforthe
separatepurecomponents.Methaneisasignifcantcomponentofreservoirfuids.
1

0 100 200 300 400


1400
1200
1000
800
600
400
200
0
500 600
Temperature F
C
2
C
1
A
1
A
2
A
3
B
1
B
2
B
3
B
A
C
C
3
C
7
D
e
w

P
o
i
n
t

l
i
n
e

N
-
H
e
p
t
a
n
e

E
t
h
a
n
e

B
u
b
b
le

P
o
in
t

L
in
e

Composition
No Wt % Ethane
C 100.00
C
1
90.22
C
2
50.25
C
3
9.78
C
7
N-Heptane
P
r
e
s
s
u
r
e
,

l
b
s
.
/
S
q
.

I
n
.

A
B
S
Figure 12 Pressure-TemperatureDiagramfortheEthane-HeptaneSystem
2
Phase Behaviour of Hydrocarbon Systems
Institute of Petroleum Engineering, Heriot-Watt University
1

0
1000
2000
3000
4000
5000
6000
0 -100 0 100 200 300 400 500 600 700
Temperature F
P
r
e
s
s
u
r
e

L
b
s
.

(
p
s
i
a
)
M
e
t
h
a
n
e

E
th
a
n
e

P
ro
p
a
n
e

N
-B
uta
n
e

N- P
e
n
ta
n
e

N-Hexan
e

N-Heptan
e

N-Decane
Two Phases
Single Phase
Figure 13 CriticalPointLociforaSeriesofBinaryHydrocarbonMixtures
2
4.4 Retrograde Condensation
Withinthetwophaseregionourtwocomponentsystemtherecanbetemperaturesand
pressureshigherthanthecriticaltemperaturewheretwophasesexistandsimilarly
pressures.Theselimitingtemperaturesandpressuresarethecricondenthermand
cricondenbar .Thecricondenthermcanbedefnedasthetemperatureabovewhich
liquidcannotbeformedregardlessofpressure,orexpresseddifferently,asthemaxi-
mumtemperatureatwhichtwophasescanexistinequilibrium.Thecricondenbar
canbedefnedasthepressureabovewhichnogascanbeformedregardlessoftem-
peratureorasthemaximumpressureatwhichtwophasescanexistinequilibrium.
(Figure14).
Theselimitsareofparticularsignifcanceinrelationtotheshapeofthediagramin
fgure14.
ConsiderasingleisothermonFigure14.Forapuresubstanceadecreaseinpressure
causesachangeofphasefromliquidtogas.Foratwo-componentsystembelowT
c

adecreaseinpressurecausesachangefromliquidtogas.

Wenowconsidertheconstanttemperaturedecreaseinpressure,1-2-3,infgure14at
atemperaturebetweenthecriticaltemperatureandthecricondentherm.Aspressure
isdecreasedfrom1thedewpointisreachedandliquidforms,i.e.,at2thesystemis
suchthat5%liquidand95%vapourexists,i.e.adecreaseinpressurehascauseda
changefromgastoliquid,oppositetothebehaviouronewouldexpect.Thephenom-
enaistermed Retrograde Condensation.From2-3,theamountofliquiddecreases
1
andvaporisationoccursandthedewpointisagainreachedwherethesystemisgas.
Retrogradecondensationoccursattemperaturesbetweenthecriticaltemperatureand
cricondentherm.Theretrograderegionisshownshadedinthefgure.


B
u
b
b
l
e

P
o
i
n
t

L
i
n
e

Dew Point Line
% Liq.
100
75
50
25
5
10
0
P
r
e
s
s
u
r
e
Temperature
Liquid
Gas
1
2
3
Cricondenbar
C
r
i
c
o
n
d
e
n
t
h
e
r
m
Region of retrograde condensation
Figure 14 PhaseDiagramShowingConditionsforRetrogradeConsiderations

5. MULTI-COMPONENT HYDROCARBON
Usingtwocomponentsystemswehaveexaminedvariousaspectsofphasebehaviour.
Reservoirfuidscontainhundredsofcomponentsandthereforearemulticomponent
systems.Thephasebehaviourofmulticomponenthydrocarbonsystemsintheliq-
uid-vapour region however is very similar to that of binary systems however the
mathematicaland experimentalanalysis ofthe phasebehaviour ismorecomplex.
Figure15givesaschematicPT&PVdiagramforareservoirfuidsystem.Systems
whichincludecrudeoilsalsocontainappreciableamountsofrelativelynon-volatile
constituentssuchthatdewpointsarepracticallyunattainable.
Phase Behaviour of Hydrocarbon Systems
Institute of Petroleum Engineering, Heriot-Watt University
1

PVT CELL PHASE DIAGRAM


All Liquid
Gas / 40% Liquid
All Gas
First Gas Bubble
Last Drop of Liquid
"a"
Critical Point
Dew Point
Bubble Point
Bubble Point
Temperature
P
r
e
s
s
u
r
e
P
r
e
s
s
u
r
e
Volume
L
iq
u
id

B
u
b
b
l
e

P
o
i
n
t

L
i
n
e

D
e
w
P
o
in
t L
in
e

8
0
%







L
iq
u
i
d

6
0
%

4
0
%

2
0
%

Dew Point
Figure 15 PhaseDiagramsforMulticomponentSystems
Wewillconsiderthebehaviourofseveralexamplesoftypicalcrudeoilsandnatural
gases:
Low-shrinkageoil(heavyoil-blackoil)
High-shrinkageoil(volatileoil)
Retrogradecondensategas
Wetgas
DryGas
Figure16isausefuldiagramtoillustratethebehaviouroftherespectivefuidtypes
above.Howeveritshouldbeemphasisedthatforeachfuidtypetherewillbedifferent
scales.Theverticallineshelptodistinguishthedifferentreservoirfuidtypes.
Isothermalbehaviourbelowthecriticalpointdesignatesthebehaviourofoilsystems
andthefuidisliquidinthereservoir,whereasbehaviourtotherightofthecritical
pointillustratesthebehaviourofsystemswhicharegasinthereservoir.
1

X
5
P
r
e
s
s
u
r
e
Temperature
% Liquid
Gas
(Gas)
Black
Oil
Volatile
Oil
Gas
Condensate Gas

T
M
2
75
100
50
25
20
15
10
5
0 Single Phase Region
Single Phase Region (Liquid) Single Phase Region
Two Phase Region
CP
Where:
P
b
= Bubble point pressure
at indicated temperature
P
m
= Maximum pressure at which
two phases can coexist
T
m
= Maximum temperature at
which two phases can coexist
C = Critical conditions
X
5
= Cricondentherm
B
u
b
b
le
P
o
in
t L
in
e

D
ew
P
o
in
t L
in
e

P
m
P
b
Figure 16 Phasediagramforreservoirfuids
5.1 Oil Systems ( Black Oils and Volatile Oils)
Figures17&18 illustratethePTphasediagramsforblackandvolatileoils.
Thetwo-phaseregioncoversawiderangeofpressureandtemperature.T
c
ishigher
thanthereservoirtemperature.Infgure17theline1-2-3representstheconstant
reservoirtemperaturepressurereductionthatoccursinthereservoirascrudeoilis
producedforablack oil.Theseoilsareacommonoiltype.Thedottedlineshows
theconditionsencounteredasthefuidleavesthereservoirandfowsthroughthe
tubingtotheseparator.
If the initial reservoir pressure and temperature are at 2, the oil is at its reservoir
bubble pointandissaidtobesaturated,thatis,theoilcontainsasmuchdissolved
gasasitcanandafurtherreductioninpressurewillcauseformationofgas.Ifthe
initialreservoirpressureandtemperatureareat1,theoilissaidtobeundersaturated,
i.e.ThepressureinthereservoircanbereducedtoP
b
beforegasisreleasedintothe
formation.Foranoilsystemthesaturation pressure is the bubble point pressure.
Phase Behaviour of Hydrocarbon Systems
Institute of Petroleum Engineering, Heriot-Watt University
1

Sep.
P
r
e
s
s
u
r
e
Temperature
Liquid
Gas
1 Undersaturated
2 Saturated
3
100
75
50
25
0
Critical Point
D
e
w

P
o
i
n
t

l
i
n
e

B
u
b
b
l
e

P
o
i
n
t

L
i
n
e

Mole % Liq.
P
b
Figure 17 PhaseDiagramforaBlackOil
Asthepressureisdroppedfromtheinitialconditionasaresultofproductionoffu-
ids,thefuidsremaininsinglephaseinthereservoiruntilthebubblepointpressure
correspondingtothereservoirtemperatureisreached.Atthispointthefrstbubbles
ofgasarereleasedandtheircompositionwillbedifferentfromtheoilbeingmore
concentratedinthelighter(morevolatile)components.Whenthefuidsarebrought
tothesurfacetheycomeintotheseparatorandasshownonthediagram,thesepara-
torconditionsliewellwithinthetwophaseregionandthereforethefuidpresents
itselfasbothliquidandgas.Thepressureandtemperatureconditionsexistinginthe
separatorindicatethataround85%liquidisproduced,thatisahighpercentageandas
aresultthevolumeofliquidatthesurfacehasnotreducedagreatamountcompared
toitsvolumeatreservoirconditions.Hencethetermlow-shrinkageoil.
Asthepressureisfurtherreducedasoilisremovedfromthereservoir,point3will
bereachedand75%liquidand25%gaswillbeexistinginthereservoir.Strictly
speakingoncethereservoirpressurehasdroppedtothebubblepoint,beyondthatthe
phasediagramdoesnottrulyrepresentthebehaviourofthereservoirfuid.Aswewill
seeinthechapterondrivemechanisms,belowthebubblepointgasproducedfows
morereadilythantheassociatedoilandthereforethecompositionofthereservoir
fuiddoesnotremainconstant.Thesystemiscontinuallychanginginthereservoir
andthereforetherelatedphasediagramchanges.
Thesummarycharacteristicsforablackoilsometimestermedaheavyoilorlow
shrinkageoilareasfollows.
Broad-phaseenvelope
Highpercentageofliquid
Highproportionofheavierhydrocarbons
GOR<500SCF/STB
0
Oilgravity30APIorheavier
Liquid-blackordeepcolour
Volatile oilcontainsamuchhigherproportionoflighterandintermediatehydocar-
bonsthanheavierblackoilandthereforetheyliberaterelativelylargevolumesofgas
leavingsmalleramountsofliquidcomparedtoblackoils.Forthisreasontheyused
tobecalledhighshrinkageoils.Thediagraminfgure18showssimilarbehaviourto
theblackoilexceptthatthelinesofconstantliquidtogasaremorecloselyspaced.
Points1and2havethesamemeaningasfortheblackoil.Asthepressureisreduced
below2alargeamountofgasisproducedsuchthatat3thereservoircontains40%
liquidand60%gas.
Atseparatorconditions65%ofthefuidisliquid,i.e.lessthanpreviousmixture.
The summary characteristics for a volatile sometimes termed a heavy oil or high
shrinkageoilwhencomparedtoblackoilsareasfollows.
Notsobroadphaseenvelopeasblackoil
Fewerheavierhydrocarbons
Deepcoloured
API<50
GOR<8000SCF/STB

P
r
e
s
s
u
r
e
Temperature
Liquid
Gas
1
2
3
100
75
50
25
0
Critical Point
Mole % Liq.
Sep.
B
u
b
b
l
e

p
o
i
n
t

l
i
n
e
D
e
w

p
o
i
n
t

l
i
n
e
40
Figure 18 PhaseDiagramforaVolatileOil
Clearly,forthesefuids,itisthecompositionofthefuidthatdeterminesthenature
ofthephasebehaviourandtherelativepositionofthesaturationlines,(bubblepoint
anddewpointlines),thelinesofconstantproportionofgas/liquidandthecritical
point.
Phase Behaviour of Hydrocarbon Systems
Institute of Petroleum Engineering, Heriot-Watt University
1
Forbothofthesefuidstypesonecanpreventthereservoirfuidgoingtwophase
bymaintainingthereservoirpressureaboveitssaturationpressurebyinjectingfu-
idsintothereservoir.Themostcommonpractiseistheuseofwaterasapressure
maintenancefuid.

5.2 Retrograde Condensate Gas


Ifthereservoirtemperatureliesbetweenthecriticalpointandthecricondentherma
retrograde gas condensatefeldexistsandFigure19givesthePTdiagramforsuch
afuid.Abovethephaseenvelopeasinglephasefuidexists.Asthepressurede-
clinesto2adewpointoccursandliquidbeginstoforminthereservoir.Theliquid
isricherinheaviercomponentsthantheassociatedgas.Asthepressurereducesto
3theamountofliquidincreases.Furtherpressurereductioncausesthereductionof
liquidinthereservoirbyre-vaporisation.Itisimportanttorecognisethatthephase
diagrambelowforaretrogradecondensatefuidrepresentsthediagramforaconstant
compositionsystem.
Beforeproductionthefuidinthereservoirexistsasasinglephaseandisgenerally
calledagas.Itisprobablymoreaccuratetocallitadense phase fuid.Ifthereservoir
dropsbelowthesaturationpressurethedewpoint,thenretrogradecondensationoc-
curswithintheformation.Thenatureofthiscondensingfuidisonlyinrecentyears
beingunderstood.Itwaspreviouslyconsideredthatthecondensingfuidwouldbe
immobilesinceitsmaximumproportionwasbelowthevalueforittohavemobil-
ity.Itwasconsideredthereforethatsuchvaluablecondensedfuidswouldbelostto
productionandtheviabilityoftheprojectwouldbethatfromthewetgas.

B
u
b
b
le

P
o
in
t L
in
e

D
e
w
P
o
in
t L
in
e

P
r
e
s
s
u
r
e
Temperature
Liquid
Gas
1
2
3
100
75
50
25
10
5
0
Critical Point
Mole % Liq.
Sep.
Figure 19 PhaseDiagramforaRetrogradeCondensateGas
Oneofthedevelopmentoptionsforsuchafeldthereforeistosetinplaceapressure
maintenance procedure whereby the reservoir pressure does not fall below the
saturationpressure.Watercouldbeusedasforoilsbutgasmightbetrappedbehind
the water as the water advances through the reservoir. Gas injection, called gas

cycling (Figure20),isthepreferredyetveryexpensiveoption.Inthisprocessthe
producedfuidsareseparatedatthesurfaceandtheliquidcondensates,highvalue
productrelativetoheavyoil,aresentforexport,inanoffshoresituationprobablyby
tanker.Thedrygasisthencompressedandreinjectedintothereservoirtomaintain
thepressureabovethedewpoint.Clearlywiththisprocessthepressurewillstill
decline because the volume occupied by the gas volume of the exported liquid is
not being replaced. Full pressure maintenance is obtained by importing dry gas
equivalenttothisexportedvolumefromanearbysource.Eventuallytheinjecteddry
gasdisplacesthewetgasandthenthefeldcanbeblowndownasaconventional
drygasreservoir,ifasuitableexportrouteforthegasistheninplace.Theprocess
describedisverycostlyandcarrieswithitanumberofrisksnotleastthepossibility
ofearlydrygasbreakthrough.

Imported Gas
Gas
Surface Separation
Gas Water Contact
Dry Gas Reinjection
Injection Well
Production Well
Condensate Sales
Figure 20 Gascyclingprocess
Recentresearchhasshownthatthenatureofoilforminginporousmediabythisret-
rogradeprocessmaynotbeasfrstconsidered.Theisolationofcondensingliquidsin
porousrockisdependantontherelativestrengthoftheinterfacialtensionandviscous
forcesworkingintherock.Iftherelativemagnitudeoftheseishighthenthefuid
willbetrappedhoweveriftheyarelowasaresultoflowinterfacialtension,which
isthecasenearerthecriticalpoint,thenthecondensingliquidsmaybemobileand
moveasaresultofviscousandgravityforces.Condensateliquidshavebeenable
to fow at saturations well below the previously considered irreducible saturation
proportion.Establishedrelativepermeabilitythinkingishavingtobereconsideredin
thecontextofgascondensates.Thephenomenajustdescribedmaygiveexplanation
totheobservationsometimesmadeofanoilrimbelowagascondensatefeld.
LookingatthePTphasediagramonemightconsiderthat"blowingthereservoirdown"
Phase Behaviour of Hydrocarbon Systems
Institute of Petroleum Engineering, Heriot-Watt University

quicklymightbeanoptionandasaresultvaporisethecondensedliquidsinthefor-
mation.Thisisnotaseriousoptionsinceoncethereservoirpressurefallsbelowthe
dewpointtheimpactoftheincreasingliquidproportionremaininginthereservoir
causesthephasediagramtomovetotherightrelativetoreservoirconditions,andany
vaporisingwillbeofthelightestcomponentswhicharelikelytobeingoodsupply
andthereforenotofsignifcantvalue.

Thesummarycharacteristicsforaretrogradegascondensatefuidareasfollows.
ContainsmorelighterHCsandfewerheavierHCsthanhigh-shrinkageoil
APIupto60API
GORupto70,000SCF/STB
Stocktankoiliswater-whiteorslightlycoloured

5.3 Wet Gas


The phase diagram for a mixture containing smaller hydrocarbon molecules lies
wellbelowthereservoirtemperature.Figure21.Thereservoirconditionsalways
remainoutsidethetwo-phaseenvelopegoingfrom1to2andthereforethefuidex-
istsasagasthroughoutthereductioninreservoirpressure.Forawetgassystem,
theseparatorconditionsliewithinthetwo-phaseregion,thereforeatsurfaceheavy
componentspresentinthereservoirfuidcondenseunderseparatorconditionsandthis
liquidisnormallycalledcondensate.Theseliquidcondensateshaveahighpropor-
tionoflightendsandsellatapremium.Theproportionofcondensatesdependon
thecompositionalmixofthereservoirfuidasrepresentedbytheiso-volumelines
onthePTdiagram.

P
r
e
s
s
u
r
e
Temperature
Liquid
Gas
1
2
100
75
50
25
5
0
Critical Point
Mole % Liq.
Sep.
Figure 21 PhaseDiagramforaWetGas
Thereferencewetgas,clearlydoesnotrefertothesystembeingwetduetothepres-
enceofwaterbutduetotheproductioncondensateliquids.

Insomelocationswheretherearenaturalpetroleumleakagesatthesurface,when
condensatesareproducedtheyaresometimescalledwhiteoil.
Thesummarycharacteristicsforwetgasareasfollows.

GOR<100,000SCF/STB
Condensateliquid>50API
5.5 Dry Gas
Thephaseenvelopeofthedrygas,whichcontainsasmallerfractionoftheC2-C6
components,issimilartothewetgassystembutwiththedistinctionthattheseparator
alsoliesoutsidetheenvelopeinthegasregion(Figure22).Thetermdryindicates
therefore that the fuid does not contain enough heavier HCs to form a liquid at
surfaceconditions.
Thesummarycharacteristicsforadrygasareasfollows.
GOR>100,000SCF/STB

P
r
e
s
s
u
r
e
Temperature
Liquid
Gas
1
2
75
50
25
Critical Point
Sep.
Figure 22 PhaseDiagramforaDryGas
6 COMPARISON OF THE PHASE DIAGRAMS OF RESERVOIR FLU-
IDS
Figure16gavearathersimplisticrepresentationofthevarioustypesoffuidswith
respect to the relative position of reservoir temperature with respect to the phase
diagram.Inrealityitisthephasediagramwhichchangesaccordingtocomposition
andtherelativepositionofthereservoirtemperatureandseparatorconditions,and
thesedeterminethecharacterofthefuidbehaviour.Figure23givesabetterindica-
tionofthevariousreservoirtypeswithrespecttoaspecifcpressureandtemperature
Phase Behaviour of Hydrocarbon Systems
Institute of Petroleum Engineering, Heriot-Watt University

scales.Astheproportionofheaviercomponentsintherespectivefuidsincreases
thephaseenvelopemovestotheright.

Dry Gas
Wet Gas
Gas
Condensate
Separator
Critical Point
Volatile
Oil
Black
Oil
Temperature (C)
P
r
e
s
s
u
r
e
Figure 23 Relativepositionsofphasesenvelopes
7 RESERVOIRS WITH A GAS CAP
Figure24illustratesasimplifcationofthephasediagramsassociatedwithanoil
reservoirwithagascap.Thephasediagramforthegascapfuid,theoilreservoir
fuidandforafuidrepresentingthecombinationfuidofamixtureofgasandliquid
inthesameproportionsastheyexistinthereservoirarepresented.

P
r
e
s
s
u
r
e
Temperature
C
G

C
L

Reservoir
Liquid
Total Reservoir Fluid
Reservoir Temperature
Reservoir Gas
C
Separator
Initial
Reservoir
Pressure
Pd=Pb
Figure 24 PhaseDiagramforanOilReservoirwithaGasCap
Thediagramillustratesthatatthegas-oilcontactthegasisatitsdewpressure,theoil
isatitsbubblepointpressureandthecombinationfuidliesontheconstantpropor-
tionqualitylinerepresentingtheratioofthegasandoilastheyexistinthereservoir
system.Thegascapmaybedry,wetorcondensatedependingonthecomposition
andphasediagramofthegas.

8 CRITICAL POINT DRYING


Althoughnotpartofthetopicofphasebehaviourinthecontextofreservoirfuidsit
isusefultoillustratetheapplicationinaverypracticalapplicationinthecontextof
theevaluationofrockproperties.Criticalpointdryinghasbeenusedbyanumber
ofsciencestopreparespecimensofdelicatematerialsforsubsequentmicrovisual
analysis where conventional preparation techniques will destroy delicate fabric.
Criticalpointdryingtakesadvantageofthebehaviouroffuidsaroundthecritical
pointwhereonecangofromonephasetype,likeliquidtogaswithoutavisually
observedphasechange.

Inthe1980sitwasobservedinaUKoffshorefeldthattheinterpretedpermeability
forawellsandinthezonewherewaterinjectionwasproposedwasdifferentfrom
wellinjectivitytestswhencomparedtothecoreanalysisvaluewherethevaluewas
manytimesmore.Theextentofthisdifferencewassuchthatpermeabilitiesfrom
thewelltestgavevalueswhichwouldpreventinjectiontotakeplacewhereasthose
fromthecoretestswouldresultinpracticalinjectivities.Clearlythedifferencewas
important.
Phase Behaviour of Hydrocarbon Systems
Institute of Petroleum Engineering, Heriot-Watt University

Thecompanyconcernedembarkedonamoresophisticatedcorerecoveryandanaly-
sisprocesssuspiciousthatperhapsthefabricoftherockwasbeingaffectedbycore
preparationmethods.Theyresortedtocritical point drying.
Thecorerecoveredfromthewaterzoneofthereservoirfromasubsequentnewwell
wasimmersedandtransferredtothetestlaboratorysubmergedinformationwater.
At the laboratory a core plug sample was extracted, cut to size and loaded into a
coreholderstillsubmergedinthewater.Thecorewasthenmountedinafowrig
(fgure25)andanalcoholwhichismisciblewithwaterdisplacedthewaterinthe
core.Carbondioxideatapressureandtemperaturewhereitisintheliquidstatewas
thenintroducedwhichmiscibledisplacedthealcohol.Thetemperatureandpressure
wasthenadjustedtakingthemaroundthecriticalpointratherthanacrossthevapour
pressurelineofthePTphasediagram(fgure26)endingupwithatemperatureand
pressurebelowthevapourpressurelinewiththefuidnowinagaseousstate.After
thisprocessthepermeabilitywasmeasuredtobeofthesameorderasthatinterpreted
fromthewellinjectivitytest.
Thereasonforthisdifferencewassubsequentlydemonstratedtobeaveryfragile
claywhichduringconventionalcorerecoveryandcleaningwasdamagedtoanextent
thatitsporeblockingstructurewasdestroyed.

P
T
Core In Holder
Figure 25 Criticalpointdryingsystem

Temperature
P
r
e
s
s
u
r
e
Vapour
Pressure Line
GAS
LIQUID
Critical Point
Critical Point Drying Route
Figure 26 Criticalpointdrying

REFERENCES
1.Fig1Daniels,FFarrington:OutlinesofPhysicalChemistry,JohnWiley
&Sons,IncNewYork,1948
2.Fig 2 Brown,GG et al. Natural Gasoline and Volatile Hydrocarbons,
NaturalGasolineAssociationofAmerica,Tulsa,Okl.,1948.
Fig10Sage,S.G.,Lacy,W.N.VolumetricandPhaseBehaviourofHydrocarbons,
GulfPublishingCo.Houston1949
CONTENTS
1 IDEALGASES
1.1 Boyle'sLaw
1.2 Charles'Law
1.3 Avogadro'sLaw
1.4 TheEquationofStateForanIdealGas
1.5 TheDensityofanIdealGas
1.6 StandardConditions
1.7 MixturesofIdealGases
1.7.1 Dalton'sLawofPartialPressures
1.7.2 Amagat'sLaw
1.8 ApparentMolecularWeight
1.9 SpecifcGravityofaGas
2 BEHAVIOUROFREALGASES
2.1 CompressibilityFactorForNaturalGases
2.2 LawofCorrespondingStates
2.3 PseudocriticalPropertiesofNaturalGases
2.4 ImpactofNonhydrocarbonComponentson
zValue
2.5 StandardConditionsForRealReservoir
Gases
3 GASFORMATIONVOLUMEFACTOR
4 COEFFICIENTOFISOTHERMAL
COMPRESSIBILITYOFGASES
5 VISCOSITYOFGASES
5.1 Viscosity
5.2 ViscosityofMixtures
6 EQUATIONSOFSTATE
6.1 OtherEquations-of-State
6.2 VandeWaalsEquation
6.3 Benedict-Webb-RubinEquation(BWR)
6.4 Redlich-KwongEquation
6.5 Soave,RedlichKwongEquation
6.6 PengRobinsonEquationofState
6.7 ApplicationtoMixtures
Behaviour of Gases

LEARNING OBJECTIVES
Having worked through this chapter the Student will be able to:
Presenttheidealequationofstate,PV=nRT.
CalculatethemassofanidealgasgivenPV7Tvalues.
Deriveanequationtocalculatethedensityofanidealgas.
Convertamixturecompositionbetweenweightandmolefraction.
Presentanequationandcalculatetheapparentmolecularweightofamixture.
Defneandcalculatethespecifcgravityofagas.
Present the equation of state, EOS, for a real gas and explain what Z is,
PV=ZnRT.
Defnethepseudocriticalpressureandpsuedocriticaltemperatureandbeable
tousethemtodeterminetheZvalueforagasmixture.
Express and calculate reservoir gas volumes in terms of standard cubic
volumes.
Defnethegasformationvolumefactorandderiveanequationforeitusingthe
EOS.
Calculatethevolumeofgasinareservoirintermsofstandardcubicvolumes
givenprerequisitedata.
Calculate the viscosity of a gas of a specifc composition given perquisite
equationsandfgures.
BeawareofthedevelopmentofEOSstopredictreservoirfuidproperties.
Institute of Petroleum Engineering, Heriot-Watt University

INTRODUCTION
Agasisahomogenousfuidthathasnodefnitevolumebutfllscompletelythevessel
inwhichitisplaced.Thesystembehaviourofgasesisvitaltopetroleumengineers
andthelawsgoverningtheirbehaviourshouldbeunderstood.Forsimplegasesthese
lawsarestraightforwardbutthebehaviourofactualhydrocarbongasesparticularly
attheconditionsoccurringinthereservoiraremorecomplicated.
Wewillreviewthelawsthatrelatetothepressure,volumeandtemperaturesofgases
andtheassociatedequations.Theserelationshipswerepreviouslytermedgaslaws;
itisnowmorecommontodescribethemasequationsofstate.
1 IDEAL GASES
Thelawsrelatingtogasesarestraightforwardinthattherelationshipsofpressure,
temperatureandpressurearecoveredbyoneequation.Firstconsideranidealgas.
Anidealgasisonewherethefollowingassumptionshold:
Volumeofthemoleculesi.e.insignifcantwithrespecttothetotalvolumeof
thegas.
Therearenoattractiveorrepulsiveforcesbetweenmoleculesorbetween
moleculesandcontainerwalls.
Thereisnointernalenergylosswhenmoleculescollide.
Outoftheseassumptionscomethefollowingequations.
1.1 Boyles Law
Atconstanttemperaturethepressureofagivenweightofagasisinverselyproportional
tothevolumeofagas.
i.e.

V
1
P
orPV = constant, Tisconstant
(1)
P=pressure,V=volume,T=temperature.
1.2 Charles Law
Atconstantpressure,thevolumeofagivenweightofgasvariesdirectlywiththe
temperature:
i.e.

V Tor
V
T
= constant, Pisconstant
(2)
Thepressureandtemperatureinbothlawsareinabsoluteunits.
Behaviour of Gases

1.3 Avogadros Law


Avogadros Law can be stated as: under the same conditions of temperature and
pressureequalvolumesofallidealgasescontainthesamenumberofmolecules.That
is,onemolecularweightofanyidealgasoccupiesthesamevolumeasthemolecular
weightofanotheridealgasatagiventemperatureandpressure.
Specifcally,theseare:
(i) 2.73x10
26
molecules/lbmoleofidealgas.
(ii) Onemolecularweight(inlbs)ofanyidealgasat60Fand14.7psia
occupiesavolumeof379.4cuft.
Onemoleofamaterialisaquantityofthatmaterialwhosemassintheunitsystem
selectedisnumericallyequaltothemolecularweight.
eg. onelbmoleofmethaneCH
4
=16lb
onekgmoleofmethaneCH
4
=16kg
1.4 The Equation of State for an Ideal Gas
Bycombiningtheabovelawsanequationofstaterelatingpressure,temperatureand
volumeofagasisobtained.

PV
T
constant
(3)
Ristheconstantwhenthequantityofgasisequaltoonemole.
Itistermedthe Universal Gas Constant andhasdifferentvaluesdependingonthe
unitsystemused,sothat;
Rinoilfeldunits=
10 732 .
cuftpsia
lbmoleR

Table1givesthevaluesfordifferentunitsystems.

p V T n R

psia cu ft R lb - mole 10.73


atm cu ft K lb - mole 1.3145
atm cc K gm - mole 82.06
atm litre K gm - mole 0.08206
atm cu ft R lb - mole 0.730
mm Hg litre K gm - mole 62.37
in.Hg cu ft R lb - mole 21.85

Table 1 ValuesofRfordifferentunitsystems
Institute of Petroleum Engineering, Heriot-Watt University

Fornmolestheequationbecomes:
PV=nRT (4)
T=absolutetemperature
o
Kor
o
Rwhere
K=273+
o
Cand
o
R=460+
o
F
Tofndthevolumeoccupiedbyaquantityofgaswhentheconditionsoftemperature
andpressurearechangedfromstate1tostate2wenotethat:

n
PV
RT
isaconstantsothat
P V
T
=
P V
T
1 1
1
2 2
2


EXERCISE 1.
A gas cylinder contains methane at 1000 psia and 70F. If the cylinder has a vol-
ume of cu.ft assuming methane is an ideal gas calculate the mass of methane in
the cylinder.
1.5 The Density of an Ideal Gas
Sincedensityisdefnedastheweightperunitvolume,theidealgaslawcanbeused
tocalculatedensities.

g
= weight / volume =
m
V
wheregisthegasdensity
For1molem=MW MW=Molecularweight

V
RT
P
=
MW.P
RT
g

(5)
EXERCISE .

Calculate the density of the gas in the cylinder in exercise 1.
Behaviour of Gases

1.6 Standard Conditions


Oilandgasatreservoirconditionsclearlyoccurunderawholerangeoftemperatures
andpressures.
It is common practice to relate volumes to conditions at surface, ie 14.7 psia and
60F.
ie

P V
T

P V
T
res res
res
sc sc
sc

(6)
sc-standardconditionsres-reservoirconditions
Thisrelationshipassumesthatreservoirpropertiesbehaveasideal.ThisisNOTthe
caseaswillbediscussedlater.
EXERCISE .

Assuming methane is at the conditions of exercise 1, calculate the volume the gas
would occupy at standard conditions.
1.7 Mixtures of Ideal Gases
Petroleumengineeringisconcernednotwithsinglecomponentgasesbutmixtures
ofanumberofgases.
Laws established over early years governing ideal gas mixtures include Daltons
LawandAmagatsLaw.
1.7.1 Daltons Law of Partial Pressures
Thetotalpressureexertedbyamixtureofgasesisequaltothesumofthepressures
exertedbyitscomponents.Thepartialpressureisthecontributiontopressureof
theindividualcomponent.
ConsideragasmadeupofcomponentsA,B,Cetc
Thetotalpressureofthesystemisthesumofthepartialpressures
ie

P = P + P + P + .....
A B C
(7)
whereA,BandCarecomponents.
therefore
Institute of Petroleum Engineering, Heriot-Watt University
7

P = n
RT
V
n
RT
V
n
RT
V
i.e.P =
RT
V
n

P
P
=
n
n
= y
A B C
j
j j
j
+ +


(8)
wherey
j
=molefractionofj
th
component.
Thepressurecontributionofacomponent,itspartialpressure,isthetotalpressure
timesthemolefraction.
1.7.2 Amagats Law
AmagatsLawstatesthatthevolumeoccupiedbyanidealgasmixtureisequaltothe
sumofthevolumesthatthepurecomponentswouldoccupyatthesametemperature
andpressure.Sometimescalledthelawofadditivevolumes.
i.e.

V = V + V + V
A B C
(9)

V = n
RT
P
+ n
RT
P
+ n
RT
P
V =
RT
P
n
V
V
=
n
n
= y
A B C
j
j j
j

i e . .
(10)
i.e,foranidealgasthevolumefractionisequaltothemolefraction.
Itisconventionaltodescribethecompositionsofhydrocarbonfuidsinmoleterms.
Thisisbecauseoftheabovelaws.Insomecircumstanceshoweverweightcompositions
mightbeusedasthebasisanditisstraightforwardtoconvertbetweenthetwo.
EXERCISE .
A gas is made up of the following components; lb of methane, lb of ethane and
1. lb of propane. Express the composition of the gas in weight and mole fractions.
Behaviour of Gases

1.8 Apparent Molecular Weight


Amixturedoesnothaveamolecularweightalthoughitbehavesasthoughithada
molecularweight.Thisiscalledtheapparent molecular weight.AMW
Ify
j
representsthemolefractionofthej
th
component:

AMW = y MW
j j

( )
AMWforair=28.97,avalueof29.0isusuallysuffcientlyaccurate.
EXERCISE .
What is the apparent molecular weight of the gas in exercise

1.9 Specifc Gravity of a Gas


Thespecifcgravityofagas,
g
istheratioofthedensityofthegasrelativetothatof
dryairatthesameconditions.

g
g
air
=
(11)
Assumingthatthegasesandairareideal.

g
g
air
g
air
g
=
M P
RT
M P
RT
=
M
M
=
M
29

Mg=AMWofmixture,Mair=AMWofair.
EXERCISE .
What is the gas gravity of the gas in exercise ?
2 BEHAVIOUR OF REAL GASES
The equations so far listed apply basically to ideal systems. In reality, however,
particularlyathighpressuresandlowtemperaturesthevolumeofthemoleculesare
nolongernegligibleandattractiveforcesonthemoleculesaresignifcant.
Institute of Petroleum Engineering, Heriot-Watt University

The ideal gas law, therefore, is not too applicable to light hydrocarbons and their
associatedfuidsanditisnecessarytouseamorerefnedequation.
Therearetwogeneralmethodsofcorrectingtheidealgaslawequation:
(1)ByusingacorrectionfactorintheequationPV=nRT
(2)Byusinganotherequation-of-state
2.1 Compressibility Factor for Natural Gases
Thecorrectionfactorzwhichisafunctionofthegascomposition,pressureand
temperatureisusedtomodifytheidealgaslawto:
PV=znRT (12)
wherethefactorzisknownasthecompressibility factorandtheequationisknown
asthecompressibilityequation-of-stateorthecompressibilityequation.
Thecompressibilityfactorisnotaconstantbutvarieswithchangesingascomposition,
temperatureandpressureandmustbedeterminedexperimentally(Figure1).
Tocomparetwostatesthelawnowtakestheform:

P V
z T
=
P V
z T
1 1
1 1
2 2
2 2
(13)
zisanexpressionoftheactualvolumetowhattheidealvolumewouldbe.
i.e.

z
Vactual
Videal

(14)

T
e
m
p
e
r
a
t
u
r
e

=

c
o
n
s
t
a
n
t

0
0
0.5
1.0
PRESSURE, P
C
o
m
p
r
e
s
s
i
b
i
l
i
t
y

f
a
c
t
o
r
,

Z
Figure 1 Typicalplotofthecompressibilityfactorasafunctionofpressureatconstant
temperature.
Behaviour of Gases
10
Althoughallgaseshavesimilarshapeswithrespecttoztheactualvaluesarecomponent
specifc.Howeverthroughthelaw of corresponding states allpuregasesareshown
tohavecommonvalues.
2.2 Law of Corresponding States
Thelawofcorrespondingstatesshowsthatthepropertiesofmanypureliquidsand
gaseshavethesamevalueatthesamereducedtemperature(T
r
)andpressure(P
r
)
where:

T =
T
T
andP =
P
P
r
c
r
c
(15)
Where,T
c
andP
c
arethepurecomponentcriticaltemperatureandpressure.
Thecompressibilityfactorzfollowsthislaw.ItisusuallypresentedvsTrandPr.
AlthoughinmanycasespuregasesfollowtheLawofCorrespondingStates,thegases
associatedwithhydrocarbonreservoirsdonot.TheLawhashoweverbeenusedto
apply to mixtures by defning parameters called pseudo critical temperature and
pseudocritical pressure .
For mixtures a pseudocritical temperature and pressure, T
pc
and P
pc
is used such
that:

T = y T andP = y P
pc j cj pc j cj

(16)
whereyisthemolefractionofcomponentjandT
cj
andP
cj
arethecriticaltemperature
andpressureofcomponentj.
It should be emphasised that these pseudo critical temperature and pseudocritical
pressures are not the same as the real critical temperature and pressure.Bydefnition
thepseudovaluesmustliebetweentheextremecriticalvaluesofthepurecomponents
whereastheactualcriticalvaluesformixturescanbeoutsidetheselimits,aswas
observedinthePhaseBehaviourchapter.
EXERCISE 7.
Calculate the pseudo critical temperature and pseudocritical pressure of the mixture
in exercise .

Formixturesthecompressibilityfactor(z)hasbeengeneratedwithrespecttonatural
gases
1
, where z is plotted as a function of pseudo reduced temperature,T
pr
and
pseudoreducedpressureP
pr
where
Institute of Petroleum Engineering, Heriot-Watt University
11

0
1.0
1.1
0.9
0.8
0.7
0.6
0.5
0.4
0.3
0.25
1.1
1.0
0.9
1.0
1.05
1
.
0
5
1
.
1
1
.
2
1
.
3
1
.
4
1
.
5
1
.
6
1
.
7
1
.
8
1
.
9
2
.0 2
.2
2
.4
2
.6
3
.0
3.0
2.8
1.2
1.3
1.1
1.1
0.95
1.7
1.6
1.5
1.4
1.3
1.2
1.1
1.0
0.9
1 2 3 4 5 6 7 8
7 8 9 10 11 12 13 14 15
Compressibility of
Natural Gases
(Jan. 1, 1941)
Compressibility Factors for Natural Gases as a
Function of Pseudoreduced Pressure and Temperature.
C
o
m
p
r
e
s
s
i
b
i
l
i
t
y

F
a
c
t
o
r
,

z
Pseudo Reduced Temperature
Pseudo Reduced Pressure, P
r
3.0
2.8
2.6
2.4
2.2
2.0
1.9
1.8
1.7
1.6
1.5
1.45
1.35
1.4
1.3
1.25
1.2
1.15
1.1
2.6 2.4
2.2
2.0
1.9
1.7
1.6
1.4
1.3
1.2
1.1
1.05
1.05
1.8
1
.
4
1
.
5
Pseudo Reduced Pressure, P
r
Figure 2 Compressibilityfactorsfornaturalgas
1
(Standing&Katz,TransAIME,1942)
Behaviour of Gases
1

T
T
T
and
P
P
pr
pc pc
P
pr
(17)
Theuseofthischart,fgure2,hasbecomecommonpractisetogeneratezvaluesfor
naturalgases.PoettmannandCarpenter
2
havealsoconvertedthecharttoatable.
Variousequationshavealsobeengeneratedbasedonthetables.
EXERCISE .
For the gas of exercise determine the compressibility factor at a temperature of
10F and a pressure of 00psia.
2.3 Pseudocritical Properties of Natural Gases
Thepseudocriticalpropertiesofgasescanbecomputedfromthebasiccomposition
but can also be estimated from the gas gravity using the correlation presented in
Figure3.

0.5 0.6 0.7 0.8 0.9 1.0 1.1 1.2


Pseudocritical Properties of Natural Gases
P
s
e
u
d
o
c
r
i
t
i
c
a
l

T
e
m
p
e
r
a
t
u
r
e
,


R
P
s
e
u
d
o
c
r
i
t
i
c
a
l

P
r
e
s
s
u
r
e
,

p
s
i
a
Gas Gravity (air = 1)
700
650
600
550
500
450
400
350
300
Condensate W
ell Fluids
Miscellaneous Gases
M
i
s
c
e
l
l
a
n
e
o
u
s

G
a
s
e
s

C
o
n
d
e
n
s
a
t
e
W
e
ll F
lu
id
s

Figure 3 Pseudocriticalpropertiesofnaturalgases
3
Institute of Petroleum Engineering, Heriot-Watt University
1
2.4 Impact of Nonhydrocarbon Components on z value.
Componentslikehydrogensulphide,andcarbondioxidehaveasignifcantimpact
onthevalueofz.Ifthemethodpreviouslyappliedisusedlargeerrorsinzresult.
Wichert andAziz
4
haveproducedanequation whichenablestheimpactof these
twogasestobecalculated.
T'
pc
=T
pc
-r (18)
and



+
( )
p
p T
T y y
pc
pc pc
pc H S H S
2 2
1 r
(19)
T'
pc
andp'
pc
areusedtocalculateT
pr
andp
pr
.Thevalueforrisobtainedfrom
thefgure4fromtheWichertandAzizpaper

0 10 20 30 40 50 60 70 80
0
10
20
30
40
50
60
70
80
PER CENT H
2
S
P
E
R

C
E
N
T

C
0
2
5
10
15
20
25
30
15
20
25
30
E
34.5
Figure 4 Adjustmentfactorsforpseudocritiaclpropertiesfornonhydrocarbon
gases(Wichert&Aziz)
Behaviour of Gases
1
EXERCISE .
Calculatethepseudocriticalpropertiesofthegasinexercise4ifitalsocontained3
lbofhydrogensulphide,10lbofcarbondioxideand2.5lbofnitrogen

1
2
3
Gas
Components
Mol
weight
Mole
fraction
p
c
-psi T
c
R p
pc
psia
Methane 25 0.56 16.04 0.035 0.743 667.00 344 495.8 255.70
Ethane 3 0.07 30.07 0.002 0.048 708.00 550 33.7 26.17
Propane 1.5 0.03 44.09 0.001 0.016 616.00 666 10.0 10.81
Hydrogen 3 0.07 34.08 0.002 0.042 1306 673 54.8 28.25
sulphide
Carbon 10 0.22 44.01 0.005 0.108 1071 548 116.1 59.38
Dioxide
Nitrigen 2.5 0.06 28.02 0.002 0.043 493 227 21.0 9.66
Total 45 1.00 0.0466 1.000 731 390
T
pc
Weight Wgt
fraction
lb moles
4
5
6
FromWichert&AzischartforcompositionsofH
2
SandCO
2
r=19



+
( )

T = T - = 371 R
P = 694.3
pc pc
o
pc
r
r
p
p T
T y y
pc
pc pc
pc H S H S
2 2
1
2.5 Standard Conditions for Real Reservoir Gases
Asindicatedinsection1.6foridealgasesitisconvenienttodescribethequantityof
gastoacommonbasisandthisistermedthestandardconditions,givingrisetothe
standardcubicfootandthestandardcubicmetre.Thepetroleumengineerisprimarily
interestedinvolumecalculationsforgaseousmixtures.Throughouttheindustrygas
volumesaremeasuredatastandardtemperatureof60F(15.6C)andatapressureof
14.7psia(oneatmosphere).Theseconditionsarereferredtoasstandard temperature
and pressure STP.StandardCubicFeet,theunitofvolumemeasuredunderthese
conditionsissometimesabbreviatedSCForscf(SCMisStandardCubicMetres).It
ishelpfultoconsidertheseexpressionsnotasvolumesbutasanalternateexpression
ofthequantityofmaterial.Forexampleamassofgascanbeexpressedassomany
standardcubicfeetormetres.
EXERCISE 10.
Express the quantity of 1 lb mole of a gas as standard cubic feet.
Institute of Petroleum Engineering, Heriot-Watt University
1
EXERCISE 11.
Express the mass of gas in exercise as standard cubic feet.
3 GAS FORMATION VOLUME FACTOR
Thepetroleumindustryexpressesitsreservoirquantitiesatacommonbasisofsurface
conditionswhichforgasesisstandardcubicvolumes.Toconvertreservoirvolumes
tosurfacevolumestheindustryusesformationvolumefactors.Forgaseswehave
B
g
, the gas formation volume factor,whichistheratioofthevolumeoccupiedat
reservoirtemperatureandpressurebyacertainweightofgastothevolumeoccupied
bythesameweightofgasatstandardconditions.TheshapeofB
g
asafunctionof
pressureisshowninfgure5.

B
volumeoccupiedatreservoirtemperatureandpressure
volumeoccupiedatSTP
g

ThegasformationvolumefactorcanbeobtainedfromPVTmeasurementsonagas
sampleoritmaybecalculatedfromtheequations-of-statediscussedpreviously.
One defnition of the gas formation volume factor is: it is the volume in barrels
that one standard cubic foot of gas will occupy as free gas in the reservoir at the
prevailing reservoir pressure and temperature.
Depending on the defnition the units will change and the units will be; rb free
gas/scfgasorrm
3
freegas/scmgas

.008
.006
.004
.002
1000 2000 3000
Bg
rb/scf
PRESSURE (psig)
Figure 5 GasFormationVolumeFactor,Bg
Behaviour of Gases
1
ForexampleB
g
forareservoiratcondition2is;

B
V
V
P T z
P T z
g
2
sc
sc 2 2
2 sc sc

(20)
screferstostandardconditions.zatstandardconditionsistakenas1.0
ThereciprocalofB
g
isoftenusedtocalculatevolumesatsurfacesoastoreducethe
possibilityofmisplacingthedecimalpointassociatedwiththevaluesofB
g
being
lessthan0.01,ie:

volumeatsurface
volumeinformation B
g

1
E
Eissometimesreferredtoastheexpansion factor.
UsuallytheunitsofB
g
arebarrelsofgasatreservoirconditionsperstandardcubic
footofgas,iebbl/SCForcubicmetresperstandardcubicmetre.

B
V
V
g
R
sc

(21)
Randscarereservoirandstandardconditionsrespectively.

V
znRT
P
R

(22)
TandPatreservoirconditions:

V
z nRT
P
sc
sc sc
sc

(23)
z=1forstandardconditions

B z
T
T
P
P
cu.ft
SCF
g
sc
sc
. .
(24)
SinceTsc=520R
m
Psc=14.7psiaformostcases

B 0
zT
P
cu.ft
SCF
g
.0283

B 0
zT
P
cu.ft
SCF
bbl
5.615cuft
B 0
zT
P
resbbl
SCF
g
g

.
.
0283
00504
or

Institute of Petroleum Engineering, Heriot-Watt University
17

B 0
zT
P
cu.ft
SCF
bbl
5.615cuft
B 0
zT
P
resbbl
SCF
g
g

.
.
0283
00504
or
(25)
EXERCISE 1.
Calculate the gas formation factor for a gas with the composition of exercise
existing at the reservoir conditions given in exercise .
EXERCISE 1.
A reservoir exists at a temperature of 10F (as for exercise ) suitable for storing
gas. It has an areal size of miles by miles and is 00ft thick. The average porosity
is 0% and there is no water present. How much gas of the composition of exercise
can be stored at a pressure the same as in exercise i.e. 00 psia ? (1 mile=
0 ft.)

4 COEFFICIENT OF ISOTHERMAL COMpRESSIBILITy OF GAS-


ES
Thecompressibilityfactor,z,mustnotbeconfusedwiththecompressibilitywhichis
defnedasthechangeinvolumeperunitvolumeforaunitchangeinpressure,or

c
V
V
P
or
V
V
P
g
m
m

j
(
,
\
,
(
j
(
,
\
,
(
1 1

(26)

V
m
isthespecifcvolumeorvolumepermole.
cgis notthesameasz,thecompressibilityfactor.
Foranidealgas:
PV=nRTor:

dV
dP
nRT
P
c =
1
V
nRT
P
=
1
P
2
g 2
j
(
\
,

j
(
\
,

j
(
\
,
(27)
Forrealgases:

V =
znRT
P
Behaviour of Gases
1

V
P
nRT
P
dz
dP
P
c
P
nRTz
nRT
P
P
z
P
z
c
P
1
z
z
P
T
2
g 2
g
j
(
,
\
,
(


j
(
,
\
,
(
,

,
]
]
]

z
1
.
(28)
dz/dPcanbeobtainedfromtheslopeofthezvsPcurve.

The Law of Corresponding states can be used to express the above equation in
anotherform

P = P P
z
P
P
P
z
P
z
P
z
P
pc pr
pr
pr
pr

j
(
,
\
,
(
j
(
,
\
,
(

j
(
,
\
,
(
j
(
,
\
,
(
P
P P
P
pr
pc
pc
1
1
Combiningthisequationwitheqn28aboveyields

c
P P
1
zP
z
P
c P
P
1
z
z
P
g
pc pr pc pr
g pc
pr pr

j
(
,
\
,
(

j
(
,
\
,
(
1
1

T
T
pr
pr
(29)
Unitsofc
g
=P
-1
,andc
g
P
c
isdimensionless
c
p
P
pc
iscalledpseudo reduced compressibility,c
pr
Institute of Petroleum Engineering, Heriot-Watt University
1
Sincethepseudoreducedcompressibilityisafunctionofzandpseudoreduced
pressure,thegraphofFigure2canbeusedwithEquation29tocalculatevaluesof
c
pr
.
5 VISCOSITy OF GASES
5.1 Viscosity
Viscosityisameasureoftheresistancetofow.Itisgiveninunitsofcentipoise.
Acentipoiseisagm/100sec.cm.Theviscositytermiscalled dynamic viscosity
whereaskinematicviscosityisthedynamicviscositydividedbythedensity.

kinematic vis ity


density
cos
dynamicviscosity
Kinematicviscosityhasunitsofcm
2
/100secandthetermiscalledcentistoke.
Gasviscosityreducesasthepressureisdecreased.Atlowpressuresanincreasein
temperatureincreasesgasviscositywhereasathighpressuresgasviscositydecreases
asthetemperatureincreases.Figure6givesthevaluesforpurecomponentethane.

1000
900
800
700
600
500
400
300
200
100
90
80
70
50 100 150 200 250 300 350 400
Temperature, deg F
V
i
s
c
o
s
o
t
y
,

m
i
c
r
o
p
o
i
s
e
s
Viscosity of ethane
Pressure, psia
5000
4000
3000
2000
15000
1000 750
600
14.7
Figure 6 Viscosityofethane
Theviscosityofgasesatlowpressurescanbeobtainedfromcorrelationspresented
bydifferentworkers.
Behaviour of Gases
0

50 100 150 200 250 300 350 400


V
i
s
c
o
s
i
t
y
,

c
p
Temperature, ?F
0.020
0.022
0.024
0.018
0.016
0.014
0.012
0.010
0.008
0.006
0.004
H
e
liu
m

N
it
r
o
g
e
n

C
a
r
b
o
n
D
io
x
id
e

M
e
th
a
n
e

E
th
y
le
n
e

E
th
a
n
e

p
ro
p
a
n
e

i-B
u
ta
n
e

n
-B
u
ta
n
e
n
-p
e
n
ta
n
e

n
-H
exan
e
n-H
eptane
n-O
ctane
n-Nonane
n-Decane
H
y
d
r
o
g
e
n
S
u
lf
id
e

A
ir

Figure 7 Viscosityofparaffnhydrocarbongasesatoneatmosphere
Figure 7 and Figure 8 give the viscosities of individual components and paraffn
hydrocarbonsatoneatmosphere.Forsystemsgreaterthan1atmostheviscosities
can be obtained from the literature. Another way is by calculating the reduced
temperatureandreducedpressureandusethechartdevelopedbyCarr
6
whichgives
a ratio of at reservoir conditions. This is given in Figure 9 in terms of pseudo
reducedconditions.

Institute of Petroleum Engineering, Heriot-Watt University
1
400 F
300 F
200 F
100 F
0.5 1.0 1.5 2.0 2.5 3.0 3.5
10 20 30 40 50 60 70 80 90 100
0.004
0.005
0.006
0.007
0.008
0.009
0.010
0.011
0.012
0.013
0.014
0.015
0.016
Molecular Weight
V
i
s
c
o
s
i
t
y
,

a
t

1

a
t
m
,

1
,

c
e
n
t
i
p
o
i
s
e
Gas Gravity (Air = 1)
N
2
Mole per cent N
2
G = 20
G = 06
G = 20
G = 06
1.5
1.0
1.5
1.0
G = 20
G = 06
1.5
1.0
0
0.0015
0.0010
0.0005
0
5 10 15
C
o
r
r
e
c
t
i
o
n

a
d
d
e
d

t
o

V
i
s
c
o
s
i
t
y
,

c
.
p
.
0.0015
0.0010
0.0005
C
o
r
r
e
c
t
i
o
n

a
d
d
e
d

t
o

V
i
s
c
o
s
i
t
y
,

c
.
p
.
0.0015
0.0010
0.0005
C
o
r
r
e
c
t
i
o
n

a
d
d
e
d

t
o

V
i
s
c
o
s
i
t
y
,

c
.
p
.
CO
2
Mole per cent CO
2
0
0
5 10 15
H
2
S
Mole per cent H
2
S
0
0
5 10 15
Figure 8 Viscosityofgasesatatmosphericpressure
6

p
s
e
u
d
o
r
e
d
u
c
e
d
p
r
e
s
s
u
r
e
, p
R

0.8 1.0
1.0
1.5
2.0
2.5
3.0
3.5
4.0
5.0
6.0
1.2 1.4 1.6 1.8 2.0 2.2 2.4 2.6 2.8 3.0 3.2 3.4
= Viscosity at operating temperature
and pressure, centipoises

A
= Viscosity at 14.7 psia (1atm) and
operating temperatures, centipoises

20
15
10
8
6
4
3
2
1
Pseudoreduced Temperature, T
R
V
i
s
c
o
s
i
t
y
,

A
Figure 9 Viscosityratiovspseudoreducedtemperatureandpseudopressure.
Behaviour of Gases

5.2 Viscosity of Mixtures


Anotherformulathatisusedformixturesis:



mix
j j j
j j
y M
y M

(30)
j=1,n
where:

y = molefractionofjthcomponent
M = molecularweightofcomponent
= theviscosityofjthcomponent
n = numberofcomponents
j
j
j

Thepresenceofothergasescanalsomakeasignifcantdifferenceontheviscosity
(Figure7).
EXERCISE 1.

Calculate the viscosity of the gas mixture in exercise at 00F and a pressure of
one atmosphere.
EXERCISE 1.
Use the gas gravity method to calculate the viscosity of the gas in exercise
EXERCISE 1.
Determine the viscosity of the gas in exercise at 10F and 00 psia (ref ex , 7,
&)
Institute of Petroleum Engineering, Heriot-Watt University

6 EQUATIONS OF STATE
6.1 Other Equations-of-State
Asindicatedatthestartofsection2thecompressibilityfactorevolvedoutofthe
need to use an equation derived out of ideal gas behaviour and incorporating it
intoitacorrectionfactortosuitrealgasbehaviour.Oneofthediffcultiesofthe
compressibilityequation:
PV=ZnRT
todescribethebehaviourofgasesisthatthecompressibilityfactorisnotconstantand
thereforemathematicalmanipulationscannotbemadedirectlybutmustbecarriedout
throughgraphicalornumericaltechniques.Ratherthanusethismodifedequation
ofstatemanyhavedevelopedequationsspecifcallytorepresentthebehaviourof
realgases.Itisanironyhoweverthatbecauseofthelonguseoftheequationabove
incorporatingzmanyoftherealgasequationofstateshavebeenworkedtocalculate
zforuseintheaboveequation.
6.2 Van de Waals Equation 1873
ThewellknownvanderWaalsequationwasoneoftheearliestequationstorepresent
thebehaviourofrealgases.ThismostbasicEOS,whichcorrectsforthevolumeofthe
moleculesandattractiveandcollisionforcesusingempiricalconstraintsaandb.
(P+a/V
2
)(V-b)=RT (31)
The two corrective terms to overcome the limiting assumptions of the ideal gas
equationare:
(i)Theinternalpressureorcohesionterm,whichaccountsforthecohesionforces,
isa/V
2
.
(ii)Theco-volumeb,whichrepresentsthevolumeoccupiedbyonemoleatinfnite
pressureandresultsfromtherepulsionforceswhichoccurwhenthemolecules
moveclosetogether.
Theequationcanalsobewrittenas:
V
3
-(+b)V
2
+(a/P)V-ab/P=0
Suchequationsarethereforecalledcubicequationsofstate.
Theequationwrittentosolveforz,thecompressibilityfactor,becomes:
Z
3
-Z
2
(1+B)+ZA-AB=0 (32)
where

A
aP
RT
and B
bP
RT

( )
2

(33)
Behaviour of Gases

Values of a and b are positive constants for a particular fuid and when they are
zerotheidealgasequationisrecovered.OnecancalculatePasafunctionofVfor
variousvaluesofT.Figure10isafgureof3isotherms.Alsodrawnisthecurve
forsaturatedliquidandsaturatedvapour.
IsothermT
1
isthesinglephaseisotherm,T
c
isthecriticalisothermandT
2
givesthe
isothermbelowthecriticaltemperature.

V
sat
(liq) V
sat
(vap)
V
P
P
sat
c
T
1
>T
c
T
c
T
2
<T
c
Figure 10 PVbehavioursofpurecomponentspredictedbyEOS.
At the critical point , for a pure substance , the equation of state should be such
that:

P
V
P
V
T T
T T c
c
j
(
,
\
,
(
j
(
,
\
,
(


2
2
0

That is the critical isotherm exhibits a horizontal infection point at the critical
point.
Institute of Petroleum Engineering, Heriot-Watt University

TheapplicationoftheseconditionstothevandeWaalsequationyields:

a
R T
P
b
RT
P
c
c c

27
64 8
2 2
and
(34)
EXERCISE 17.
Calculate the critical constants for n- heptane.
Forthecurve,T2<Tc,thepressuredecreasesrapidlyintheliquidregionwithincreasing
V;aftercrossingtheliquidsaturatedlineaminimumoccurs,risestoamaximum
andthendecreasesatthesaturatedvapourline.Realbehaviourdoesnotfollowthis
behaviour.Theycontainahorizontalsegmentwheresaturatedliquidandsaturated
vapourcoexistinvaryingproportions.

Thisequationisnotabletorepresentgaspropertiesoverawiderageoftemperatures
andpressuresandoversubsequentyearsmanyequationshavebeendeveloped.A
numberaregivenincludingthosewhicharefndingfavourintheirapplicationin
thisindustry.
6.3 Benedict-Webb - Rubin Equation (BWR) 1940
Thisequationdevelopedforpurelighthydrocarbonsfoundconsiderableapplication
in predicting thermodynamic properties of natural gases, since natural gases are
essentiallymixturesoflighthydrocarbonsanditcanbewritteninaformsimilarto
VanderWaalsequation.

P
PT
V
B RT A C T
V
bRT a
V
a
V
C
V T V V
o o o
o
+

+

+
+ +
j
(
\
,

j
(
\
,
/
exp
2
2 3
6 3 2 2 2
1

(35)

wherea,b,c,Ao,BoandCoareconstantsforagivengas.
Theseequationsarederivedforpurecomponentsforwhichtheempiricalparameters
need to be obtained. For mixtures mixing rules are required to obtain these
constants.
6.4 Redlich-Kwong Equation 1949
Numerousequationsweredevelopedwithincreasingnumbersofconstantsspecifcto
purecomponents.Morerecentlytherehasbeenamovebacktothecubicequations
likevanderWaals.Wewilldescribebriefythosewhichhavefoundfavourinthe
oilandgassector.
Thismoderndevelopmentofcubicequationsofstatestartedin1949withtheRedlich
andKwongequationwhichinvolvesonlytwoempiricalconstants.
Behaviour of Gases

P =
RT
V- b
a T

( )
+ ( ) V V b
(36)
whereaandbarefunctionsoftemperature.
Theterma(T)dependsonthetemperatureandRedlichKwongexpressedthisasa
functionofthereducedtemperatureTrusing

a T
a
T
c
R
( )

Byapplyingthelimitingconditionatthecriticalpointsyieldsvaluesofacandb
relatedtocriticalconstants.Suchthat;

a
R T
P
b
RT
P
c
c
c
c
c
0 42748 0 08664
2 2
. . and
(37)
6.5 Soave, Redlich Kwong equation
Soave,in1972,modifedtheRedlick-Kwong(RK)equationandreplacedthea/T
0.5

termwithatemperaturedependentterma
T
wherea
T
=a
c
..
TheSoave,Redlich-Kwong(SRK)equationistherefore:

P
RT
V b
a
V V b
c

( )

+ ( )
[ ]

(38)
where
isanondimensionlesstemperaturedependenttermwhichhasavalueof1.0atthe
criticaltemperature.
isobtainedfrom

+
( )
[ ]
1 1
2
2
m T
r
wherem = 0.480 + 1.574 - 0.176
where isthePitzeraccentricfactor .
8
6.6 Peng Robinson Equation of State 1975
PengandRobinsonmodifedpreviousequationsinrelationtotheattractiveterm.
TheyintroducedittoimprovethepredictionsoftheSoavemodifcationinparticular
forthecalculationofliquiddensities.
Institute of Petroleum Engineering, Heriot-Watt University
7

P
RT
V b
a
V V b b V b
c


+ ( ) + ( )
[ ]

(39)

a
R T
P
b
RT
P
c
c
c
c
c
0 457235 0 0778
2 2
. . and
(40)
and
is the same function as for the Soave equation except the function is
different;
where m=0.37464+1.54226w-0.26992w2
Theseequations,inparticulartheSRKandPRequationarewidelyusedinsimulation
software used to predict behaviour in reservoirs, wells and processing. There are
other equations of state which are as competent at predicting physical properties
whichhavebeendevelopedmainlyfocusingontheneedtoimprovetheaccuracyof
liquidvolumespredictions.Thereis,however,greatreluctancetochangefromthose
presentlyusedbecauseoftheinvestmentintheirassociatedparameters.Anexcellent
reviewoftheseequationsandapplicationisgivenbyDanesh
9
.
6.7 Application to Mixtures
Whenpropertiesofmixturesarerequiredmixingrulesarerequiredtocombinethe
datafrompurecomponents.
ForboththeSRKandPRequation

b y b y y a aj k
j j
j
i j i
j i
ij

( )

anda 1
(41)
wherethetermk
ij
istermedthe binary interaction coeffcientswhichareindependent
ofpressureandtemperature.Valuesofbinaryinteractioncoeffcientsareobtained
byfttingequationofstate(EOS)predictionstogas-liquiddataforbinarymixtures.
They have NO physical property signifcance. Each equation has its own binary
interactioncoeffcient.
Effortisunderwayandmethodsexisttonotusebinaryinteractionparametersbutto
usephysicalpropertyrelatedparameterstoenablegoodqualitypredictions.
Behaviour of Gases

EXERCISE 1.
A PVT cell contains 0.01 cu ft ( 00cc) of gas with at composition of ; methane 0.7
mol.frac, ethane 0. and n-butane 0.0. The temperature is increased to 00C.
Use the SRK equation to calculate the pressure at this increased temperature. Use
binary interaction coefficients of C1-nC 0.0, C-nC 0.01 and C1-C 0.0
Solutions to Exercises

EXERCISE 1.
Agascylindercontainsmethaneat1000psiaand70
o
F.Ifthecylinderhasavolume
of 3 cu.ft assuming methane is an ideal gas calculate the mass of methane in the
cylinder.
SOLUTION
PV =nRT
n =m/M
wheren =numberofmoles
m =mass
M =molecularweight
m =PMV/RT

m
psia
lb
lbmole
cuft
psia cuft
lbmole R
R
o
o

( )
j
(
\
,
( )
j
(
\
,
( )
1000 16 04 3
10 73 530
.
.
.
.
Massofmethane,m=8.46lb
Institute of Petroleum Engineering, Heriot-Watt University

EXERCISE 2.
Calculatethedensityofthegasinthecylinderinexercise1.
SOLUTION

g
=
MW.P
RT
g
g
psia
lb
lbmole
psia cuft
lbmole oR
R
Density of gas
lb
cu ft

( )
j
(
\
,
j
(
\
,
( )

1000 16 04
10 73 530
2 82
0
.
.
.
.
, .
. .
EXERCISE 3.
Assumingmethaneisattheconditionsofexercise1,calculatethevolumethegas
wouldoccupyatstandardconditions.
SOLUTION

P V
T
=
P V
T

P V
T
=
P
P

T
T

V
=
1000
1

520 R
530 R
x3ft
= 200.23scf
1 1
1
2 2
2
sc sc
sc
1
sc
sc
1
psia
psia
o
o
3

V
V
V
sc
sc
sc
4 7 .
EXERCISE 4.
Agasismadeupofthefollowingcomponents;25lbofmethane,3lbofethaneand1.5
lbofpropane.Expressthecompositionofthegasinweightandmolefractions.
Behaviour of Gases
0
SOLUTION

Gas
Components
A
Weight
B
Mol weight
C
lb moles
D
Mole fraction
Methane 25 16.04 1.559 0.921
Ethane 3 30.07 0.100 0.059
Propane 1.5 44.09 0.034 0.020
Totals 29.05 1
1
2
3
EXERCISE 5.
Whatistheapparentmolecularweightofthegasinexercise4
SOLUTION

Apparent Molecular weight= 17.43


Gas
Components
A
Mol weight
B
Mol fraction
C
Methane 16.04 0.921 14.77
Ethane 30.07 0.059 1.77
Propane 44.09 0.020 0.89
1.000 17.43
1
2
3
mw
y
i
A*B
EXERCISE 6.
Whatisthegasgravityofthegasinexercise4?
SOLUTION

g
g
air
g
M
M
M

29

g
=AMW=17.43
Gas gravity = 0.6
EXERCISE 7.
Calculatethepseudocriticaltemperatureandpseudocriticalpressureofthemixture
inexercise4.

Institute of Petroleum Engineering, Heriot-Watt University


1
SOLUTION

1
2
3
Gas
Components
A
Mol weight
mw
B
Mole fraction
yi
C
p
c
-psi
D.
T
c
R p
pc
Methane 16.04 0.921 667.00 344 614.3 316.81
Ethane 30.07 0.059 708.00 550 41.7 32.42
Propane 44.09 0.020 616.00 666 12.4 13.39
Total 1.0 668.4 362.6
T
pc
Pseudocriticalpressure=668.4psia
Pseudocriticaltemperature=362
o
R
EXERCISE 8.
Forthegasofexercise4determinethecompressibilityfactoratatemperatureof150
o
Fandapressureof3500psia.
SOLUTION
P
pr
=P/P
pc
,T
pr
=T/T
pc
Fromexercise6P
pc
=668psia,T
pc
=362.6R
P=3500psia,andT=150Cie.610R
Ppr=5.24,andTpr=1.68
FromstandingKatzchart,fgure2
Compressibility factor, z = 0.88
EXERCISE 9.
Calculatethepseudocriticalpropertiesofthegasinexercise4ifitalsocontained3
lbofhydrogensulphide,10lbofcarbondioxideand2.5lbofnitrogen

1
2
3
Gas
Components
Mol
weight
Mole
fraction
p
c
-psi T
c
R p
pc
psia
Methane 25 0.56 16.04 0.035 0.743 667.00 344 495.8 255.70
Ethane 3 0.07 30.07 0.002 0.048 708.00 550 33.7 26.17
Propane 1.5 0.03 44.09 0.001 0.016 616.00 666 10.0 10.81
Hydrogen 3 0.07 34.08 0.002 0.042 1306 673 54.8 28.25
sulphide
Carbon 10 0.22 44.01 0.005 0.108 1071 548 116.1 59.38
Dioxide
Nitrigen 2.5 0.06 28.02 0.002 0.043 493 227 21.0 9.66
Total 45 1.00 0.0466 1.000 731 390
T
pc
Weight Wgt
fraction
lb moles
4
5
6
FromWichert&AzischartforcompositionsofH
2
SandCO
2
r=19

Behaviour of Gases



+
( )

T = T - = 371 R
P = 694.3
pc pc
o
pc
r
r
p
p T
T y y
pc
pc pc
pc H S H S
2 2
1
EXERCISE 10.
Expressthequantityof1lbmoleofagasasstandardcubicfeet.
SOLUTION
EquationofstatePV=RTfor1mole
R=10.732psia.cu.ft/lb.moleRT=60+460=520R,P=14.65psia
or V for 1 lb.mole = RT/P = 380.9 scf/lb.mole.
EXERCISE 11.
Expressthemassofgasinexercise4asstandardcubicfeet.
SOLUTION

Totalmassofgas=29.5lb.
Apparentmol.wgtofgasexercise5=17.43lb./lb.mole
lb.molesofgas=1.6924
Standardcubicfeetofgas=380.9x1.6924
=644.68scf
EXERCISE 12.
Calculatethegasformationfactorforagaswiththecompositionofexercise4existing
atthereservoirconditionsgiveninexercise8.
SOLUTION
T=150
o
Fie610
o
RandP=3500psia
Compressibilityfactorattheseconditionsfromexercise8=0.88
B
g
usingequationabove=0.0008resbbl/scf
Institute of Petroleum Engineering, Heriot-Watt University

EXERCISE 13.
Areservoirexistsatatemperatureof150
o
F(asforexercise8)suitableforstoring
gas.Ithasanarealsizeof5milesby2milesandis200ftthick.Theaverageporosity
is20%andthereisnowaterpresent.Howmuchgasofthecompositionofexercise
4canbestoredatapressurethesameasinexercise8i.e.3500psia.?(1mile=
5280ft.)
SOLUTION
Volumeofreservoirporespace=5x2x(5280)
2
x200x0.2
=11,151,360,000cu.ft.
=1,985,994,657bbls
Bg,exercise11=0.00077299res.bbls/SCF
Volume of gas =2.56923E+12 scf
EXERCISE 14.
Calculatetheviscosityofthegasmixtureinexercise4at200Fandapressureof
oneatmosphere.
SOLUTION

Gas
Components
Mol Weight Mole fraction
y
j
Viscosity
from fig 7

j

M
j
y
j
M
j
Methane
Ethane
Propane

j
y
j
M
j
16.04
30.07
44.09
0.921
0.059
0.020
1.000
0.013
0.0112
0.0098
4.0050
5.4836
6.6400
SUM
3.6884
0.3233
0.1335
4.1451
0.0470
0.0036
0.0013
0.529



mix
j j j
j j
y M
y M

mix
=0.0529/4.1451

mix
=0.01275cp
EXERCISE 15.
Usethegasgravitymethodtocalculatetheviscosityofthegasinexercise4
SOLUTION

Gas
Components
Mol Weight
mw
Mole fraction
y
j
Methane
Ethane
Propane
16.040
30.070
44.090
0.000
0.921
0.059
0.020
1.000
14.7720
1.773
0.886
17.431
Behaviour of Gases


g
=AMW/M
air

g
=AMW/29Temperature=150F
Molweightair= 29.000
AMWofgas= 17.431
GasGravity= 0.601

g
= 0.01265fromfg8
EXERCISE 16.
Determinetheviscosityofthegasinexercise4at150
o
Fand3500psia(refex4,7,
&8)
SOLUTION
Fromexercise7

P
T
P
T
T
pc
pc
r
r
c

668 4
362 6
.
.
P
P
=
3500
668.4
= 5.24
T =
610
362.6
=1.68
c
FromLeecorrelation

/atmos=1.75
Viscosityatatmosphericpressure
Fromexercise13and14= 0.01275cp
Viscosityatconditions =0.0223cp
EXERCISE 17.
Calculatethecriticalconstantsforn-heptane.
SOLUTION
R=10.732.T
c
forheptane=973
o
RandP
c
=397psia
Usingequationsabovea=115,872cuft
2
/lbmole
andb=3.2878cuft./lbmole
Institute of Petroleum Engineering, Heriot-Watt University

EXERCISE 18.
A PVT cell of volume 0.01 cu ft ( 300cc) contains 0.008 lb mole. of gas with
a composition of; methane 0.67 mol.frac, ethane 0.235 and n-butane 0.05. The
temperatureisincreasedto300C.UsetheSRKequationtocalculatethepressure
at this increased temperature. Use binary interaction coeffcients of C1-nC4 0.02,
C2-nC40.01andC1-C20.0
SOLUTION
Calculatetheconstantsaandbforeachcomponent

a
R T
P
b
RT
P
m T
c
c
c
c
c
r

+
( )
[ ]
0 42748 0 08664
1 1
2 2
2
. . and
wherem = 0.480 + 1.574 - 0.176 2

Components p
c
a
c
m a a=a* y T
c
R) b
j
Methane
ethane
n-butane
0.67 344 667 0.4759 8735 0.0104 0.49635 0.57546 5027
0.235
0.05
550
766
708
551
0.7223
1.2926
21036
52429
0.0979
0.1995
0.63241
0.78701
0.79033
1.00619
16625
52753
Nowcalculatethemixturevalues.

b y b y y a aj k
j j
j
i j i
j i
ij

( )

anda
wherea = (1- k )(a a )
ij ij i j
0.5
1

Components k
ij
Methane
k
ij
n-butane
a
ij
Methane
a
ij
ethane
a
ij
n-butane
sum yi b k
ij
ethane
Methane
ethane
n-butane
0.67 0.312 0.00 0.00 0.02 2123.7 1485.5 1037.29 4646.52
0.235
0.05
0.181
0.129
0.00 0.01
0.00
1485.5
1037.3
1039.1
732.97
732.969
527.535
3257.56
2297.8
1 0.622 sum 10201.9
NowuseSRKtocalculatepressure.
Behaviour of Gases

P
RT
V b
a
V V b
c

( )

+ ( )
[ ]

V = 1.25cuft / lbmole
b = 0.622a = 10201.9
P = 8617.6psia
m
c
REFERENCES
1.StandingMBandKatzDLDensityofNaturalGases.TransAIME,146(1942).
p140
2.PoettmannFHandCarpenterPGTheMultiphaseFlowofGasandWaterthrough
VerticalFlowStringswithApplicationtotheDesignofGasLiftInstallations.
APIDrillingandProductionPractise.1952,pp279-91
3.BrownGGetal.NaturalGasolineandVolatileHydrocarbonsNationalGasoline
Assoc.ofAmerica,Tulsa,Okl.1948
4. Wichert,EandAziz,KCalculateZsforsourgasesHydProc.(May1972)
51,119-122
5.Katz,D.L.,HandbookofNaturalGasEngineering,McGrawHill,NY,1959
6.CarrNetal.Viscosityofnaturalgasesunderpressure.TransAIME201,264,
(1954)
7.LeeetalTheviscosityofnaturalgases.TransAIME1966237,997-1000
8.PitzerKSetalTheVolumetricandThermodynamicPropertiesofFluidsII.
CompressibilityFactor,VapourPressureandEntropyofVaporisation.J.Am.
Chem.Soc.(1955)77,No13,3433-3440
9.Danesh,APVTandPhaseBehaviourofPetroleumReservoirFluids.1998
ElsevierISBN:0444821961p129-162
CONTENTS
1 COMPOSITIONBLACKOILMODELS
2 GASSOLUBILITY,R
s
3 OILFORMATIONVOLUMEFACTOR,B
o
4 TOTALFORMATIONVOLUMEFACTOR,B
T
5 BELOWTHEBUBBLEPOINT
6 OILCOMPRESSIBILITY
7 BLACKOILCORRELATIONS
8 FLUIDDENSITY
8.1 SpecifcGravityofaLiquid
8.2 DensityBasedonIdealSolutionPrinciples
9 FORMATIONVOLUMEFACTOROFGAS
CONDENSATE,B
gc
10 VISCOSITYOFOIL
11 INTERFACIALTENSION
12 COMPARISONOFRESERVOIRFLUID
MODELS
Properties of Reservoir Liquids

LEARNING OBJECTIVES
Having worked through this chapter the Student will be able to:
Defnegassolubility,R
s
andplotvs.Pforareservoirfuid.
Defneundersaturatedandsaturatedoil.
Explainbriefyfashanddifferentialliberation
Defne the oil formation volume factor B
o
, and plot B
o
vs. P for a reservoir
fuid.
DefnetheTotalFormationVolumefactorB
t
,andplotB
t
vs.PalongsideaBo
vs.Pplot.
PresentanequationtoexpressB
t
intermsofB
o
,R
s
andB
g
.
Expressoilcompressibilityintermsofoilformationvolumefactor.
Use black oil correlations and their graphical form to calculate fluid
properties.
Calculatethedensityofareservoirfuidmixture,usingidealsolutionprinciples,
atreservoirpressureandtemperature,usingdensitycorrectionchartforC1&
C2andotherprerequisitedata.
Defnetheformationvolumefactorofagascondensate
Calculatethereservesandproductionofgasandcondensateoperatingabove
thedewpoint,givenprerequisitedata.
Useviscosityequationsandcorrelationstocalculateviscosityoffuidatreservoir
conditions.
Calculatetheinterfacialtensionofequilibriumgas-oilsystemsgivenprerequisite
equationsanddata.
List the comparisons of the black oil and compositional model in predicting
liquidproperties
Institute of Petroleum Engineering, Heriot-Watt University

1 COMPOSITION - BLACK OIL MODEL


As introduced in the chapter on Composition, petroleum engineers are requiring
acompositionaldescriptiontooltouseasabasisforpredictingreservoirandwell
fuidbehaviour.Thetwoapproachesthatarecommonlyusedarethemulticomponent
compositional modeldescribedintheearlierchapterandthetwocomponentblack oil
model.Thelattersimplisticapproachhasbeenusedformanyyearstodescribethe
compositionandbehaviourofreservoirfuids.ItiscalledtheBlack Oil Model.
Theblackoilmodelconsidersthefuidbeingmadeupoftwocomponents-gasdissolved
inoilandstocktankoil.Thecompositionalchangesinthegaswhenchangingpressure
andtemperatureareignored.Tothoseappreciatingthermodynamicsthissimplistic
twocomponentmodelisdiffculttocopewith.TheBlackOilModel,illustratedin
Figure1,isatthecoreofmanypetroleumengineeringcalculations,andassociated
proceduresandreports.
AssociatedwiththeblackoilmodelareBlackOilmodeldefnitionsinrelationto
Gas Solubility and Formation Volume Factors.

Reservoir Fluid
Solution Gas
Stock Tank Oil
/ = R
s
/ = B
o
B
o
= Oil Formation Volume Factor
R
s
= Solution Gas to Oil Ratio
Figure 1 "BlackOilModel"
Properties of Reservoir Liquids

2 GAS SOLUBILITY
Althoughthegasassociatedwithoilandtheoilitselfaremulticomponentmixtures
itisconvenienttorefertothesolubilityofgasincrudeoilasifweweredealingwith
atwo-componentsystem.
The amount of gas forming molecules in the liquid phase is limited only by the
reservoirconditionsoftemperatureandpressureandthequantityoflightcomponents
present.
The solubility is referred to some basis and it is customary to use the stock tank
barrel.
Solubility = f(pressure,temperature,compositionofgas
compositionofcrudeoil)
Forafxedgasandcrude,atconstantT,thequantityofsolutiongasincreaseswith
p,andatconstantp,thequantityofsolutiongasdecreaseswithT

Ratherthandeterminetheamountofgaswhichwilldissolveinacertainamountof
oilitiscustomarytomeasuretheamountofgaswhichwillcomeoutofsolutionas
thepressuredecreases.Figure2illustratesthebehaviourofanoiloperatingoutside
thePTphasediagraminitssinglephasestatewhenthereservoirpressureisabove
itsreservoirbubblepointat1.Fluidbehaviourinthereservoirissinglephaseand
theoilissaidtobeundersaturated.Inthiscaseaslightreductionofpressurecauses
thefuidtoremainsinglephase.Iftheoilwasontheboundarybubblepointpressure
lineat2thenafurtherreductioninpressurewouldcausetwophasestobeproduced,
gasandliquid.Thissaturatedfuidisonethatuponaslightreductionofpressure
somegasisreleased.Theconceptofgasbeingproducedorcomingoutofsolution
givesrisetothisgassolubilityperspective.Clearlywhenthefuidsareproducedto
thesurfaceasshownbytheundersaturatedoilinfgure2thesurfaceconditionslie
withinthetwophaseareaandgasandoilareproduced.Thegasproducedistermed
solution gasandtheoilatsurfaceconditionsstock tank oil.Thesearethetwocom-
ponentsmakingupthereservoirfuid,clearlyaverysimplisticconcept.
The gas solubility R
s
is defned as the number of cubic feet (cubic metre) of gas
measured at standard conditions, which will dissolve in one barrel (cubic metre)
of stock tank oil when subjected to reservoir pressure and temperature.
Inmetricunitsthevolumesareexpressedascubicmetreofgasatstandardconditions
whichwilldissolveinonecubicmetreofstocktankoil.
Institute of Petroleum Engineering, Heriot-Watt University

Solution Gas
Stock Oil Tank
Oil Reservoir
Oil and Dissolved Gas
R
si
scf/stb
1 st b. oil
B
o
rb.oil
P
r
e
s
s
u
r
e
Temperature
Pi 1
2
P
+
Surface
Phase Diagram
Figure 2 Productionofreservoirhydrocarbonsabovebubblepoint
Figure3givesatypicalshapeofgassolubilityasafunctionofpressureforareser-
voirfuidatreservoirtemperature.Whenthereservoirpressureisabovethebubble
pointpressurethentheoilisundersaturated,i.e.capableofcontainingmoregas.As
thereservoirpressuredropsgasdoesnotcomeoutofsolutionuntilthebubblepoint
isreached,overthispressurerangethereforethe gas in solution is constant.Atthe
bubblepointpressure,correspondingtothereservoirtemperature,twophasesare
produced,gasandoil.Thegasremaininginsolutionthereforedecreases.
Thenatureoftheliberationofthegasisnotstraightforward.Withinthereservoir
whengasisreleasedthenitstransportandthatoftheliquidisinfuencedbytherelative
permeabilityoftherock(discussedinChapter10).Thegasdoesnotremainwithits
associatedoili.e.thesystemchanges.Intheproductiontubingandintheseparator
itisconsideredthatthegasandassociatedliquidremaintogetheri.e.thesystemis
constant.Theamountofgasliberatedfromasampleofreservoiroildependsonthe
conditionsoftheliberation.Therearetwobasicliberationmechanisms:
Properties of Reservoir Liquids

1000 2000 3000


200
600
400
Pressure (psig)
P
b
R
si
R
s


s
c
f
/
s
t
b
Figure 3 SolutionGas-OilRatioasaFunctionofPressure.
Flashliberation - thegasisevolvedduringadefnitereductionin
pressureandthegasiskeptincontactwiththeliquid
untilequilibriumhasbeenestablished.
Differentialliberation - thegasbeingevolvedisbeingcontinuously
removedfromcontactwiththeliquidandtheliquidisin
equilibriumwiththegasbeingevolvedoverafnite
pressurerange.
ThetwomethodsofliberationgivedifferentresultsforR
s
.Thistopiciscoveredin
moredetailinthePVTanalysischapter.
Productionofacrudeoilatreservoirpressuresbelowthebubblepointpressureoccurs
byaprocesswhichisneitherfashordifferentialvaporisation.Onceenoughgasis
presentforthegastomovetowardthewellborethegastendstomovefasterthanthe
oil.Thegasformedinaparticularporetendstoleavetheliquidfromwhichitwas
formedthusapproximatingdifferentialvaporisation,however,thegasisincontactwith
liquidthroughoutthepaththroughthereservoir.Thegaswillalsomigratevertically
asaresultofitslowerdensitythantheoilandcouldformasecondarygascap.
Fluidproducedfromreservoirtothesurfaceisconsideredtoundergoafashprocess
wherethesystemremainsconstant.
3 OIL FORMATION VOLUME FACTOR, B
o
Thevolumeoccupiedbytheoilbetweensurfaceconditionsandreservoirorother
operatingchangesisthatofthetotalsystem;thestocktankoilplusitsassociated
ordissolvedsolutiongas.Theeffectofpressureonthecomplexstocktankliquid
andthesolutiongasistoinducesolutionofthegasintheliquiduntilequilibriumis
reached.Aunitvolumeofstocktankoilbroughttoequilibriumwithitsassociated
Institute of Petroleum Engineering, Heriot-Watt University

gasatreservoirpressureandtemperaturewilloccupyavolumegreaterthanunity
(unlesstheoilhasverylittledissolvedgasatveryhighpressure).
The relationship between the volume of the oil and its dissolved gas at reservoir
conditiontothevolumeatstocktankconditionsiscalledthe Oil Formation Volume
Factor B
o
.TheshapeoftheB
o
vs.pressurecurveisshowninFigure4.Itshows
thatabovethebubblepointpressurethereductioninpressurefromtheinitialpres-
surecausesthefuidtoexpandasaresultofitscompressibility.Thisrelatestothe
chapteronPhaseBehaviourwhereforanoilthePVdiagramshowsalargedecline
in pressure for a small increase in volume, being again an indication of the com-
pressibilityoftheliquid.Belowthebubblepointpressurethisexpansiondueto
compressibilityoftheliquidissmallcomparedtotheshrinkageoftheoilasgasis
releasedfromsolution.
The oil formation volume factor, is the volume in barrels (cubic metres) occupied in
the reservoir, at the prevailing pressure and temperature, by one stock tank barrel
(one stock tank cubic metre) of oil plus its dissolved gas.

1000 2000 3000


1.0
1.2
1.1
Pressure (psig)
P
b
B
o


r
b
.
/
s
t
b
Units - rb (oil and dissolved gas)
Figure 4 Oilformationvolumefactor
Theseblackoilparameters,B
o
andRsareillustratedinFigure5a,b,&cfromCraft
andHawkins
1
reservoirengineeringtext.,wheretheypresenttheR
s
andB
o
curvefor
theBigSandyfeldintheUSA.Thevisualconceptofthechangesduringpressure
andtemperaturedecreaseisalsopresented.
Properties of Reservoir Liquids

P
01
P
01
= 3500 PSIA
T
01
= 160 F
A
P
B
= 2500 PSIA
T
01
= 160 F
B
P

= 1200 PSIA
T
01
= 160 F
C
P
A
= 14.7 PSIA
T
01
= 160 F
D
P
A
= 14.7 PSIA
T
01
= 60 F
E
P
B
P
P
A
P
A
Free Gas
676 Cu. Ft. Free Gas
2.990 Cu. Ft.
Free Gas
567 Cu. Ft.
1,000 BBL 1,040 BBL 1,210 BBL 1,333 BBL 1,310 BBL
567SCF/STB
AT 1200 PSIA
R
S
= 337
B
U
B
B
L
E

P
O
I
N
T

P
R
E
S
S
U
R
E
I
N
I
T
I
A
L

P
R
E
S
S
U
R
E
S
o
l
u
t
i
o
n

G
a
s
,

S
C
F
/
S
T
B
600
500
400
300
200
100
0
0 500 1000 1500 2000
Pressure, PSIA
2500 3000 3500
(a)
(b)
Figure 5 GastooilratioandoilformationvolumefactorforBigSandyFieldreservoir
oil
1
.

F
o
r
m
a
t
i
o
n

V
o
l
u
m
e

F
a
c
t
o
r
,

B
B
L
/
S
T
B
0 500
1.40
1.30
1.20
1.10
1.00
1000 1500 2000
Pressure, PSIA
2500 3000 3500
B
U
B
B
L
E

P
O
I
N
T

P
R
E
S
S
U
R
E
I
N
I
T
I
A
L

P
R
E
S
S
U
R
E 1200 PSIA
B
O
= 1.210
14.7 PSIA & 160 F
B
O
= 1.040
2500 PSIA
B
OB
= 1.333
3500 PSIA
B
OI
= 1.310
14.7 PSIA & 60 F
B
O
= 1.000
(b)
Figure 5b
Institute of Petroleum Engineering, Heriot-Watt University

Thereciprocaloftheoilformationvolumefactoriscalledtheshrinkagefactorb
o

b
B
o
o

1

TheformationfactorB
o
maybemultipliedbythevolumeofstocktankoiltofnd
the volume of reservoir required to produce that volume of stock tank oil. The
shrinkagefactorcanbemultipliedbythevolumeofreservoiroiltofndthestock
tankvolume.
Itisimportanttonotethatthemethodofprocessingthefuidswillhaveaneffect
ontheamountofgasreleasedandthereforeboththevaluesofthesolutiongas-oil
ratioandtheformationvolumefactor.AreservoirfuiddoesnothavesingleB
o
orR
s

values.B
o
&R
s
aredependantonthesurfaceprocessingconditions.Thissimplistic
reservoirmodel(Figure6)demonstratesthattheblackoilmodeldescriptionofthe
reservoirfuidsisanaftertheevent,processing,descriptionintermsoftheproduced
fuids.Thissimplisticapproachtomodellingreservoirfuidsbecomesmorediffcult
toconsiderwhenoneisinvolvedinreservoirswhichbecomepartofatotalreservoir
system(Figure7).

R
s
B
O
Figure 6 Blackoildescriptionofreservoirfuid
Properties of Reservoir Liquids
10

R
s
3
B
o
3
R
s
2
B
o
2
R
s
4
B
o
4
R
s
1
R
s
B
o
B
o
1
?
Multi Reservoir System
Figure 7 Integratedsystemofreservoircommonpipelineandfnalcollectionsystem.
4 TOTAL FORMATION VOLUME FACTOR, B
t
Inreservoirengineeringitissometimesconvenienttoknowthevolumeoccupied
inthereservoirbyonestocktankbarrelofoilplusthefreegasthatwasoriginally
dissolved in it. A factor is used called the total formation-volume factor B
t
, or
thetwo-phasevolume-factorandisdefned as the volume in barrels that 1.0 STB
and its initial complement of dissolved gas occupies at reservoir temperature and
pressure,i.e.itincludesthevolumeofthegaswhichhasevolvedfromtheliquid
andisrepresentedby:
B
g
(R
sb
-R
s
)
i.e. B
t
=B
o
+B
g
(R
sb
-R
s
) (1)
R
sb
=thesolutiongastooilratioatthebubblepoint
Institute of Petroleum Engineering, Heriot-Watt University
11

Oil
Oil
Gas
Hg
B
0
B
t
B
0b
B
g
(R
sb
-R
s
)
Figure 8a Totalformationvolumefactorortwophasevolumefactor
ItsapplicationcomesfromtheMaterialBalanceequation(Chapter15)whereitis
sometimesusedtoexpressthevolumeofoilandassociatedgasasafunctionofpres-
sure.ItisimportanttonotethatB
t
doesnothavevolumesignifcanceinreservoir
termssincetheassumptioninB
t
isthatthesystemremainsconstant.Asmentioned
earlier if the pressure drops below the bubble point in the reservoir then the gas
comingoutofsolutionmovesawayfromitsassociatedoilbecauseofitsfavourable
relativepermeabilitycharacteristics.
Figure8bgivesacomparisonofthetotalformation-volumefactorwiththeoilfor-
mation-volumefactor.ClearlyaboveP
b
thetwovaluesareidenticalsincenofree
gasisreleased.BelowP
b
thedifferencebetweenthevaluesrepresentsthevolume
occupiedbyfreegas.

B
o
B
t
Pressure P
b
Figure 8b Totalandoilformationvolumefactor

ThevalueofB
T
canbeestimatedbycombiningestimatesofB
O
andcalculationof
B
g
andknownsolubilityvaluesforthepressuresconcerned.
Properties of Reservoir Liquids
1
5 BELOW THE BUBBLE POINT
Figure 9 depicts the behaviour below the bubble point when produced gas at the
surfacecomesfromtwosources,thesolutiongasassociatedwiththeoilenteringthe
wellboreplusfreegaswhichhascomeoutofsolutioninthereservoirandmigratedto
thewellbore.Thetotalproducinggastooilratioismadeupofthetwocomponents
solutiongasR
s
andthefreegaswhichisthedifference.Thediagramillustratesthe
volumesoccupiedbythesetwointhereservoir,thesolutiongasbeingpartofB
o
and
thefreegasvolumethroughB
g
.

Free Gas
& Solution Gas
Stock Oil Tank
Oil Reservoir
rb (oil and dissolved gas) /stb
1 st b. oil
B
o

P
r
e
s
s
u
r
e
Temperature
R= R
s
+ (R - R
s
)
+
(R - R
s
) B
g
Gas Oil
Reservoir
rb (free gas) /stb
Surface
Pi
P
Figure 9 Productionofreservoirhydrocarbonsbelowbubblepoint
6 OIL COMPRESSIBILITY
Thevolumechangesofoilabovethebubblepointareverysignifcantinthecontext
ofrecoveryofundersaturatedoil.Theoilformationvolumefactorvariationsabove
thebubblepointrefectthesechangesbuttheyaremorefundamentallyembodied
inthecoeffcientofcompressibilityoftheoil,oroilcompressibility.
Theequationforoilcompressibilityis

c
V
V
P
o
T

j
(
,
\
,
(
1

intermsofformationvolumefactorsthisequationyields
Institute of Petroleum Engineering, Heriot-Watt University
1

c
B
B
P
o
o
o
T

j
(
,
\
,
(
1
Assumingthatthecompressibilitydoesnotchangewithpressuretheaboveequation
canbeintegratedtoyield;

c P P
V
V
o 2 1
2
1
( ) ln

whereP
1
&P
2
,andV
1
&V
2
representthepressureandvolumeatconditions1&2.
7 BLACK OIL CORRELATIONS
Overtheyearstherehavebeenmanycorrelationsgeneratedbasedonthetwocom-
ponent based black oil model characterisation of oil. The correlations are based
ondatameasuredontheoilsofinterest.Theseempiricalcorrelationsrelateblack
oilparameters,thevariablesofB
o
andR
s
to;reservoirtemperature,andoilandgas
surfacedensity.Itisimportanttoappreciatethatthesecorrelationsareempiricaland
areobtainedbytakingagroupofdataforaparticularsetofoilsandfndingabestft
correlation.Usingthecorrelationforfuidswhosepropertiesdonotfallwithinthose
forthecorrelationcanresultinsignifcanterrors.Danesh
2
hasgivenanexcellent
reviewofmanyofthesecorrelations
Anumberofempiricalcorrelations,basedonlargelyUScrudeoils,andotherloca-
tionsacrosstheworldhavebeenpresentedtoestimateblackoilparametersofgas
solubilityandoilformationvolumefactor.ThemostcommonlyusedisStandings
3
correlation.Othercorrelationsinclude,Lasater
4
,andrecentlyGlaso
6

P
b
=f(R
s
,
g
,p
o
,T)
where P
b
=bubblepointpressureatT
o
F
R
s
=solutiongas-oilratio(cuft/bbl)

g
=gravityofdissolvedgas

o
=densityofstock-tankoil.(specifcgravity)

StandingscorrelationforthecalculationofP
b
,bubblepointpressureis:

P
R
T API
b
s
g

j
(
,
\
,
(

,

,
,
]
]
]
]
. ( . . ( )) .
.
18 2 0 00091 0 0125 1 4
0 83
10

(2)

Hiscorrelationfortheoilformationvolumefactoris;

B R T
o s
g
o
+
j
(
,
\
,
(
+
,

,
,
]
]
]
]
. . .
.
.
0 9759 0 000120 1 25
0 5
1 2

(3)
Properties of Reservoir Liquids
1

Standing's correlations have been presented as nomographs enabling quick look


predictionstobemade.Figures10&11givethenomogramformsofthesecorrelations
forgassolubilityandoilformationvolumefactor.Standingscorrelationisbasedon
asetof22Californiacrudes.
OthercorrelationshavebeenpresentedbyLasater
4
basedon137Canadian,USA
andSouthAmericancrudes,VasquezandBeggs
5
using6000datapoints,Glaso
6
us-
ing45NorthSeacrudesamples,andMahoun
7
whoused69MiddleEasterncrudes.
Danesh
2
givesaveryusefultableshowingtherangescoveredbytherespectiveblack
oilcorrelations
Institute of Petroleum Engineering, Heriot-Watt University
1
Figure 10 Oil-formationvolumefactorasafunctionofgassolubility,temperature,gas
gravityandoilgravity(Standing)
20
30
40
50
60
70
80
90
100
150
200
300
400
500
600
700
800
900
1000
1500
2000
1.02
1.03
1.04
1.05
1.06
1.07
1.08
1.09
1.10
Formation volume of bubble-point liquid
G
a
s
-
o
i
l

r
a
t
i
o
,

c
u

f
t

p
e
r

b
b
l
b
b
l

p
e
r

b
b
l

o
f

t
a
n
k

o
i
l
1.20
1.30
1.40
1.50
1.60
1.70
1.80
1.90
1
.
1
0
1
.
2
0
1
.
3
0
1
.
4
0
1
.
5
0
0
.
5
0
0
.
6
0
0
.
7
0
0
.
8
0
0
.
9
0
1
.
0
0
G
a
s

g
r
a
v
i
t
y
A
i
r
=
1
Tank oil gravity, API
50
30
10
Temperature, F
100
140
160
180
200
220
240
260
120
Properties of Reservoir Liquids
1
Figure 11 Gassolubilityasafunctionofpressure.Temperature,gasgravityandoil
gravity

6
0
0
5
0
0
4
0
0
3
0
0
2
0
0
2
0
3
0
4
0
5
0
6
0
7
0
8
0
9
0
1
0
0
1
5
0
2
0
0
3
0
0
4
0
0
5
0
0
6
0
07
0
0
7
0
0
8
0
09
0
0
1
0
0
0
1
5
0
0
2
0
0
0
3
0
0
0
4
0
0
0
5
0
0
0
6
0
0
0
8
0
0
9
0
01
0
0
0
1
5
0
0
2
0
0
0
T
a
n
k

o
i
l

g
r
a
v
i
t
y
,

A
P
I

T
e
m
p
e
r
a
t
u
r
e
,

F

G
a
s

g
r
a
v
i
t
y

A
i
r

=

1

6
0
1
.
5
0
1
.
4
0
1
.
3
0
1
.
2
0
8
0
1
0
0
1
2
0
1
4
0
1
6
0
1
8
0
2
0
0
2
2
0
2
4
0
2
6
0
1
.
1
0
1
.
0
0
0
.
9
0
0
.
8
0
1
0
1
4
1
6
1
8
2
0
2
2
2
4
2
6
2
8
3
0
3
2
3
4
3
6
3
8
4
0
1
2
4
2
4
4
4
6
4
8
5
0
5
2
5
4
5
6
5
8
Bubble-point P
r
e
s
s
u
r
e
,

p
s
i
a
G
a
s
-
o
i
l

r
a
t
i
o
,

c
u

f
t

p
e
r

b
b
l
6
0
(STANDING)
0
.
7
0
0
.
6
0
0
.
5
0
Institute of Petroleum Engineering, Heriot-Watt University
1

Correlation Standing Lasater Vasquez-Beggs Glaso Marhoun


Ref 3 4 5 6 7
Bubble - point pressure (psia) 130-7000 45-5780 15-6055 165-7142 130-3573
Temperature, F 100-258 82-272 162-180 80-280 74-240
Bo 1.024-2.15 1.028-2.226 1.025-2.588 1.032-1.997
Gas - oil ratio (scf/stb) 20-1425 3-2905 0-2199 90-2637 26-1602
Oil Gravity, oAPI 16.5-63.8 17.9-51.1 15.3-59.5 22.3-48.1 19.4-44.6
Gas Gravity 0.59-0.95 0.574-1.22 0.511-1.651 0.65-1.276 0.752-1.367
Separator Pressure 265-465 15-605 60-565 415
Searator Temperature F 100 36-106 76-150 125
Table 1 Blackoilcorrelationandtheirrangesatapplication
2
8 FLUID DENSITY
Liquidshaveamuchgreaterdensityandviscositythangases,andthedensityisaffected
muchlessbychangesintemperatureandpressure.Forpetroleumengineersitis
importantthattheyareabletoestimatethedensityofareservoirliquidatreservoir
conditions.
8.1 Specifc Gravity of a Liquid

o
o
w

(4)
Thespecifcgravityofaliquidistheratioofitsdensitytothatofwaterbothatthe
sameT&P.Itissometimesgivenas60/60,i.e.bothliquidandwateraremeasured
at60and1atmos.
ThepetroleumindustryusesanothertermcalledAPI gravity where

API
o
141 5
131 5
.
.

(5)
where
o
isspecifcgravityat60/60.
Thereareseveralmethodsofestimatingthedensityofapetroleumliquidatreservoir
conditions.Themethodsuseddependontheavailabilityandnatureofthedataof
data.Whenthereiscompositionalinformationonthereservoirfuidthenthedensity
canbedeterminedusingtheideal solution principle. Whentheinformationwehave
isthatoftheproducedoilandgasthenempiricalmethodscanbeusedtocalculate
thedensityofthereservoirfuid.
8.2 Density based on Ideal Solution Principles
Mixturesofliquidhydrocarbonsatatmosphericconditionsbehaveasidealsolutions.
Anidealsolutionisahypotheticalliquidwherenochangeinthecharacterofthe
liquidsiscausedbymixingandthepropertiesofthemixturearestrictlyadditive.
Properties of Reservoir Liquids
1
Petroleumliquidmixturesaresuchthatideal-solutionprinciplescanbeappliedforthe
calculationofdensitiesandthisenablesthevolumeofamixturefromthecomposi-
tionandthedensityoftheindividualcomponents.Theprincipleisillustratedusing
thefollowingexercise.Dataforthespecifccomponentsaregiveninthetablesat
theendofthechapter
ExErcIsE 1.
calculate the density at 1.psia and 0 F of the hydrocarbon liquid mixture with
the composition given below:
Component Mol.
fract.
1b mol.

nC4 0.25
nC5 0.32
nC6 0.43
1.00
solUtIon ExErcIsE 1

Solution
Component Mol. Mol. Weight Liquid Liquid
density
fract. weight 1b Density at volume
1b mol. 1b/1b at 60F and 14.7 cu ft
mol. psia
1b/cu ft
nC4 0.25 58.1 14.525 36.45 0.3985
nC5 0.32 72.2 23.104 39.36 0.5870
nC6 0.43 86.2 37.066 41.43 0.8947
____ _____ _____
1 74.695 1.8801
Liquidsattheirbubblepointorsaturationpressurecontainlargequantitiesofdis-
solvedgaswhichatsurfaceconditionsaregasesandthereforesomeconsideration
forthesemustbegivenintheadditivevolumetechnique.Thisphysicallimitation
doesnotimpairthemathematicaluseofapseudoliquiddensityformethaneand
ethane since it is only a step in its application to determine a reservoir condition
density.Thisisachievedbyobtainingapparentliquiddensitiesforthesegasesand
determiningapseudoliquiddensityforthemixtureatstandardconditionswhichcan
thenbeadjustedtoreservoirconditions.
Standing&Katz
8
carriedoutexperimentsonmixturescontainingmethaneplusother
compoundsandethaneplusothercompoundsandfromthiswereabletodetermine
apseudo-liquid(fctitious)densityformethaneandethane
Institute of Petroleum Engineering, Heriot-Watt University
1
Correlationshavebeenobtainedbyexperimentgivingapparentliquiddensitiesof
methaneandethaneversusthepseudoliquiddensity(Figure12).

0.1
0.2
0.3
0.4
0.5 0.6 0.7 0.8 0.9
0.3
0.4
0.5
0.6
0.4 0.3
Density of system, 60F B atm. pressure
A
p
p
a
r
r
e
n
t

d
e
n
s
i
t
y

o
f

M
e
t
h
a
n
e
,

g
/
c
c
A
p
p
a
r
r
e
n
t

d
e
n
s
i
t
y

o
f

o
f

E
t
h
a
n
e
,

g
/
c
c
Ethane - N - Butane
Ethane - Heptane
Ethane - Crystal oil
Methane - Cyclo Hexane
Methane - Crude oil
Methane - Crystal oil
Methane - Propane
Methane - Hexane
Methane - Pentane
Methane - Heptane
Methane - Benzene
Figure 12 Variationofapparentdensityofmethaneandethanewithdensityofthesystem
8
.
Tousethecorrelationsatrialanderrortechniqueisrequiredwherebythedensityof
thesystemisassumedandtheapparentliquiddensitiescanbedetermined.These
liquiddensitiesarethenusedtocomputethedensityofthemixturebyadditivevol-
umesandthevaluecheckedagainsttheinitialassumption.Theprocedurecontinues
untilthetwovaluesarethesame.
Whennonhydrocarbonsarepresent,theprocedureistoaddthemolefractionsof
thenitrogentomethane,themolefractionofcarbondioxidetoethaneandthemole
fractionofhydrogensulphidetopropane.
Properties of Reservoir Liquids
0
ExErcIsE :
Calculate the surface pseudo liquid density of the following reservoir
composition.

Component Mole percent

Methane 44.04
Ethane 4.32 Properties of
Propane 4.05 heptane +
Butane 2.84 API gravities = 34.2
Pentane 1.74 SG = 0.854
Hexane 2.9 Mol wt = 164
Heptane + 40.11

solUtIon ExErcIsE


Estimate

44.65 lb/cu ft. 0.716 gm/cc lb/cuft


From fig 12 Density 0.326 20.3424
C1
Density 0.47 29.328
C2

Component Mole Mol Weight Liq Liquid
fraction Weight Density Volume
lb/lb lb at 60F &
mole 14.7 psia
lb/cu.ft cu ft.
z M zM o zM/o
Methane 0.4404 16 7.0464 20.3424 0.34639
Ethane 0.0432 30.1 1.30032 29.328 0.04434
Propane 0.0405 44.1 1.78605 31.66 0.05641
Butane 0.0284 58.1 1.65004 35.78 0.04612
Pentane(n&i) 0.0174 72.2 1.25628 38.51 0.03262
Hexane(n&i) 0.029 86.2 2.4998 41.43 0.06034
Heptane+ 0.4011 164 65.7804 53.26 1.23508
Total 1 81.31929 1.8213

Density = 81.32 lb / 1.82 cu ft
= 44.65 lb/cu.ft
ThistrialanderrormethodisverytedioussoStandingandKatzdevisedachartwhich
removesthetrailanderrorrequiredinthecalculation.Thedensitieshavebeencon-
vertedintothedensityoftheheaviercomponents,C
3+
,andtheweightpercentofthe
twolightcomponents,methaneandethaneintheC
1+
andC
2+
mixtures.Figure13.
Institute of Petroleum Engineering, Heriot-Watt University
1

70
60
50
40
30
10
20
30
40
50
60
70
D
e
n
s
i
t
y

o
f

s
y
s
t
e
m

i
n
c
l
u
d
i
n
g

m
e
t
h
a
n
e

a
n
d

e
t
h
a
n
e
,

l
b
/
c
u

f
t
D
e
n
s
i
t
y

o
f

p
r
o
p
a
n
e

p
l
u
s
,

l
b
/
c
u

f
t
W
t

%

e
t
h
a
n
e

i
n

e
t
h
a
n
e

p
l
u
s

m
a
t
e
r
i
a
l

0 10 20 30 40 50
W
t

%

m
e
t
h
a
n
e

i
n

e
n
t
i
r
e

s
y
s
t
e
m

0
10
20
30
Figure 13 Pseudo-liquiddensityofsystemscontainingmethaneandethane
10
.
Weshallexaminethroughexamplesvariouswaysofcalculatingdownholereservoir
fuidsdensitiesdependantonthedataavailable.
Thethreeconsideredare:
1.Thecompositionofthereservoirfuidisknown.
2.Thegassolubility,thegascompositionandthesurfaceoilgravityisknown
3.Thegassolubility,andgasandliquidgravitiesareknown.
1. The composition of the reservoir fuid is known.
Theprocedureisillustratedusingthefollowingtwoexercises.
Properties of Reservoir Liquids

ExErcIsE .
calculate the surface density of the mixture in exercise using the chart of figure 1
Thepseudodensityis convertedtoreservoirconditionsfrstly bytakingtheeffect
ofpressureandsecondlyaccountingfortheeffectoftemperature.Thevariationof
density with respect to pressure and temperature has been investigated and it has
beendemonstratedthatthermalexpansionisnotaffectedbypressure.Standing&
KatztookNationalPetroleumStandardsdataandwithsupplementarydataproduced
correction factors for pressure and temperature to convert atmospheric density to
reservoirdensity.
Thecompressibilityandthermalexpansioneffectshavebeenexpressedgraphically
inFigures14and15.

10
9
8
7
6
5
4
3
2
1
0
25 30 35 40 45 50 55 60 65
Density at 60F and 14.7 psia, lb/cu ft
D
e
n
s
i
t
y

o
f

p
r
e
s
s
u
r
e

m
i
n
u
s

d
e
n
s
i
t
y

a
t

6
0


1
4
.
7

p
s
i
a

l
b
/
c
u

f
t
P
r
e
s
s
u
r
e
,

p
s
i
a

1
5
,
0
0
0

1
0
,
0
0
0

8
,
0
0
0

5
,
0
0
0

6
,
0
0
0

4
,
0
0
0

3
,
0
0
0

2
,
0
0
0

1
,
0
0
0

Figure 14 Densitycorrectionforcompressibilityofliquids
8
.
Institute of Petroleum Engineering, Heriot-Watt University

10
9
8
7
6
5
4
3
2
1
0
25 30 35 40 50 45 55 60 65
Density at 60F and pressure P, lb/cu ft
D
e
n
s
i
t
y

a
t

6
0

F

m
i
n
u
s

d
e
n
s
i
t
y

a
t

t
e
m
p
e
r
a
t
u
r
e
,

l
b
/
c
u

f
t
80
100
1
2
0

1
6
0

1
8
0

2
0
0

2
2
0

T
e
m
p
e
r
a
t
u
r
e

F

2
4
0

1
4
0

60
Figure 15 Densitycorrectionforthermalexpansionofliquids
10
.
ExErcIsE .
calculate the density of the reservoir liquid of exercise at a reservoir temperature
of ,00 psia and 10
o
F
Fullcompositionaldatamaynotalwaysbeavailableandthecharacterisationofthe
produced fuids will vary from full compositional analysis to a description of the
fuidsintermsofgasandoilgravity.Theprocedurejustdescribedisforthesitua-
tionwherethecompositionofthereservoirfuidisknown.Theprocedureswhich
followcoverthesituationwherealesscomprehensiveanalysisisavailable.These
methodsmakeuseofempiricalcorrelations.
Properties of Reservoir Liquids

2. Reservoir Density when the Gas Solubility , the gas composition and the surface
oil gravity are known
Byconsideringsurfaceliquidasasinglecomponentandknowingthecomposition
ofthecollectedgasthetechniquespreviouslydiscussedcanbeusedtodetermine
reservoirliquiddensity.Againwewillillustratetheprocedurewithanexample
ExErcIsE .
A reservoir at a pressure of ,000 psia and a temperature of 00
o
F has a producing
gas to oil ratio of 00 scf/stB. the oil produced has a gravity of
o
API. calculate
the density of the reservoir liquid. the produced gas has the following composition
component Mole Fraction
Methane 0.1
Ethane 0.1
Propane 0.0
Butane 0.0
Pentane 0.0
Hextane 0.01
3. The Gas Solubility, and Gas and Liquid gravities are known.
Katzhasproducedacorrelation(fgure16)toenabledensitiestobedeterminedwhen
theonlyinformationonthegasisitssolubilityanditsgravity.Thefguregivesap-
parentliquiddensitiesofgasesagainstgravityfordifferentAPIcrudes

0.6
15
20
25
30
35
40
45
0.7 0.8 0.9 1.0 1.1 1.2 1.3 1.4
Gas Gravity
A
p
p
a
r
e
n
t

L
i
q
u
i
d

d
e
n
s
i
t
y

o
f

D
i
s
s
o
l
v
e
d

G
a
s

a
t
6
0


F

a
n
d

1
4
.
7

p
s
i
a
,

l
b
/
c
u
.

f
t
.
20 API Crude
30
40
50
60
Figure 16 Apparentliquiddensitiesofnaturalgases
Institute of Petroleum Engineering, Heriot-Watt University

ExErcIsE .
Use the correlation of Katz to calculate the reservoir fluid density of a field with
a Gor of 00scf/stB with a gas gravity of 0. and a
o
API oil for reservoir
conditions of ,000psia and a temperature of 10
o
F.
Katz method
9 FORMATION VOLUME FACTOR OF GAS CONDENSATE
Thesituationforawetgasorgascondensateisdifferentforaconventionaloilwhen
oneisconsideringthevolumechangestakingplaceuponreleasetosurfacecondi-
tions.Forawetgasorcondensatesystemliquidatsurfaceisgasintheformation.
Thecomparisonthereforewithrespecttoconditionsinthereservoirtothoseatthe
surfaceisdistinctlydifferentfromanoilsystem,whereanoilinthereservoirproduces
gasandliquidsatthesurface.Forawetgasorcondensate,agasinthereservoir
producesgasandliquidsatthesurface.
The formation-volume factor therefore for a condensate, B
gc
is defned as the
volume of gas in the reservoir required to produce 1.0 STB of condensate at the
surface.Theunitsaregenerallybarrelsofgasatres.conditionsperbarrelofstock
tankoil.ThereareanumberofmethodsofestimatingB
gc
.
Tocalculatethepropertiesofthereservoirfuidfromtheinformationontheproduced
fuidsrequiresacombinationofthe quantitiesandcharacteristicsofthesefuids.The
methodsuseddependsonthelevelofdetailofthecharacteristicsoftheproduced
fuids.Anumberofmethodsarepresentedusingexampleswhichvaryaccording
tothelevelofdetail.
ExErcIsE .
A gas condensate produces gas and liquids with the compositions detailed below,
with a producing Gor of 0,000 scF/stB. Determine the composition of the
reservoir gas.

Component Composition
Gas Liquid
Methane 0.84
Ethane 0.08
Propane 0.04 0.15
Butane 0.03 0.36
Pentane 0.01 0.28
Hexane 0.12
Heptane + 0.09
1.00 1.00
Properties of Reservoir Liquids

ExErcIsE .
the gas condensate reservoir above is contained in reservoir sands with an
average pay thickness of 100ft, with a porosity of 0.1 and a connate water
saturation of 0.1. the aerial extent of the field is sq. miles. the initial reservoir
pressure is ,000 psia and the reservoir temperature is 10
o
F. Determine the initial
reserves of the field in terms of condensate and gas.
ExErcIsE .
calculate the gas condensate formation factor for the example in exercise .
10 VISCOSITY OF OIL
Theviscosityofoilatreservoirtemperatureandpressureislessthantheviscosity
ofthedeadoilbecauseofthedissolvedgasesandthehighertemperature.Correla-
tionsareavailablewhichenablethedissolvedgasandpressureeffectonthedead
oil viscosity to be determined. Danesh
2
has given a good review of many of the
empiricalapproaches.ThefavouredcorrelationsarethoseofBeggsandRobinson
11
,

EgbogahandNg
12
,VazquezandBeggs
13,
andLabedi
14
.Figure17givesplots,
presentedbyMcCain
17
,ofthecorrelationofdeadoilviscosityfromEgbogahand
Ng
12
,andfgure18stheimpactofdissolvedgasfromtheBeggsandRobinson
11
correlation.

R
e
s
e
r
v
o
i
r
T
e
m
p
e
r
a
t
u
r
e
,

F

100
150
200
250
300
1000
800
600
700
500
400
300
200
100
80
60
70
50
40
30
20
10
10 20 30
Stock - Tank Oil Gravity, API
40 50
8
6
7
5
4
3
2
1
0.8
0.6
0.7
0.5
0.4
0.3
0.2
0.1
V
i
s
c
o
s
i
t
y

o
f

G
a
s
-
F
r
e
e

O
i
l
,

o
D
,

c
p
Figure 17 Deadoilviscosities
17
.
Institute of Petroleum Engineering, Heriot-Watt University

0
1
0
0
2
0
0
5
0
0
1
0
0
0
1
5
0
0
2
0
0
0
200
100
80
60
70
50
40
30
20
10
0.4 2 3 4 5 6 78 10 20 30 200 300 40 60 80100 0.6 0.8 1
Viscosity of Gas-Free Oil,
oD
, cp
8
6
7
5
4
3
2
1
0.8
0.6
0.7
0.5
0.4
0.3
0.2
0.1
V
i
s
c
o
s
i
t
y

o
f

G
a
s
-
S
a
t
u
r
a
t
e
d

O
i
l
,

o
D
,

c
p
S
o
l
u
t
i
o
n

G
a
s
-
O
i
l

R
a
t
i
o

Figure 18 Viscositiesofsaturatedblackoils
11
.
BeggsandRobinson
11
examined600oilsamplesoverawiderangeofpressureand
temperatureandcameupwiththefollowingcorrelation.

od
=10
A
-1 (6)
where,logA=3.0324-0.0202
o
API-1.163logT

od
isthedeadoilviscosityincpandTisin
o
F.
EgbogahandNg
12
,hadadifferentexpressionforA
logA=1.8653-0.025086
o
API-0.56441logT
Examinationofthesecorrelationshasshownthattheyarenotveryreliablewith
errorsoftheorderof25%(DeGetto
15
)
BeggsandRobinson
11
gaveacorrelationtogivetheimpactofdissolvedgas.

ob
=C
od
B
(7)
where C =10.715(R
s
+100)
-0.515
and B =5.44(R
s
+150)
-0.338


ob
isthesaturatedoilviscosity
VazquezandBeggs
13
presentedanequationtotakeintoaccountpressureonviscosity
abovethesaturationpressure.
Properties of Reservoir Liquids

o
=
ob
(P/P
b
)
D
(8)
where D =2.6P
1.187
e
-11.513-8.98x10-5P
Thisispresentedinfgure19fromMcCain
17
.

P
r
e
s
s
u
r
e
6
0
0
0
p
s
ia
5
0
0
0
4
0
0
0
3
0
0
0
2
0
0
0
1
0
0
0
5
0
0
100
60
40
20
10
6
4
2
1
0.6
0.4
0.2
0.1
0.1
0.2
2
3
4
5
6
7
8
9
10
20
30
40
50
60
70
80
90
100
200
0.3
0.4
0.5
0.6
0.7
0.8
0.9
1.0
10,000
9,000
8,000
7,000
6,000
5,000
4,000
3,000
2.000
1.000
900
800
700
600
500
400
300
200
B
u
b
b
l
e

P
o
i
n
t

p
r
e
s
s
u
r
e
,

P
b
,

p
s
i
a
V
i
s
c
o
s
i
t
y

o
f

O
i
l

A
b
o
v
e

B
u
b
b
l
e

P
o
i
n
t
,

o
,

c
p
Viscosity of Oil At Bubble Point, cp
Figure 19 Viscositiesofundersaturatedblackoils
17
.
Labedi(ref14)alsoproducedanempiricalcorrelationtodetermineviscosityatpres-
suresabovethebubblepoint

o
=
ob
+(P/P
b
-1)(10
-2.488

ob
0.9036
P
b
0.6151
/10
0.0197oAPI
) (9)
Danesh
2
inhistextcomparedthevariouscorrelationsfromapublishedexperimental
viscosityvalueinawellknownPVTreport,usingthefollowingexercise.
Institute of Petroleum Engineering, Heriot-Watt University

ExErcIsE. 10
calculate the viscosity of oil in the PVt report of chapter 1 at a pressure of
,000psig and 0F. the API of the oil is 0.1 and the Gor, r
s

is scf/st
Beggs and robinson

od
= 10
A
-1
log A = .0 - 0.00API - 1.1 log tx

od
= dead oil viscosity cp.
(Beggs .0 0.00 1.1)
(Egbogah 1. 0.00 0.1)
Beggs Egbolgah
API = 0.1
t = 0
r
s
=
P = ,000 psig
P
b
= , psig
log A = -0.01 -0.
A = 0.10 0.
Viscosity
dead oil = 1.0 cp 1.1 cp
Measured value = 1. cp
Viscosity at bubble point
Beggs

ob
= c
ob
B

ob
= oil viscosity at bubble point pressure
c = 10.1 (r
s
+ 100)
-0.1
B = . (r
s
+ 10)
-0.
c = 0.
B = 0.

ob
= 0. cp
Measured value = 0. cp
Viscosity at pressure of 01 psig
Vazquez - Beggs

o
=
ob
(P/P
b
)
D
D = .p
1.1
e
-11.1 - .x 10
-
p
e function = -11.
D = 0. cp
labed, correlation

o
=
ob
+ (P/P
b
-1)(10
-.

ob
0.0
P
b
0.11
/10
0.01
o
API
)

o
= 0.0 cp
Measured value = 0. cp
Properties of Reservoir Liquids
0

11 INTERFACIAL TENSION
Inrecentyearsinterfacialtensionhasbecometoberealisedasanimportantphysical
propertyinthecontextoftherecoveryofreservoirheldhydrocarbons,inparticular
for gas condensates. Interfacial tension, arises from the imbalance of molecular
forcesattheinterfacebetweentwophases.Formanyyearsithasbeenneglectedbut
morerecentlyithasbeenrealisedthatingasinjectionandcondensationprocesses
the magnitude of the various forces; surface, gravitational and viscous forces can
haveasignifcantimpactonthemobilityofthevariousphases.Amajoradvancein
knowledgehasbeenthatinthecontextofgascondensateswhereitwasconsidered
thatinthetraditionofrelativepermeabilityknowledgeliquidformationbyretrograde
condensation would be immobile. Recent research has shown that such fuids are
mobilebecauseoftheassociatedlowinterfacialtension
16
.Danesh
2
inhistextcovers
thetopicofinterfacialtensionextensively.Mentionedbriefybelowaresomeofthe
techniqueswhicharecurrentlyusedinpredictingITforreservoirfuids.
Interfacialtensiondecreasesastemperatureandpressureincreasesasshownforthe
effectoftemperatureforpurecomponentsinfgure20fromMcCainstext
17
adapted
fromKatz
19
data.

Mol wt.
240
-200 -200 0
0
5
10
15
20
25
30
35
100 200 300 400 500 600
220
200
180
160
140
n - Octane
n - Heptane
n - Hexane
n - Pentane
l - Butane
n - Butane
Propane Ethane Methane
Temperature, F
S
u
r
f
a
c
e

T
e
n
s
i
o
n
,

d
y
n
e
s

p
e
r

c
m
Figure 20Interfacialtensionsofhydrocarbons.(AdaptedfromKatz,etal.,J.Pet.Tech.,
Sept.1943.)
Institute of Petroleum Engineering, Heriot-Watt University
1
ThereareseveralmethodsforpredictingIFT,andtheyrequireexperimentallydetermined
parameters.WorkonpurecompoundshaveshownthatIFTcanberelatedtodensity,
compressibilityandlatentheatofvaporisation.Themulticomponentperspectiveof
reservoirfuidpropertieshasmadeuseoftheIFT/densityrelationships.
TheParachormethodofMcLeod
18
hasgainedacceptancewheretheIFTbetween
vapourandliquidisrelatedtothedensitydifferenceoftherespectivephases.


j
(
,
\
,
(
P
M
L g
4

(10)
where
L
and
g
arethedensityoftheliquidandgasphases,andMisthemolecular
weight.istheIFT.P

iscalledtheparachor.
Katz
19
hasprovidedtheparachorsforpurecomponentsasshowninthetablebelowand
theyarealsopresentedinthefgure21preparedbyMaCainusingKatzs
19
data.

Parachors, Ps, for IFT


Component Parachor
Methane 77
Ethane 108
Propane 150.3
i-Butane 181.5
n-Butane 189.9
i-Pentane 225
n-Pentane 231.5
n-Hexane 271
n-Heptane 312.5
n-Octane 351.5
Hydrogen 34
Nitrogen 41
Carbon dioxide 78
Parachorshavebeenshowntohavealinearrelationshipwithmolecularweightac-
cordingtotherelationship;
P

=21.99+2.892M (11)

andalsotothecriticalproperties
Properties of Reservoir Liquids

600
500
400
300
200
100
0 50 100 150 200
i - C
5
i - C
4
Molecular Weight
P
a
r
a
c
h
o
r
,

P
Figure 21 Parachorsforcomputinginterfacialtensionofnormalparaffnhydrocarbons
19
.
P

=0.324T
c
1/4
v
c
7/8
whereT
c
isinKandthecriticalvolumev
c
isincm
3
/gmol.
ToapplytheparachorapproachtomixturesthemolaraveragingapproachofWeinaug
andKatz
20
canbeused.


j
(
,
\
,
(
,

,
,
]
]
]
]

P x
M
yj
M
j
L
L
g
g j
4

(12)
x
j
andy
j
arethemolefractionsofthecomponentsintheliquidandgasphases.
Firoozabadi
21
hasprovidedparachorstoenablecalculationstobemadeforheavy
componentsusingthefollowingequation.
P
s
=-11.4+3.23M-0.0022M
2
(13)
whereMisthemolecularweightoftheheavycomponent.Figure22.
Institute of Petroleum Engineering, Heriot-Watt University

Molecular weight
P
a
r
a
c
h
o
r
.

P
1400
1200
1000
800
600
400
200
0
100 200 300 400 500
Figure 22 Parachorsofheavyfractionsforcomputinginterfacialtensionofreservoir
liquids.McCain
17
ThismethodisillustratedusinganexamplefromMcCain
17
.
ExErcIsE 11.


calculate the IFt of the following volatile oil mixture at 1 psia and 10F for
the oil with the following composition.
12 COMPARISON OF RESERVOIR FLUID MODELS
It is useful to summarise the differences between the Black Oil Model approach
comparedtotheCompositionalModelinpredictedfuidproperties.
Thesuitabilityofthetwoapproachesislargelyrelatedtothenatureofthefuid.For
heavieroilswheretherearelowGORsascomparedtovolatileoilswithhighGORs,
blackoilmodelsarelikelytobesuitable.Forthemorevolatilesystemswherethere
aremoresignifcantcompositionalvariationsinareservoiraspressureisdepleted,
compositionalmodelsareconsideredmorecapableofpredictingfuidbehaviour.
Thecomputationalrequirementsofcompositionalmodelsusedtobearestrictionwhen
carryingoutlargereservoirsimulation.Thecontinueddevelopmentofcomputing
andassociatedequationsofstatemodellingreducestheseformerrestrictions.
Properties of Reservoir Liquids

Companiesarenowfocusingtheirattentiononbeingcapableofmodellingthetotal
processfromfuidextractionfromthereservoir,throughwellproductionandfacil-
itytreatment.Atpresentseparatemodellingoccurs,andmanyofthesemodelsare
notcompatible.Theblackoilapproachiscertainlyconsideredbymanytobefrom
aformerera.
Thelistbelowgivesasummarycomparisonofthetwoapproaches.
Black Oil Models
2components-solutiongasandstocktankoil
Bo,Rg,etc.
Empiricalcorrelations
Aftertheeventdescriptionoffuidproperties
Compositional Models
Ncomponentsbasedonparaffnseries
Equationofstatebasedcalculations
Feedforwardcalculationoffuidproperties
Inasubsequentchapteronvapourliquidequilibriawewilldescribehowthevolumes
andcompositionsofvapourandliquidequilibriummixturescanbecalculatedwhen
amixtureisprocessedataparticularpressureandtemperature.Thesecalculations
althoughcalculationintensivecanbeconsideredfeedforwardcalculationsanden-
abletheeffectsoftemperatureandpressurechangestobedeterminedonaparticular
feedmixture.
Theblackoilapproachwhichhasbeenamajorthemeofthischapterusesthechar-
acteristicsoftheproducedfuidstodeterminethecompositionandpropertiesofthe
feedreservoirmixture,i.e.abackcalculation.AswillbeseeninthesectiononPVT
reports,thequantitiesandcharacteristicsoftheproducedfuidsaredependantonthe
pressureandtemperatureconditionsusedtoseparatethefuid.
Atthebackofthischapteraretablesofphysicalpropertieswhichareusefulinmany
oftheproceduresdescribed.
Institute of Petroleum Engineering, Heriot-Watt University

Properties of Reservoir Liquids

Institute of Petroleum Engineering, Heriot-Watt University

Properties of Reservoir Liquids

Institute of Petroleum Engineering, Heriot-Watt University

Solutions to Exercises
ExErcISE 1.
Calculatethedensityat14.7psiaand60Fofthehydrocarbonliquidmixturewith
thecompositiongivenbelow:

Component Mol.
fract.
1b mol.

nC4 0.25
nC5 0.32
nC6 0.43
1.00
SoLutIon ExErcISE 1

Solution
Component Mol. Mol. Weight Liquid Liquid
density
fract. weight 1b Density at volume
1b mol. 1b/1b at 60F and 14.7 cu ft
mol. psia
1b/cu ft
nC4 0.25 58.1 14.525 36.45 0.3985
nC5 0.32 72.2 23.104 39.36 0.5870
nC6 0.43 86.2 37.066 41.43 0.8947
____ _____ _____
1 74.695 1.8801
ExErcISE 2:
Calculate the surface pseudo liquid density of the following reservoir
composition.

Component Mole percent



Methane 44.04
Ethane 4.32 Properties of
Propane 4.05 heptane +
Butane 2.84 API gravities = 34.2
Pentane 1.74 SG = 0.854
Hexane 2.9 Mol wt = 164
Heptane + 40.11
Properties of Reservoir Liquids
0
SoLutIon ExErcISE 2


Estimate

44.65 lb/cu ft. 0.716 gm/cc lb/cuft


From fig 12 Density 0.326 20.3424
C1
Density 0.47 29.328
C2

Component Mole Mol Weight Liq Liquid
fraction Weight Density Volume
lb/lb lb at 60F &
mole 14.7 psia
lb/cu.ft cu ft.
z M zM o zM/o
Methane 0.4404 16 7.0464 20.3424 0.34639
Ethane 0.0432 30.1 1.30032 29.328 0.04434
Propane 0.0405 44.1 1.78605 31.66 0.05641
Butane 0.0284 58.1 1.65004 35.78 0.04612
Pentane(n&i) 0.0174 72.2 1.25628 38.51 0.03262
Hexane(n&i) 0.029 86.2 2.4998 41.43 0.06034
Heptane+ 0.4011 164 65.7804 53.26 1.23508
Total 1 81.31929 1.8213

Density = 81.32 lb / 1.82 cu ft
= 44.65 lb/cu.ft
ExErcISE 3.
Calculatethesurfacedensityofthemixtureinexercise2usingthechartoffgure
13
SoLutIon ExErcISE 3


Component Mole Mol Weight Liq Liquid
fraction Weight Density Volume
lb/lb lb at 60F &
mole 14.7 psia
lb/cu.ft cu ft.
z M zM
o
zM/
o
Methane 0.4404 16 7.0464
Ethane 0.0432 30.1 1.30032
Propane 0.0405 44.1 1.78605 31.66 0.05641
Butane 0.0284 58.1 1.65004 35.78 0.04612
Pentane(n&i) 0.0174 72.2 1.25628 38.51 0.03262
Hexane(n&i) 0.029 86.2 2.4998 41.43 0.06034
Heptane+ 0.4011 164 65.7804 53.26 1.23508
1
Weight of propane + 72.97
lbs. = Volume = 1.43
Density of propane + = 51.01 lb cu ft
Weight per cent ethane in
ethane + 1.75
Weight per cent methane in 8.67
methane +
From figure 13 pseudo liquid
density = 45 lb/cu ft

Institute of Petroleum Engineering, Heriot-Watt University
1
ExErcISE 4.
Calculatethedensityofthereservoirliquidofexercise3atareservoirtemperature
of5,500psiaand180
o
F
SoLutIon ExErcISE 4
Densityoffollowingreservoirliquidat6,000psiaand180F.
Step 1
Pseudoliquiddensityatstandardconditions
fromexercise3
o
=45lb/cuft
Step 2
Adjustto60Fand5,500psia
i.e.correction=+1.9lb/cuft (Figure14)
i.e.
o
=45+1.9=46.9lb/cuftat60F6,000psi
Step 3
Adjustto180F. (Figure15)
i.e.thermalcorrection=-3.18lb/cuft

o
=46.9-3.18=42.32lb/cuftat180and6,000psia

o
=42.32lb/cuft@180Fand6,000psia
ExErcISE 5.
Areservoiratapressureof4,000psiaandatemperatureof200
o
Fhasaproducinggas
tooilratioof600scf/STB.Theoilproducedhasagravityof42
o
API.Calculatethe
densityofthereservoirliquid.Theproducedgashasthefollowingcomposition
Component MoleFraction
Methane 0.71
Ethane 0.13
Propane 0.08
Butane 0.05
Pentane 0.02
Hextane 0.01
Properties of Reservoir Liquids


Calculation of pseudo density of gas.
From PV=znRT, Solubility of gas, Rs = 600 scf/STB
1 lb mole = 379 scf
Oil = 42 API
Density of crude = 50.87 lb/cuft 285.62 lb/STB
Density of water = 62.37 lb./cuft

Component Mole Solubility Mol Weight Liq Density Liquid Volume
fraction Weight
volume scf lb/lb mole lb/STB at 60F
fraction gas/STB & 14.7 psia
lb/cu.ft cu ft/STB.
z zRs M zRsM/379
o
zm/
o

Methane 0.71 426 16 17.98
Ethane 0.13 78 30.1 6.19
Propane 0.08 48 44.1 5.59 31.66 0.176
Butane 0.05 30 58.1 4.60 35.78 0.129
Pentane(n&i) 0.02 12 72.2 2.29 38.51 0.059
Hexane(n&i) 0.01 6 86.2 1.36 41.43 0.033
Oil 42 API 285.62 5.615
Totals 600 323.63 lb 6.01 cu ft

Density of propane + = 323/6.01/lb cuft = 49.81 lb/ cu ft
Weight % C2+ = 2.315
Weight% C1+ = 5.557
From Figure 13 Pseudoliquid density of reservoir fluid at 60F & 14.7 psia = 46.5 lb / cu ft

Correction for pressure Fig 14 = 1.23 + = 47.73
Correction for temperature Fig 15 3.55 - = 44.18

Density of Reservoir Fluid = 44.18 lb/cu ft
ExErcISE 6.
UsethecorrelationofKatztocalculatethereservoirfuiddensityofafeldwitha
GORof500scf/STBwithagasgravityof0.8anda35
o
APIoilforreservoircondi-
tionsof4,000psiaandatemperatureof180
o
F.
Katzmethod
SoLutIon ExErcISE 6.
MassofgasperSTB.
Molecularweightofgas=molecularweightairx0.8=29.2x0.8=23.2
Mas og gas STB
scf
stb
x
lb mole
scf
x
lb
lb mole
lb STB /
.

. / 500
379
23 2
30 61
Institute of Petroleum Engineering, Heriot-Watt University

Component Weight Liq Density Liquid Volume


lb/STB at 60F cu ft/STB.
& 14.7 psia
lb/cu.ft
Gas 30.61 26.3 1.164
Oil 297.62 from chart 5.615
328.23 6.779
Pseudodensity
of reservoir fluid= 328.23 / 6.779 = 48.42
Correction for pressure at Fig 14 +1.13 = 49.55

Correction for pressure at Fig 15 -2.9 = 46.65

Reservoir density= 46.65 lb/cu ft
ExErcISE 7.
Agascondensateproducesgasandliquidswiththecompositionsdetailedbelow,
with a producing GOR of 30,000 SCF/STB. Determine the composition of the
reservoirgas.

Component Composition
Gas Liquid
Methane 0.84
Ethane 0.08
Propane 0.04 0.15
Butane 0.03 0.36
Pentane 0.01 0.28
Hexane 0.12
Heptane + 0.09
1.00 1.00
Properties of Reservoir Liquids

SoLutIon ExErcISE 7

Liquid
Component Mol. Fractn Mol.Wgt. Wgt. Liquid Liquid
lb mole lb/lb mol lb/lb mole density volume
lb/cu ft cu ft
C3 0.15 44.1 6.615 31.66 0.223
C4 0.36 58.1 20.916 35.78 0.585
C5 0.28 72.2 20.216 38.51 0.506
C6 0.12 86.2 10.344 41.3 0.25
C7+* 0.09 114.2 10.278 43.68 0.235
* C8 used for C7+ 68.369 1.799
Mol.Wgt. 68.369
liq.
Density of liquid= 38.00 lb/cu ft
GOR= 30000 scf/STB 213.39 lb/STB
= 79.16 lb mole gas/STB 3.12 lb mole /STB
Note: 1 lb mole = 379 SCF
GOR = 25.36 lb mole gas/lb mole liquid

2. Recombination according to the above GOR of 25.36 lb mole gas / lb moleliquid

Component Composition lb mole gas/ lb moles Composition
Gas Liquid lb mole oil Res fluid Res Fluid
lb mole lb mole 25.36
y x 25.36y 25.36y + x
Methane 0.84 21.30 21.30 0.808
Ethane 0.08 2.03 2.03 0.077
Propane 0.04 0.15 1.01 1.16 0.044
Butane 0.03 0.36 0.76 1.12 0.043
Pentane 0.01 0.28 0.25 0.53 0.020
Hexane 0.12 0.12 0.005
Heptane + 0.09 0.09 0.003
1 1 25.36 26.36 1.000

ExErcISE 8.
Thegascondensatereservoiraboveiscontainedinreservoirsandswithanaverage
paythicknessof100ft,withaporosityof0.18andaconnatewatersaturationof0.16.
Theaerialextentofthefeldis5sq.miles.Theinitialreservoirpressureis5,000psia
andthereservoirtemperatureis180
o
F.Determinetheinitialreservesofthefeldin
termsofcondensateandgas.
Institute of Petroleum Engineering, Heriot-Watt University

SoLutIon ExErcISE 8


Component Mol. Fract. Critical Temperature Critical Pressure
R R psia psia
lb mole y
j
T
cj
y
j
T
cj
P
cj
y
j
P
cj
C1 0.808 344 278.00 667 539.026
C2 0.077 551 42.41 708 54.491
C3 0.044 666 29.42 616 27.210
C4 0.043 750 31.89 540 22.960
C5 0.020 838 16.96 489 9.899
C6 0.005 914 4.16 437 1.989
C7+ 0.003 1025 3.50 360 1.229
Totals 1 406.34 656.80
T
pc
= 406.34 P
pc
= 656.80

Reservoir pressure = 5000 psia
Reservoir temperature = 180 F = 640 R

Pseudo reduced pressure = 7.61
Pseudo reduced temperature = 1.58
Compressibility factor from Standing & Katz chart figure 2 Gas properties chapter
z= 0.98
R= 10.73 cu ft. psi/lb.mol R
Volume of the reservoir = 5 square miles x 100 feet (1 mole = 5280 ft)
Volume of the reservoir = 2.1076 x 10
9
cu ft
PV=znRT
V/n=zRT/P
Specific volume at reservoir conditions = 1.3460 cu ft/lb.mol
No of lb moles in reservoir= 1.5658 x 10
9
lb moles
No. of standard cubic feet of gas in reservoir = 5.9345 x 10
11
SCF (1 lb mole 379 scf)
Reserves in reservoir in terms of produced fluids
From previous exercise GOR of = 30,000 SCF/STB
= 25.36 lb mole gas/lb mole condensate
For each 26.36 lb mole of reservoir fluid 25.36 lb mol is produced gas
and 1 lb mole is condensate
Reserves in terms of produced fluids
Gas 1.506428 x 10
9
lb moles = 5.70936 x 10
11
SCF
Condensate 1.9643E+09 lb moles = 6.2935E+08 STB
ExErcISE 9.
Calculatethegascondensateformationfactorfortheexampleinexercise8.
SoLutIon ExErcISE 9.
B
gc
=bblsofgasinreservoir/STBcondensate
Volumeofgasinreservoir=6.9696x10
10
cuft=1.2412x10
10
bbls
Condensate=6.2935x10
6
STB
B
gc
=1972.2 bblsresgas/STBcondensate

Insomecasesfullcompositionalinformationmaynotbeavailablebutonlyblack
oildescriptionsoftheoilandgasgravityforthegas.Inthiscasecorrelationscanbe
usedtoprovidethenecessarydatatocalculatethesamedataasforexercise8&9.
Properties of Reservoir Liquids

ExErcISE 10
CalculatetheviscosityofoilinthePVTreportofchapter12atapressureof5,000psig
and220F.TheAPIoftheoilis40.1andtheGOR,R
s

is795scf/ST
Beggs and robinson

od
=10
A
-1
LogA=3.0324-0.0202API-1.163logTx

od
=deadoilviscositycp.
(Beggs3.03240.02021.163)
(Egbogah1.86530.0250860.56441)
Beggs Egbolgah
API= 40.1
T= 220
R
s
= 795
P= 5,000psig
P
b
= 2,635psig
logA=-0.5031-0.46
A= 0.3140 0.34
Viscosity
deadoil= 1.06cp 1.21cp
Measuredvalue=1.29cp
Viscosityatbubblepoint
Beggs

ob
=Cm
ob
B

ob
=oilviscosityatbubblepointpressure
C=10.715(R
s
+100)
-0.515
B=5.44(R
s
+150)
-0.338
C=0.3234
B=0.5369

ob
=0.3584cp
Measuredvalue=0.355cp
Viscosity at pressure of 01 psig
Vazquez - Beggs

o
=
ob
(P/P
b
)
D
D = .p
1.1
e
-11.1 - .x 10
-
p
e function = -11.
D = 0. cp
labed, correlation

o
=
ob
+ (P/P
b
-1)(10
-.

ob
0.0
P
b
0.11
/10
0.01
o
API
)

o
= 0.0 cp
Measuredvalue=0.45cp
Institute of Petroleum Engineering, Heriot-Watt University

ExErcISE 11

CalculatetheIFTofthefollowingvolatileoilmixtureat2315psiaand190Ffor
theoilwiththefollowingcomposition.
SoLutIon ExErcISE 11

Component Liquid Composition Gas Composition


Mole fraction Mole fraction
Carbon dioxide 0.0159 0.0259
Nitrogen 0.0000 0.0022
Methane 0.3428 0.8050
Ethane 0.0752 0.0910
Propane 0.0564 0.0402
i - Butane 0.0097 0.0059
n - Butane 0.0249 0.0126
i - Pentane 0.0110 0.0039
n - Pentane 0.0140 0.0044
Hexane 0.0197 0.0040
Heptanes plus 0.4303 0.0049
Propertiesofheptanesplusofliquid
Specifcgravity=0.868
Molecularweight=217lb/lbmole
Densityofliquidsandgasfrompreviousmethods
P
L
=0.719g/cc
P
g
=0.137g/cc
Molecularweight M
L
=110.1g/smole
M
g
=21.1g/gmole

Component x
j
y
i
P

Equation 12
Co
2
0.0159 0.0259 78.0 -0.0050
N
2
0.0000 0.0022 41.0 -0.0006
C
1
0.3428 0.8050 77.0 -0.2301
C
2
0.0752 0.0910 108.0 -0.0108
C
3
0.0564 0.0402 150.3 0.0161
i-C
4
0.0097 0.0059 181.5 0.0046
n-C
4
0.0249 0.0126 189.9 0.0154
i-C
5
0.0110 0.0039 225.0 0.0105
i-C
5
0.0141 0.0044 231.5 0.0147
C
6
0.0197 0.0040 271.0 0.0278
C
7+
* 0.4303 0.0049 *586.6 1.6297
1.000 1.000 1.4723
fromfgure23
Properties of Reservoir Liquids

REFERENCES
1.Craft,BC&Hawkins,MF.AppliedReservoirEngineering1959PrenticeHall,
NY
2.Danesh,A,PVT and Phase Behaviour of Petroleum Reservoir Fluids. 1998
Elsevier.pp66-77
3.Standing MB A pressure-Volume-Temperature Correlation for Mixtures of
CalifornianOilsandGases,Drill&Prod,Proc.275-287(1947)
4.Lasater,J.A.BubblePointCorrelation,TransAIME,213,379-381(1958).
5.Vasquez,MandBeggs,HDCorrelationsforFluidPhysicalPropertyPrediction
JPT,968-970,(June1980)
6.Glaso, O Generalised Pressure Volume Temperature Correlations JPT,785
795(May1980)
7.Marhoun,MA,PVTCorrelationsforMiddleEastCrudeOilsJPT,650-665
(May1988)
8.Standing,M.B.andKatz,D.L.DensityofCrudeOilsSaturatedwithNatural
GasTransAIME146,159(1942)
9.Kessler,MGandLee,BI,:ImprovedPredictionofEnthalpyofFractions,Hyd
Proc.(Mar.1976)55,153-158.
10.Standing,MVolumetricandPhaseBehaviourofOilFieldHydrocarbonSystems
SPEDallas1951
11.Beggs,HD.andRobinson,JR:EstimatingtheViscosityofCrudeOilSystems
JPT,27,1140-1141(1975)
12.Egboghah,EO and Ng,JT: An improvedTemperatureViscosity Correlations
forCrudeOilSystems,J.PetSciandEng.,5,197-200(1990)
13.Vasquez,M.andBeggs,HD:CorrelationsforFluidPhysicalPropertyPredictions.
JPT,968(June1980)
14.Labedi,R:UseofProductionDatatoEstimateVolumeFactor,Densityand
CompressibilityofReservoirFluids,J.ofPet.SciandEng.4.375-90,(1990)
15.DeGhetto,G.,Paone,F.andVilla,M.:ReliabilityAnalysisofPVTCorrelations
,SPE28904,ProcofEuro.PetConf.Lndn,375-393(Oct.,1994)
16.Danesh,A.,Krinis,D.,HendersonG.D.,andPeden,J>M>VisualInvestigation
of Retrograde Phenomena and Gas Condensate Flow in Porous Media 5th
EuropeanSymposiumonImprovedOilRecovery,Budapest(1988)
17.McCain,WD., The Properties of Petroleum Fluids Pennwell Books ,Tulsa,
Ok1990.ISBN0-87814-335-1
18.Macleod,DB.,OnaRelationBetweenSurfaceTensionandDensity,Trans.,
FaradaySoc.(1923)19,38-42.
19. Katz,DL.,Handbook of Natural Gas Engineering, McGraw Hill Book Co
Inc.,NewYk,(1959)
20.Weinaug,KGandKatz,DL,:SurfaceTensionofMethane-PropaneMixtures.
I&EC,239-246(1943)
21.Firoozabadi,A,Katz,D.L.,Soroosh,H.MandSajjadian,V.A.:SurfaceTension
of Reservoir Crude-Oil/Gas Systems Recognising theAsphalt in the Heavy
Fraction,SPEResEng.(Feb)1988,3,No1,265-272.
CONTENTS
INTRODUCTION
1. CHARACTERISTICS OF RESERVOIR ROCKS
2. PHYSICAL CHARACTERISTICS OF
RESERVOIR ROCKS
3. POROSITY
3.1 Range of Values
3.2 Factors Which Affect Porosity
3.2.1 Packing and Size of Grains
3.2.2 Particle Size Distribution
3.2.3 Particle Shape
3.2.4 Cement Material
3.3 Subsurface Measurement of Porosity
3.3.1 Density Log
3.3.2 Sonic Log
3.3.3 Neutron Log
3.4 Average Porosity
4. PERMEABILITY
4.1 Darcy's Law
4.2 Factors Affecting Permeability
4.3 Generalised Form of Darcy's Law
4.4 Dimensions of Permeability
4.5 Assumptions For Use of Darcy's Law
4.6 Applications of Darcy's Law
4.7 Field Units
4.8 Klinkenberg Effect
4.9 Reactive Fluids
4.10 Average Reservoir Permeability
5. STRESS EFFECTS ON CORE
MEASUREMENTS
5.1 Stress Regimes
5.2 Compressibility of Poros Rock
5.3 Types of Compressiblilty
5.4 Measurements of Pore Volume
Compressiblity
5.5 Effect of Stress on Permeability
6. POROSITY - PERMEABILITY
RELATIONSHIPS
7. SURFACE KINETICS
7.1 Capillary Pressure Theory
7.2 Fluid Distribution in Reservoir Rocks
7.3 Impact of Layered Reservoirs
8. EFFECTIVE PERMEABILITY
8.1 Defnition
8.2 Water Displacement of Oil
8.2.1 Water - Oil Relative Permeability
8.3 Gas Displacement of Oil and Gas - Oil
Relative Permeability
fundamental Properties of Reservoir Rocks

LEARNING OBJECTIVES
Having worked through this chapter the Student will be able to:
Defneporosityandexpressitasanequationintermsofpore,bulkandgrain
volume.
Explainthedifferencebetweentotalandeffectiveporosity.
Defnepermeabilityandpresentanequation,DarcysLaw,relatingfowrateto
permeability in porous media.
ListtheassumptionsfortheapplicabilityofDarcysLaw.
DeriveanequationbasedonDarcysLawrelatingfowofgasinacoreplug
and the upstream and downstream pressures.
Deriveanequationrelatingfowratetopermeabilityforaradialincompressible
system.
Commentonthedifferencebetweengasandliquidpermeability(Klinkenberg
effect ).
Sketchafgurerelatingliquidpermeabilitytogaspermeabilitiesplottedasa
function of reciprocal mean pressure.
Briefydescribetheimpactofreservoirstressesonpermeabilityandporosity
Drawasketchdemonstratingtheresultofinterfacialtensionbetweenoil,water
andasolid,andlocatethecontactangleanddefneitsvaluesforwettingand
non-wetting phases.
ExpressthecapillarypressurePcastwoequations,oneintermsofinterfacial
tension, contact angle and pore radius, and the other in terms of height and
densityoffuids.
Defnethefreewaterlevel.
Draw the Pc or height vs. saturation capillary pressure curve and identify
signifcantfeatures.
Sketch and explain the impact of saturation, history, density difference and
interfacial tension in capillary pressure curves.
Sketch the impact of capillary pressure effects on the saturation distribution of
stratifedformations
Defneeffectiveandrelativepermeabilityandplottypicalshapes.
Defneimbibitionanddrainageinthecontextofcapillarypressureandrelative
permeability curves.
Sketchtheporedoubletmodelanduseittoexplaintheretentionoftrappedoil
inlargepores and briefyrelateittothe principlebehindsomeenhancedoil
recovery methods.
Defnemobilityratio.
Sketch a shape for gas- oil relative permeability curves.
Institute of Petroleum Engineering, Heriot-Watt University

fundamental Properties of Reservoir Rocks


INTROduCTION
Thepropertiesofreservoirrockswithrespecttothefuidstheycontainandwithrespect
tothefuidswhichwillbeinjectedintothemareimportantwhencharacterisinga
reservoirintermsofitsreservesandthemobilityofthefuids.Thisnextsectiongives
abriefoverviewoftheseproperties,andisfollowedbychaptersontheirmeasurement
and variation. In relation to the detailed description of rock characteristics the reader
is referred to the Geology module of this Petroleum Engineering course.
Thereservoirengineerisconcernedwiththequantitiesoffuidscontainedwithinthe
rocks,thetransmissivityoffuidsthroughtherockandotherrelatedproperties.

1. ChARACTERISTICS Of RESERVOIR ROCkS
The specifcations of a reservoir rock are such that there must be a large enough
capacity to store economically viable amounts of hydrocarbon and the hydrocarbon
mustfowateconomicalrateswhenpenetratedbyawell.Thefactorswhichmay
affectthecapacityandthefowpropertiesaretheporosity,permeability,capillary
pressure,compressibilityandfuidsaturation.Inthecaseofareservoirrock,theseare
notstandardcharacteristicsdeterminedbeforeformationoftherock,butareclosely
linked to the geological processes that brought the sediments together and deposited
theminthesequencesandunderthechemicalandphysicalchangesinherentinthe
system.
In order to contain enough oil or gas to make production economically viable, a
reservoirrockmustexceed:aminimumporosity,aminimumthickness,aminimum
permeability,andaminimumarea.
Inordertoextractthefuidstherockmustbepermeablewhichrequiresthattherebe
suffcientlylarge,interconnectingpores.
Although a permeable rock must also be porous, a porous rock is not necessarily
permeable. Certain volcanic rocks are porous but not permeable because the voids are
notinterconnecting;shalemaybequiteporousbutimpermeablebecausetheporesare
extremelysmall,therebypreventingfreemovementofthefuidscontainedwithin.
2. PhySICAL ChARACTERISTICS Of RESERVOIR ROCkS
Considering a common reservoir rock, sandstone, the grains making up this rock
areallirregularinshape.Thedegreeofirregularity,orlackofroundnessrefects
the source sediments and the physical and chemical processes to which they were
subsequently exposed. Violent crushing or grinding action between rocks causes
grains to be very irregular and sharp-edged. The tumbling action of grains along the
bottomofstreamsorseabedssmoothessandgrains.Wind-blownsand,asoccursin
movingdunesindeserts,resultsinsandgrainsthatareevenmorerounded.Sand
grains that make up sandstone beds and fragments of carbonate materials that make
uplimestonebedsdonotfttogethercongruently:thevoidspacebetweenthegrains
forms the porosity.

The pore spaces (or interstices) in reservoir rock provide the container for the
accumulation of oil and gas and these give the rock its characteristic ability to absorb
and hold fuids. Most commercial reservoirs of oil and gas occur in sandstone,
limestoneordolomiterocks,however,somereservoirsoccurinfracturedshaleand
even in basement rocks such as in Vietnam. Knowledge of the physical characteristics
oftheporespaceandoftherockitself(whichcontrolsthecharacteristicsofthepore
space) is of vital importance in understanding the nature of a given reservoir.
Forthereservoirengineer,porosity is one of the most important rock properties as
ameasureofthespaceavailableforaccumulationofhydrocarbonfuids.
3. POROSITy
Thefrststepinformingasandstone,forexample,istohaveasourceofmaterial
which is eroded and transported to low lying depressions and basins such as would
befoundoffthecoastsofalandmass.Thematerialwouldconsistofamixtureof
minerals,butforasandstone,themajoritywouldbemadeofquartzintheformof
grains.Whentheseweredeposited,theywouldbesurroundedbyseawaterorbrine,
andasthesedimentthicknessincreased,theweightorthepressureproducedbythe
overlying sediments would force the grains together. Where they contacted each other
large stresses would be produced and a phenomenon called pressure solution would
occurwhichdissolvedthequartzatthepointsofcontactbetweenthegrainsuntilthe
stresses reduced to a level which was sustainable by the grains. The dissolved material
would be free to precipitate in other regions of the sediment. In this way the initially
loosematerialwouldbesolidifedwithdiscreteconnectionsbetweenthegrains.
Initially,ifsubsea,theporespaceswouldbeflledwithbrine,andasthelithifcation
processoccurred,someporespaceswouldbeisolatedwiththebrinetrappedinside.
Ifthevastmajoritywereinterconnectedthentheinitialporefuidwouldbefreetobe
sweptthroughtherockbyotherfuidssuchashydrocarbons.Inthiswaythegeometry
of the grains produces an assembly of solids with voids in between them. The grains
vary in diameter but may be from a few microns to several hundred microns. The
geometryoftheporespacesissuchthattheyhavenarrowentrances(porethroats)
wheretheedgesofthegrainstoucheachotherandlargerinternaldimensions(between
the grains). The complicated nature of these interconnected pores is illustrated in
fgure1whichisametalcastoftheporesinasandstonerock.
Institute of Petroleum Engineering, Heriot-Watt University

fundamental Properties of Reservoir Rocks



Figure 1 Metallic Cast of Pore Spaces in a Consolidated Sand
Onemethodofclassifyingreservoirrocks,therefore,isbasedonwhetherporespaces
(inwhichtheoilandgasisfound)originatedwhentheformationwaslaiddown
or whether they were formed through subsequent earth stresses or ground water
action.
The frst type of porosity is termed original porosity and the latter, secondary or
inducedporosity.Thisisillustratedinfgure2.

Cementing material
Sand grain
Effective porosity 25%
Isolated porosity 5%
Total porosity 30%
Figure 2 Effective,isolatedandtotalporosity
Secondary porosity in limestone beds occurred as a result of fracturing, jointing,
dissolution,recrystallisationoracombinationoftheseprocesses.
Wherewaterispresentinacarbonateformation,thereisacontinuousprocessof
solution and deposition or recrystallization. If solution is greater than deposition in
anyzone,porositywillbedevelopedbetweenthecrystalgrains.Animportanttype

ofporosityofthiskindisfoundindolomitezoneswhichoccurinconjunctionwith
large limestone deposits. Dolomite may be deposited originally as a sedimentary
rock,oritmaybeformedbyreplacingthecalciumcarbonateinlimestonerockwith
magnesium.
The impact of isolated pore space clearly cannot contribute to recoverable reserves
offuidnorcontributetopermeableporespaceasillustratedinfgure3.

Total Pore Space
dead End
Pore
Isolated Pore Space
Effective Pore Space
Permeable Pore Space
Figure 3 Total,effective,isolatedpermeableanddeadendporespace
Porosityisdefnedastheratioofthevoidspaceorporespace(V
p
) in a rock to the
bulkvolume(V
b
)ofthatrockanditisnormallyexpressedasapercentageoftotal
rock volume. The porosity is usually given the symbol .Thematrixvolumeisthe
volumeofthesolidgrains,V
m
.

Porosity
Void volume
Bulk volume
Porosity
B volume
Bulk volume

+

x
ulk Grain volume
x
Porosity
pore volume
void volume grain volume
100
100
100

Institute of Petroleum Engineering, Heriot-Watt University

fundamental Properties of Reservoir Rocks



Bulk Volume
Representation
Grain Volume
Representation
Pore Volume
Representation
Figure 4 Representationofbulk,grainandporevolumes
Thesecomponentsareillustratedinfgure4formonosizespheres.
Total porosityisdefnedastheratioofthevolumesofalltheporestothebulkofa
material,regardlessofwhetherornotalloftheporesareinterconnected.Effective
porosityisdefnedastheratiooftheinterconnectedporevolumeofamaterial.
Ifthegrainsarerepresentedbyspheresstackedtogetherasinfgure4,thenthepore
space can be seen between the solid grains.

Total Porosity
Total Void Space
V
Effective porosity
Interconnected Void Space
V
b
b



InducedorSecondaryPorosity=porosityfromfracturesorvugs(largechambersformed
incertaincarbonatesandlimestonescausedbygroundwaterfowanddissolution).
3.1 Range Of Values
Themaximumporosityofporousmediacanbeconsideredinrelationtoanassembly
of spheres arranged as a cubic packing of spheres. If the sides of a cube are assumed
tobeformedbythelinesdrawnfromthecentreofeachspheretotheadjacentspheres,
thecubeinfgure5wouldbeproduced.


Figure 5 Cubedefnedbythecentresofeachadjacentsphere
Thelengthofeachsidewouldbe2xradius,givingthebulkvolumeas:
V
b
=(2r)
3
= 8r
3
Thegrainvolumewouldbetheequivalentofthevolumeofonesphere

V
r
m

4
3
3

andtheporosity(giventhesymbol) would be

V V
V
r
r
r
b m
b
8
4
3
8 6
3
3
3
1 - = 0.476
Ifthespheresftinthecuspsgeneratedbythelowerlayerthenaporosityof26%
occurs. For a size distribution of spheres the ultimate minimum porosity would be
zerowhichwouldbethecaseifsuffcientgrainswereavailabletocompletelyfllthe
porespacesasshowninfgure6forpartfllingofthevoid.

Figure 6 Minimumporositywhenallporespacesareflled

Institute of Petroleum Engineering, Heriot-Watt University

fundamental Properties of Reservoir Rocks


Severalfactorsmaycombinetoaffecttheporosityofarock,butthemaindistinction
tobemadeisasfollowsbasedontheamountofconnectedporevolume,andwhether
the pore space has been altered by dissolution or by fracturing after deposition and
lithifcation.
3.2 Factors Which Affect Porosity
The porosity (and permeability) of sandstone depend upon many factors, among
which are the packing, size and shape of the grains, variations in size of grains,
arrangement in which grains were laid down and compacted, and amount of clay
and other materials which cement the sand grains together.
3.2.1 Packing And Size Of Grains
The absolute sizes of the sand grains which make up a rock do not infuence the
amount of porosity occurring in the rock. However variations in the range of sand
grainssizesdoinfuenceconsiderablytheporosity.

3.2.2 Particle Size Distribution
Ifspheresofvaryingsizesarepackedtogether,porositymaybeanyamountfrom48
percenttoaverysmallamountapproaching0percentasshowninfgure7.
3.2.3 Particle Shape
Ifthesandgrainsareelongatedorfatandarepackedwiththeirfatsurfacestogether,
porosityandpermeabilitymaybothbelowwewilldiscussfurtherinthecontextof
permeability.

Pore Space
Figure 7 Reduction in porosity due to a range of particle sizes
3.2.4 Cement Material
Sandstones are compacted and usually cemented together with clays and minerals. The
porosityandpermeabilityofasandstonearebothinfuencedtoamarkeddegreeby
the amount of cementing material present in the pore space and the way this material
occupies the pore space between the sand grains. The cementing material may be
uniformly located along the pore channels to reduce both porosity and permeability
10
or the cementing material may be located at the pore throats which reduces the ability
offuidtoenterthepore,butmaynotreducetheoverallporosityoftherockbya
signifcantamount.
Limestoneformationsmayhaveintergranularporosity.However,theporeopenings
aremoreofteninter-crystalline,thatisspacesbetweenmicroscopiccrystals.Theyalso
maytaketheformofpitsorvugscausedbysolutionandweathering,orbyshrinkage
ofthematrix.Theseformsofporosityarecalledsecondaryporosity.Anothertypeof
secondary porosity is that caused by fracturing and is very important in that it permits
manylimestonerocksofotherwiselowporositytobecomeexcellentreservoirs.
Porositymayrangefrom50%to1.5%andactualaveragevaluesarelistedbelow:
Recentsands(looselypacked) 35-45%
Sandstones(moreconsolidated) 20-35%
Tight/wellcementedsandstones 15-20%
Limestones(e.g.MiddleEast) 5 -20%
Dolomites(e.g.MiddleEast) 10-30%
Chalk(e.g.NorthSea) 5 -40%
Apointthatneedstobeemphasisedisthattheconceptofporosityiscomplexand
thereforediffculttodefneanddetermine.Itmayrefertospacesbetweensandgrains
oritmayrefertolimestonecaves:itmayevenexcludeafractionofthefreewater
(waternotboundchemically)presentintherock.Sometimesgoodestimates,(i.e.
relevanttoreservoirdevelopmentproblems)maybeobtainedfromlaboratorystudies,
orcoresamples,andsometimessuchmeasurementsareirrelevant.
Insummary,theamountofporosityisprincipallydeterminedbyshapeandarrangement
ofsandgrainsandtheamountofcementingmaterialpresent,whereaspermeability
depends largely on the size of the pore openings and the degree and type of cementation
between the sand grains. Although many formations show a correlation between
porosityandpermeability,thefactorsinfuencingthesecharacteristicsmaydifferwidely
ineffect,producingrockhavingnocorrelationbetweenporosityandpermeability.
3.3. Subsurface Measurement Of Porosity
Porosity is measured directly from recovered rock samples as part of core analysis
and also downhole by special tools which indirectly measure a property which can
berelatedtotheformationporosity.Thesedownholemeasurementtechniquesare
verysophisticatedinboththeirengineeringandintheirpractice.Forexample,the
porosityofaformationcanbeloggedwhiletheholeisbeingdrilled,givingalmost
real time indications of the nature of the reservoir. Core analysis procedures will be
reviewed later.
In general the downhole porosity may be related to the acoustic and radioactive
properties of the rock.
3.3.1 Density Log
The density log is derived from the response of the atoms in the minerals in the
rocktobombardmentwithgammaradiation.Theatomsacceptenergyofaspecifc
frequency and emit energy of a different frequency; this energy is detected. The
Institute of Petroleum Engineering, Heriot-Watt University
11
fundamental Properties of Reservoir Rocks
energy density is related to the number of atoms and therefore to the density of the
rockbeingbombarded.Iftheformationundertestisknown,forinstanceasandstone,
then changes in the density measured within the sandstone result from a change in
the porosity of the formation rather than a change in the mineralogical nature of the
sandstone. This obviously relies on a good description of the geology of the formation.
Inaporousformation,theporefuidwillalsoaffecttheresponseofthetoolinthat
the atoms of the fuid will also react to the bombardment and affect the energy
detected.Withreferencetocalibrationsamplesofdifferentrocktypes,theeffectof
bothmineralogyandporefuidcontentcanbeaccountedfor.Empiricalrelationships
have been developed to relate the porosity to the values of density which have been
logged.Inthefollowingrelationship,theloggeddensity,
L
,matrixdensity,
m
,and
thefuiddensity,
f
,arerelatedtotheporosity,



L m f
= (1- ) +
=
L m
f m

Thecontributionofthematrixandtheporefuidareinrelationtotherelativeamounts
ofeach,andthesearerelatedtotheporosity.Typically,matrixdensitiesandfresh
water density are as follows
Quartz = 2.65 gcm
-3
Limestone = 2.71 gcm
-3
Water = 1.00 gcm
-3
3.3.2 Sonic Log
Thislogissimilarinconcepttothedensitylog,however,itisacousticenergywhichis
radiated into the formation from sonic transducers in the logging tool. These produce
compression waves which travel along the side of the borehole in the formation. The
timetakenforthewavetotravelfromthetransmittertothereceiver(traveltime)is
relatedtotheacousticpropertiesoftheformation.Asforthecaseofthedensitylog,
iftheformationisknownanditsmineralogyisnotchanging,thenvariationsinthe
traveltimemustresultfromthechangesintheformationacousticproperties,themost
signifcantofwhichisthedensitywhichisrelatedtotheporosity.Aswiththedensity
tool,thedensityoftheformationfuidsintheporespaceswillaffectthetraveltime
and this must be accounted for. Calibration samples of different rock types have lead
toanempiricalrelationshipbetweentheloggedtraveltime,T
L
,matrixtraveltime,
T
m
,thefuidtraveltime,T
f
,andtheporosity, .




T = T (1- ) + T
=
L m f

T T
T T
L m
f m

Thecontributionofthematrixandtheporefuidareinrelationtotherelativeamounts
ofeach,andthesearerelatedtotheporosity.Typically,matrixtraveltimesandfresh
1
water travel time are as follows
TQuartz = 55s ft
-1
TLimestone = 47s ft
-1
TWater = 190s ft
-1

3.3.3 Neutron Log
Thisisanotherradioactiveloggingtechniquewhichmeasurestheresponseofthe
hydrogen atoms in the formation and can give an indication of the porosity. Neutrons
of a specifc energy are fred into the formation and they disrupt the steady state
activityofhydrogenatoms.Theythenradiateenergywhichisdetectedbythetool:
the energy returned is related to the number of hydrogen atoms which is related to
thehydrocarbonandwaterintheporespaces.Bycalibration,theporositycanbe
determined.
3.4 Average Porosity
Porosity is normally distributed and an arithmetic mean can be used for averaging.
Forunclassifeddata,

a
i
i 1
n
n

(1)
where
a
isthemeanporosity,
i
is the porosity of the ith core measurement and n is
the number of measurements.
4 PERMEABILITy
4.1 Darcy's Law
The permeabilityofarockisthedescriptionoftheeasewithwhichfuidcanpass
through the pore structure.
Atoneextreme,thepermeabilityofmanyrocksissolowastobeconsideredzero
even though they may be porous. Such rocks may constitute the cap rock above a
porous and permeable reservoir and they include in their members clays, shales,
chalk,anhydriteandsomehighlycementedsandstones.
Thepermeabilityisatermusedtolinkthefowratethroughandpressuredifference
across a section of porous rock. The problem is complicated in that the number of
porespaces,theirsizeandtheinterconnectionsisnotstandard.Thustheapplication
of the general energy equation, for example as in the case of fow through pipes,
becomesverydiffcultforfowthroughporousmedia.
In petroleum engineering the unit of permeability is the Darcy, derived from the
empirical equation known as Darcys Law named after a French scientist who
investigatedthefowofwaterthroughflterbedsin1856.Hisworkprovidedthe
basisofthestudyoffuidfowthroughporousrock.
Institute of Petroleum Engineering, Heriot-Watt University
1
fundamental Properties of Reservoir Rocks

Q
k P A
L

(2)
where:
Q = fowrateincm
3
/sec
A = cross sectional area of sample in cm
2
P= pressuredifferentacrosssample,atm
= viscosity in centipoise
L = length of sample in cm
k = permeability in Darcy
Darcys law of fuid fowstatesthatrateoffowthroughagivenrockvariesdirectly
withthepressureapplied,theareaopentofowandvariesinverselywiththeviscosity
of the fuid fowing and the length of the porous rock. In terms of equating the
parameters,theconstantofproportionalityintheequationistermedthepermeability.
The unit of permeability is the Darcywhichis defnedas thepermeabilitywhich
willpermitafuidofonecentipoiseviscosity(=viscosityofwaterat68F)tofow
at a linear velocity of one centimetre per second under a pressure gradient of one
atmosphere per centimetre. Permeability has the units Darcys. Figure 8 illustrates
the concept and the units of permeability

L = 1 cm
k = 1 darcy
1cm
2
Q = 1 cm
3
= 1 cp
p = 1 atmos
sec
Figure 8 Concept of permeable rocks
DarcysLawexperimentconsistedofasandpackthroughwhichwaterfowedata
constantrate(fgure9).
1

Manometric
heads of water
Length, L
Flowrate, Q
Flowrate, Q
h
1
h
2
S
a
n
d
Area of the end of the sandpack
Figure 9 SchematicofDarcysexperiment
Hisresultsshowedthatthefowratewasdirectlyproportionaltotheareaopentofow,
thedifferenceinpressureandinverselyproportionatetothelengthofthesandpack,
i.e.

Q A h
L
or
Q k
A h h
L


, ,
( )

1
1 2
whereQisthefowrate,Aistheareaoftheendofthecore,h
1
and h
2
are the static
headsofwaterattheinletandoutletofthecore(theequivalentofthestaticpressure),
L is the length of the core. K is the constant of proportionality. It is constant for a
particularsandpack.Whenotherworkersreplicatedtheexperiment,theresultswere
different to those of Darcy. This was accounted for by inclusion of the viscosity of
thefowingfuidandtheequationbecomes:

Q
kA h h
L

( )
1 2

where the original terms have the same meaning and istheviscosityofthefuid
in centipoise.
Onamoretheoreticalbasis,Poiseuilleformulatedtherelationshipbetweenfowrate
andpressuredropforfuidfowinginapipe.Theformoftherelationshipis

Q
r P
8 L
4


(3)
whereQisthefowrate,ristheradiusofthetube,istheviscosityofthefuidand
Listhelengthofthetube.Inthiscasethedependenceofthefowrate/pressuredrop
relationshipcanbeseentobedependentontheradiusofthetube.Inasimilarmanner,
theradiusoftheporesinarockdictatethenatureoftherelationship,specifcally,the
Institute of Petroleum Engineering, Heriot-Watt University
1
fundamental Properties of Reservoir Rocks
radiusoftheporethroatsisofmostsignifcance,sincethesearethesmallestradii
andthereforeaffectthefowrate/pressuredroprelationshipmost.
The practical unit is the millidarcy(mD)whichis10
-3
Darcy. Formation permeabilities
varyfromafractiontomorethan10000milli-Darcies.Atthelowendoftherange,
clays and shales have permeabilities of 10
-2
to 10
-6
mD. These very low permeabilities
make them act as seals between more permeable layers.
4.2 Factors Affecting Permeability
Permeabilityalongthefatsurfaceswillbehigher,thanthepermeabilityinadirection
perpendicular to the fat surfaces of the grains. In a reservoir, the permeability
horizontally along the bed is usually higher than the permeability vertically across the
bed because the process of sedimentation causes the grains to be laid down with their
fattestsidesinahorizontalposition(minimisingtheareaexposedtotheprevailing
currents during deposition). Figure 10 illustrates the concept.
If sand grains of generally fat proportions are laid down with the fat sides non-
uniformly positioned and located in indiscriminate directions, both porosity and
permeabilitymaybeveryhigh.Toillustrate,ifbricksarestackedproperly,thespace
betweenthebricksisverysmall;ifthesamebricksaresimplydumpedinapile,the
spacebetweenthebricksmightbequitelarge.

Horizontal permeability 400mD
Vertical permeability 200mD
Horizontal permeability 900mD
Vertical permeability 500mD
Porosity 16% Porosity 32%
Figure 10 Directional Permeability
The shape and size of sand grains are important features that determine the size of the
openingsbetweenthesandgrains.Ifthegrainsareelongated,largeanduniformly
arrangedwiththelongestdimensionhorizontal,permeabilitytofuidfowthroughthe
porechannelswillbequitelargehorizontallyandmedium-to-largevertically.Ifthe
grainsaremoreuniformlyrounded,permeabilitywillbequitelargeinbothdirections
and more nearly the same. Permeability is found generally to be lower with smaller
grainsizeifotherfactors(suchassurfacetensioneffects)arenotinfuential.This
occursbecausetheporechannelsbecomesmallerasthesizeofthegrainsisreduced,
anditismorediffcultforfuidtofowthroughthesmallerchannels.
1
This directional perspective to any property is termed anisotropy. As shown above
permeability is a directional property and gives rise to different permeabilities
depending on the shape and depositional characteristics. Very dramatic anisotropy
isgeneratedifarockisfractured.Theseanisotropicperspectivesareillustratedinfgure
11. Porosity is a non directional property and therefore is isoptropic.

Sandstone Fractured Core
Figure 11 Directional permeability.
4.3 Generalised Form Of Darcys Law
Athreedimensionalrockcanbedefnedwithintheco-ordinatesystemillustrated
infgure12.

-Z
+y
+Z
+x
Vs
s
0
Figure 12 Co-ordinate system for rock permeability
Thexandyco-ordinatesincreasefromzerototheleftandoutfromthepage;the
zco-ordinateincreasesdownwards.Thefowvelocityinaparticulardirectioncan
bedefnedasthefowrateinthatdirectiondividedbytheareaopentofow.Inany
direction,s,thefowvelocityistermedVsandisequatedtothestaticpressuregradient
inthatdirection(i.e.thechangeinpressure,dP,overasmallelementoflength,dsin
thatparticulardirection)minusacontributionfromthedifferenceinhead(because
ofthedifferenceinelevation)ofthefuidacrossthesectionds.Therefore,

V = -
k
s

(
.
)
dp
ds
g
x
dz
ds

1 0133 10
6
(4)
Institute of Petroleum Engineering, Heriot-Watt University
1
fundamental Properties of Reservoir Rocks
andthechangeinelevationheadisequaltothesineoftheangletothehorizontal

Q k
A(h
1
h
2
)
L
Q
kA(h
1
h
2
)
L
Q
r
4
P
8L
V
s
= -
k

(
dp
ds

g
1.0133x10
6
dz
ds
)
dz
ds
= sin , where is in degrees.
V
s
= -
K

(
dP
ds

g
1.0133x10
6
dz
ds
)
V
s
=
L
T
=
M
LT
=
M
L
3
P =
M
LT
2
g =
L
T
2
dP
ds
=
M
L
2
T
2
L
T
=
kLT
M
(
M
L
2
T
2

ML
L
3
T
2
)
L
T
=
K
LT
K = L
2
V
s
= -
K

(
dP
ds

g
1.0113x10
6
dz
ds
),
g
1.0113x10
6
dz
ds
= zero
V
s
= V
x
=
Q
A
Q = -
kA

dP
dx
Q dx
0
L

= -
kA

dP
P
1
P
2

Q(L- 0) = -
kA

(P
2
P
1
)
Q =
kA(P
1
P
2
)
L
(6)
V
s
= V
x
= - k(
dP
ds

g
1.0113x10
6
dz
ds
),
g
1.0113x10
6
dz
ds
= zero
V
s
=
Q
A
Q = -
kA

dP
dx
= sine ,where is in degrees.
TheDarcyunitsare:
V
s
= velocity along path s - cms
-1
k = permeability - Darcys
= viscosity - centipoise
= densityoffuid-gcm
-3
g = acceleration due to gravity - 980 cms
-2

Q k
A(h
1
h
2
)
L
Q
kA(h
1
h
2
)
L
Q
r
4
P
8L
V
s
= -
k

(
dp
ds

g
1.0133x10
6
dz
ds
)
dz
ds
= sin , where is in degrees.
V
s
= -
K

(
dP
ds

g
1.0133x10
6
dz
ds
)
V
s
=
L
T
=
M
LT
=
M
L
3
P =
M
LT
2
g =
L
T
2
dP
ds
=
M
L
2
T
2
L
T
=
kLT
M
(
M
L
2
T
2

ML
L
3
T
2
)
L
T
=
K
LT
K = L
2
V
s
= -
K

(
dP
ds

g
1.0113x10
6
dz
ds
),
g
1.0113x10
6
dz
ds
= zero
V
s
= V
x
=
Q
A
Q = -
kA

dP
dx
Q dx
0
L

= -
kA

dP
P
1
P
2

Q(L- 0) = -
kA

(P
2
P
1
)
Q =
kA(P
1
P
2
)
L
(6)
V
s
= V
x
= - k(
dP
ds

g
1.0113x10
6
dz
ds
),
g
1.0113x10
6
dz
ds
= zero
V
s
=
Q
A
Q = -
kA

dP
dx
= pressure gradient along s - atm cm
-1
1.0133x10
6
converts from dynes cm
2
to atmospheres
4.4 Dimensions Of Permeability
FromDarcysequation,
V = -
k
s

(
.
)
dp
ds
g
x
dz
ds

1 0133 10
6
the dimensions of each
termcanbededucedintermsoflength,L,mass,Mandtime,T

V = = =
P = g = =
s
L
T
M
LT
M
L
M
LT
L
T
dP
ds
M
L T

3
2 2 2 2
Therefore,theequationintermsofthedimensions(andkeepingpermeabilityask)
is

L
T
kLT
M
M
L T
ML
L T
L
T
K
LT
=
=
K = L
2
( )
2 2 3 2

(5)
Itcanbeseenthatthedimensionsrefectthenatureoftheconstantofproportionality
anditshouldnotbeconfusedwith,forexample,theareaopentofow,A,oftheend
ofacoreorasandpack.Intermsofmetricunits,since1atm=14.73psi=1.013
bar and 1 cp = 10
-3
Pas it follows that
1D =9.87x10
-13
m2~1x10
-12
m
2
1mD =9.87x10
-16
m
2
~1x10
-15
m
2
1
Other units of inches
2
or cm
2
could be used but they are all too large for porous media
andtheywouldalsorequireconversiontorelatetopermeabilitiesquotedinother
units. Darcys and milliDarcys are most commonly used.
4.5 Assumptions For Use Of Darcys Law
The simple Darcy Law, as used to determine permeability, only applies when the
followingconditionsexist:
(i) Steadystatefow
(ii) Laminarfow;
(iii) Onephasepresentat100%porespacesaturation.
(iv) Noreactionbetweenfuidandrock;
(v) Rockishomogenous

1. Steady state fow, i.e. no transient fow regimes. This becomes unrealistic in
terms of fow in a reservoir where the nature of the fuids and the dimensions of
thereservoirmayproducetransientfowconditionsformonthsorevenyears.For
laboratorybasedtests,thecoresaresmallenoughthattransientconditionsusually
last only a few minutes.
2.Laminarfow,i.e.noturbulentfow.Formostreservoirapplicationsthisisvalid
howeverneartothewellborewhenvelocitiesarehighforexampleingasproduction
turbulentfowoccurs.Sometimesitistermednon- darcy fow. Figure 13

Laminar Flow
Turbulent Flow
=
P
Q
A

L
K

=
. .
Q
A
k

.
P
L
Q
A
P
L
Figure 13 Effect of Turbulent Flow on Measured Permeability
Institute of Petroleum Engineering, Heriot-Watt University
1
fundamental Properties of Reservoir Rocks
3.Rock100%saturatedwithonefuid,i.e.onlyonefuidfowing.
Inthelaboratorythiscanbeachievedbycleaningcores,however,therewillbea
certainconnatewatersaturationinthereservoir,andtheremaybegas,oilandmobile
waterfowingthroughthesameporespace.Theconceptofrelativepermeabilitycan
beusedtodescribethismorecomplexreservoirfowregime.Relativepermeability
is discussed later.
4.Fluiddoesnotreactwiththerock,i.e.itisinertandthereisnochangetothepore
structure through time.
Therearecaseswhenthismaynothappen,forexamplewhenawellisstimulated
duringanhydraulicfracturingworkover.Thefuidsusedmayreactwiththeminerals
oftherockandreducethepermeability.Insuchcases,testsontherocktodetermine
thecompatibilityofthetreatingfuidsmustbeconductedbeforetheworkover.
5. Rock is homogeneous and isotropic, i.e. the pore structure and the material
propertiesshouldbethesameinalldirectionsandnotvary.Inreality,thelayered
natureandlargearealextentofareservoirrockwillproducevariationsinthevertical
and horizontal permeability.
4.6 Applications of Darcys Law
Toexaminetheapplicabilityofthissimplerelationship,approximationstothetype
offowencounteredinareservoircanbemade:linearfowalongareservoirsection
andradialfowintoawellbore.Morecomplexgeometriescannotbeanalysedusing
thissimpleanalyticalequationandformsofapproximatingthegeometryandfow
arerequired.
Inthefollowingexpressions,thenomenclatureisidenticaltothatusedabove.
(i)Horizontal,linear,incompressibleliquidsystem(fgure14)

A
L
P
1
P
2
Q

Figure 14 Linearfowregime
FromthebasicDarcyequation

V = - , = zero
s
K dP
ds
g
x
dz
ds
g
x
dz
ds

(
.
)
.

1 0113 10 1 0113 10
6 6
0
Thefowrateandareaopentofowissubstitutedforthefowvelocity.Thevariables
areseparatedandintegratedoverthelength(forthefowrate)andthepressuresP
1

to P
2
for the change in pressure. The pressure drop P
2
minus P
1
is negative and is
correctedbythenegativesignonthelefthandsideoftheequation.

V = V =
Q = -
= -
Q(L - 0) = -
Q =
s x
Q
A
kA dP
dx
Q dx
kA
dP
kA
P P
kA P P
L
L
P
P

0
1
2
2 1
1 2

( )
( )

(6)
ThefnalformisasformulatedbyDarcyandthepermeabilitywillhavetheunitsof
Darcysiftheotherunitsare:
fowrate,Q-cm
3
s
-1
pressure,P-atm
areaopentofow,A-cm
2
length,L-cm
viscosity, - centipoise
(ii)Horizontal,linear,compressibleidealgassystem
ThefowregimeisthesameasforthelinearliquidsystemandfromthebasicDarcy
equation:

V = V = - , = zero
V =
Q = -
s x
s
k
dP
ds
g
x
dz
ds
g
x
dz
ds
Q
A
kA dP
dx
(
.
)
.

1 0113 10 1 0113 10
6 6
Inthiscase,thelaboratorymeasurementofthegasfowwouldusuallybeconducted
downstream from the core at almost atmospheric conditions (i.e. there would not
bealargepressuredropacrossthefowmeter).Itisassumedthatthegasusedis
ideal,however,thereneedstobeacorrectiontothevolumetricfowratemeasured
to account for the higher pressure in the core. Figure 15.
Institute of Petroleum Engineering, Heriot-Watt University
1
fundamental Properties of Reservoir Rocks

Valve
Qb
P1 P2 Pb
Flow
measurement
Core
L
A
P
Figure 15 Confgurationforgaspermeabilitymeasurements.
Thefowratemeasured,Q
b
atambientpressure,P
b
isrelatedtothefowrate,Qinthe
coreatthepressureinthecore,Pviatheidealgaslaw.Iftheassumptionismade
thatthetemperatureisconstant,then

QP = Q P
Q =
Q P
P
b b
b b
and substituting into the equation, separating the variables and integrating
produces

Q P
P
= -
Q P = -
Q P (L - 0 = -
Q =
b b
b b
b b
b
kA dP
dx
dx
kA
PdP
kA P P
kA P P
LP
L
P
P
b

0
1
2
2
2
1
2
1
2
2
2
2
2

( )
( )
(7)

k
Q P L
A P P
b b

2
1
2
2
2

( )
(8)
Comparingthetwoexpressionsequations6and7,itisseenthatthegasfowrateis
proportionaltothedifferenceinthepressuresquared,whereastheliquidfowrate
isproportionaltothedifferenceinthepressure.Inwelltesting,thefowratesare
measuredatthesurfaceandforgaswellsoneofthediagnosticplotsisthefowrate
versusdifferenceinpressuresquaredplot.Neglectingthefactthatthegasisreal,it
gives an indication of the ability of the reservoir to produce gas.


Gas Q = LiquidQ =
b
kA P P
LP
kA P P
L
b
( ) ( )
1
2
2
2
1 2
2


Incertaincircumstances,themeanfowrate,Qismeasuredatameanpressure,P
which,inthecaseofalaboratoryexperimentonacore,isthemeanoftheupstream
anddownstreampressure,i.e.

P
P P
=
1 2
2
+
and

Q=VolumefowrateatP
P Q = P
b
Q
b
substitutingthisintotheabovegasequation7.

P Q =
b b
PQ
kA P P
L

( )
1
2
2
2
2
and since

1
2
P P Q
1
2
kA
L
(P P )(P P )
1 2 1 2 1 2
+ ( ) +

Q
kA P P
L
( )
1 2
(9)
Theidealgaspermeabilitycanbecalculatedfromtheliquidequationusingmean
fowrate,Q measured at mean pressure.
(iii)Horizontal,radial,incompressibleliquidsystem(fgure16)

Well
Radial flow
Plan Elevation
P
e
P
w
r
w
r
e
r
e
r
w
h
Figure 16 Radialgeometrywithradialfowfromtheouterboundarytothewellbore
Institute of Petroleum Engineering, Heriot-Watt University

fundamental Properties of Reservoir Rocks


r
e
is the outer boundary radius
r
w
istheinnerboundaryradius(well)
P
e
isthepressureattheexternalboundary
P
w
is the pressure at the inner boundary
StartingfromthebasicDarcyexpressionagain,

V = - , = zero
s
k dP
ds
g
x
dz
ds
g
x
dz
ds

(
.
)
.

1 0113 10 1 0113 10
6 6
Substitutingforfowvelocity,
V = V =
s r
Q
A
Inthiscasethedirectionoffowisintheoppositesensetotheco-ordinatesystem,
therefore
ds = -dr
Forradialgeometry,thearea,A,isnowradiusdependenttherefore
A = 2rh
Substitutionintothebasicexpressiongives

Q
rh
k dP
dr 2
= -

(10)
separating the variables and integrating

Q
h
dr
r
k
dP
Q
h
r r
k
P P
r
w
r
e
P
w
P
e
e w e w
2
2


(ln ln ) ( ) =
whichgivesthefnalform

Q =
2

kh P P
r
r
e w
e
w
( )
ln

(11)
(iv)Horizontal,radial,compressiblerealgassystem
Inthiscasethegeometryisidenticaltothatoftheradialfowofincompressible
fuidwiththemodifcationsforthecompressibilityofagasasperthelineargas
fowsystem.


V = - , = zero
= -
s
k dP
ds
g
x
dz
ds
g
x
dz
ds
Q
rh
k dP
dr



(
.
)
.

1 0113 10 1 0113 10
2
6 6
Iftheassumptionismadethatthetemperatureisconstant,then

QP = Q P
Q =
b b
Q P
P
b b
andsubstitutingintotheequation,10

Q P
P
2 rh
k dP
dr
b b

separating the variables



Q P
dr
r
2 kh
PdP
b b
r
r
P
P
w
e
w
e

and integrating produces



Q P ln
r
r
2 kh P P
2
Q
kh
P ln
r
r
P P
b b
e
w
e
2
w
2
b
b
e
w
e
2
w
2
,

,
]
]
]

,

,
]
]
]

j
(
,
\
,
(

( )

(10)
4.7 Field Units
Measurements made in the feld are often quoted in feld units and to ensure
compatibilitywiththeDarcyequation,aconversionisrequired.Thefeldunitsare
usuallyasfollows:
Flowrate,Q-bbl/dayorft
3
/day
Permeability,k-Darcy
Thicknessorheightofreservoir,h-feet
Pressure,P-psia
Viscosity,m-centipoise
Radius,r-feet
Length,L-feet
Institute of Petroleum Engineering, Heriot-Watt University

fundamental Properties of Reservoir Rocks


InordertoconverttheDarcyequationforliquidfow, Q =
KA P P
L
( )
1 2


Q
=
bbl
day
ft
bbl
in
ft
cm
in
day
hr
hr
s
K Aft
cm
ft
Ppsia
atm
psia
Lft
cm
ft
(
.
)( )(
.
)( )( )
( )( )( )( )(
.
)
( )( )(
.
)
5 615 1728 16 39
24 3600
929
14 696
30 48
3 3
3
3
3
2
2
2

tooilfeldunits,thefollowingconversionfactorsareused:

Q = 1.1271
bbl
day
KA P P
L
( )
1 2

andtheseproducethefollowingversionofDarcysequationinfeldunits:

Q = 1.1271
bbl
day
KA P P
L
( )
1 2

(11)
4.8 Klinkenberg Effect
DarcysLawwouldindicatethatthepermeabilityshouldbethesameirrespective
ofthefuidtransmitted,sinceviscosityisincludedintheequation.Measurements
madeongasasagainstliquidforsomeconditionsgivehigherpermeabilitiesthan
theliquid.
This phenomenon is attributed to Klinkenberg, who attributed the behaviour to the
effect of the slippage of gas molecules along the solid grain surfaces. This occurs
whenthediameterofthecapillaryopening(porethroatdiameter)approachesthe
meanfreepathofthegas(i.e.thereisineffectonlyonegasmoleculepercapillary).
DarcysLawassumeslaminarfowandviscoustheoryspecifeszerovelocityatthe
boundaryofthefowchannel.Thisisnotvalidwhenthemeanfreepathofthegas
approaches the diameter of the capillary and the result is that low pressure permeability
measurementsareunrealisticallyhighbecausethereisinsuffcientgasmoleculesto
form a zero velocity boundary layer at the edges of the pores and to form a mass of
fowinggaswithinthepores.Inthiscase,toomanygasmoleculesfowthroughthe
poresandthepermeabilityappearstobehigherthanitactuallyis:theeffectreported
by Klinkenberg.

Sincethemeanfreepathisafunctionofthesizeofthemolecule,thepermeability
is a function of the type of gas used in the permeability measurement. This gas
permeability is corrected for the Klinkenberg effect by plotting the gas permeability
ateachreciprocalmeanpressure.Thisisillustratedforhydrogen,nitrogenandcarbon
dioxideinfgure17:


100
80
0 5
Reciprocal Mean Pressure: (Atm.)
G
a
s

P
e
r
m
e
a
b
i
l
i
t
y
:

M
i
l
l
i
d
a
r
c
i
e
s
60
40
20
0
1 2 3 4
Hydrogen
Nitrogen
Carbon Dioxide
Liquid permeability
Figure 17 Variation in gas permeability with reciprocal mean pressure.
P
m
isthemeanpressureofthegas(themeanoftheupstreamanddownstreampressures
either end of the core orpinfgure15).Ineffect,ifthegaspressureisraisedinfnitely
high, the gas will perform as an incompressible liquid would, therefore if several
measurementsofpermeabilityaremadeatdifferentmeanpressures,therelationship
between mean pressure and permeability can be extrapolated to the equivalent
pressureconditionsofaliquid.Inreality,extrapolationtoinfnityisimpossible,so
thereciprocalmeanpressureisusedandtheresultsareextrapolatedtozeroreciprocal
meanpressure(i.e.1/infnitelyhighmeanpressure).This point corresponds to the
liquid permeability.Thedifferentgasseshavedifferentslopes,buttheyallextrapolate
tothesameequivalentliquidpermeability.
TheformoftheequationdevelopedbyKlinkenbergisoftheform

k
k
l
b
P
L
G
m

+
(12)
where
k
L
=equivalentliquidpermeability
k
G
= permeability to gas
P
m
=meanfowingpressure
Institute of Petroleum Engineering, Heriot-Watt University

fundamental Properties of Reservoir Rocks


b=Klinkenbergconstantforaparticulargasandrock(slopeofthegaspermeability,
inverse mean pressure relationship).
The Klinkenberg effect is greatest for low permeability rocks and low mean
pressures.
4.9 Reactive Fluids
Darcys Law assumes that the fuid does not react with the formation. Many
formation
watersreactwithclaysintherocktoproducealowerpermeabilitytoliquidthan
would be obtained with gas. Therefore the permeability to water in the formation
may be much lower than would be determined to gas in the laboratory. Any water
injectedintotheformationmayseverelyreducethepermeabilityduetoclayswelling.
Thechangeinpermeabilitymaybesubstantial,forexamplefromseveralhundred
millidarcys to less than one millidarcy.
4.10 Average Reservoir Permeability
Permeabilityisnotnormallydistributedbuthasanexponentialdistribution,therefore
a geometric mean is used to obtain an average reservoir permeability.
The Geometric Mean of n numbers is the n
th
rootoftheirproduct:
5 STRESS EffECTS ON CORE MEASuREMENTS
5.1 Stress Regimes
Inreservoirengineeringtheimpactofreservoirstressesonreservoirfowandcapacity
parametershasbeenconsideredforanumberofyearsbut,increasingly,theinterest
in stress related measurement has grown. The effect of removing a core from the
formation is to remove all the confning forces on the sample, allowing the rock
matrix to expand in all directions, partially changing the shapes of the fuid-fow
paths inside the core.
It is worth considering the stresses associated with reservoir rock parameters. Figure
18illustratesthelikelyconfgurationofacoreextractedfromaverticalwell,andthe
orientationofthecoreplugextractedforpermeabilityandporositymeasurements.


Whole core
Core plug
for horizontal
k measurement
4 Inch
Formation
Core plug
for vertical
k measurement
Figure 18 Trends in Reservoir Rock Characterisation
Withinareservoirthestressesintheformationcanbeexpressedinthreedirections,
themajorandtwominorprincipalstresses.Figure19a.Themajorprincipalstress
acting mainly in the vertical direction. Clearly the depositional environment and
formation structure will result in slight changes to these orientations.
Institute of Petroleum Engineering, Heriot-Watt University

fundamental Properties of Reservoir Rocks



Major Principal Stress
Minor Principal Stress
Minor Principal Stress
Equal Stresses
Equal Stresses
K
h
(a)
(b)
(c)
Figure 19 Stress States in Reservoirs and Cores
In core analysis, service companies have been asked to measure porosity and
permeability under reservoir stress conditions. They have done this by applying
differentstressesfortheaxialandradialstresses.AscanbeseeninFigure19bfora
conventionalplugtheradialstresswouldbeacombinationofthemajorandaminor
principalstress.Toenablethetruestressfeldtoberepresented,avaryingradial
stressdistributionwouldberequired.Ifaverticalplugwasused,Figure19c,thena
constant radial stress could be an acceptable value for the average minor stresses. In
thiscase,however,thepermeabilityvaluewouldbeK
v
,theverticalpermeability.
Theeffectoftheoverburdenandtheporepressureonthematrixistoproduceanet
forcebetweenthegrainsofthematrix(which,whentheareaoverwhichtheforce
actsisaccountedforproducesanetstress).Ifthematrixisconsideredtobeelastic,
that is, there is a unique relationship between the stress and the strain within the
matrix,thenthematrixwillstrainasthestressisaltered.Ifthestressincreases,the
0
strain reduces the radius of the pore throats and reduces the volume of the pore space.
This effect may be different for different rock types and even within the same rock
typeiftheamountofcementingmaterialisaltered.Thesignifcantaspectsofthis
phenomenonarewhencoresareremovedfromsubsurfacetothelaboratory(since
the overburden and pore pressure will change) and when the pore pressure in the
reservoirchangesduetolocalpressureconditionsaroundthewells(drawdown)and
withinthereservoirasawholeasitisdepressurised,forexample.Theimpactof
thenetoverburdenstresswhichincreasesasthereservoirpressure(porepressure)
decreasesisillustratedinfgure20.

1.0
.8
0 10000
Net Overburden Pressure: PSI
P
e
r
m
e
a
b
i
l
i
t
y
:

F
r
a
c
t
i
o
n

o
f

O
r
i
g
i
n
a
l
.6
.4
.2
0
2000 4000 6000 8000
?Unconsolidated
?Friable
?Well Cemented
Figure 20 Permeability Reduction with Net Overburden Pressure
Ingeneral,thestressregimesubsurfaceisconsideredtobehydrostatic(asinthecase
oftheporefuid)andthatthestressescanberesolvedintooneverticalstress,and
twohorizontalstresses.Forhydrostaticconditions,allofthesearethesame.Incore
analysis,therefore,theporosityatequivalentsubsurfaceconditionsmaybedetermined
byapplyinganexternalpressuretothecore.Thisisusuallydonebyinsertingthecore
intoacellratedforpressuresupto10000psi(68.9MPa)andapplyingastresstothe
ends of the core and to the sides. The nature of these tests are such that usually the
stress applied to the sides of the core represents the horizontal stress and the stress
appliedtotheendsrepresentstheverticalstress.Oncetrappedinsidethecell,the
pore pressure may be increased to a representative level and measurements of pore
volume and permeability made under these stress conditions.
Institute of Petroleum Engineering, Heriot-Watt University
1
fundamental Properties of Reservoir Rocks
More recently, the effect of non-hydrostatic stress conditions has been shown to
be important in certain reservoir conditions, such as in tectonically active areas
(Columbia,SouthAmericawheretheformationoftheAndesmountainsisassociated
with large horizontal stresses) or in areas associated with faults or very compressible
reservoir rocks such as some chalks. In this case the conventional test cells are not
appropriateandspecialtruetriaxialcellsarerequired.Inthesecellstheendsofthe
corearesubjectedtotheverticalstressaspertheconventionalcells,butthesidesof
the core are wrapped in a cage of individual tubes which can be pressurised in banks
around the core to represent the different horizontal stresses.

Insummary,whenthepropertiesofthecoresaremeasuredinthelaboratory,they
canbesubjectedto
Zero stresses No effect of the stress on the property
Hydrostatic stresses The effect of the magnitude of the stresses are measured
Triaxialstresses Theeffectofstressesresolvedinthethreeprincipal
directions are measured
Real stress behaviour The effect of the magnitude and direction of the stresses
are measured
Thistopiciscoveredinmoredetailinthesubsequentchapter.
5.2 Compressibility Of Porous Rock
Astherockmatrixissubjectedtoastress,itwilldeformandaltertheporespace
volumeastherockiscompressed.Forsimplicity,theoverburdenwillbeconsidered
toproducehydrostaticstress(calledthecompactingstress)onthereservoir,i.e.a
grain-to-grainstressintherock.Withinthepores,fuidpressureactsonthesurface
ofthegrainsandreducesthegrain-to-grain(orcompacting)stress.Thereforeina
real reservoir there is a balance between the effect of the overburden stress and the
pore pressure. This can be described by the relationship
P
compacting
= P
overburden
- P
pore pressure
where P
compacting
is the grain-to-grain stress, P
overburden
is the stress produced by the
weight of the overburden at a particular depth and P
pore pressure
is the pressure of the
fuidsinthepores.Theexpressionshowsthebalancebetweentheoverburdenand
theporepressureincompactingtherockmatrix:iftheporepressuredeclines,the
compacting stress increases and the pore volume declines. This assumes that the
overburden remains constant which is logical over the time period of a producing
reservoir.Thebalancecanberepresentedbyfgure21:


Enlarged view of the pore space
Grains
Pore space filled with fluid
Reservoir
Cap Rock
Depth
P
f
and P
c
P
o
Surface
P
c
P
c
P
f
P
c
P
c
P
c
P
c
Figure 21 Thebalancebetweenoverburden&rockstressandfuidpressure
P
o
= P
f
+ P
c
P
o
= overburden pressure
P
f
=fuidpressure
P
c
= compacting stress
The effect of the change in the balance between the overburden stress and the pore
pressureistochangethecompactingstress.Ifthereisanincreaseinporepressure,
then the pore volume will increase, however, this is rare and in the main, pore
pressure declines during production and the pore spaces compact under the increasing
compactstress.Twoissuesaresignifcant:theinitialporosityinthereservoir(i.e.
tocorrectlydefnethevolumeofoilinplace)andthereductioninthatporosity(or
porevolume)asthepressuredeclines(formaterialbalanceandsimulationstudies).
Figure22showstherelationshipbetweenporosityanddepth(orstress).Asthedepth
(andstress)increases,theporositydeclines.Careneedstobetakenwhenassessing
porosity values: were they measured under overburden or at ambient conditions?
The shale sample shows a large change in porosity as the plate-like clay minerals
arecompactedandfttogetherinamorecongruentmanner.
Institute of Petroleum Engineering, Heriot-Watt University

fundamental Properties of Reservoir Rocks



Sandstone
Shale
50
40
30
20
10
0
0 3000 6000
Depth of burial (ft) or stress (psi)
P
o
r
o
s
i
t
y
,

Figure 22 Alterationinporositywithdepthofburial(orstress)
The rate of change of pore volume with pressure change can be represented by an
isothermalcompressibility(assumingtemperatureisconstant):

C = -
f
1
v
dv
dP
(15)
where C
f
istheisothermalcompressibility,visthevolume,dvisthechangeinvolume
anddPisthechangeinpressure(thenegativesignaccountsfortheco-ordinatesystem:
asthepressureincreases,thevolumedecreases).
5.3. Types Of Compressibility
An issue with regard to the compressibility is: which part of the reservoir is
being compressed and which part is signifcant in calculating the response of the
reservoir.
Threetypesofcompressibilitycanbeconsidered:
(i)Matrixvolumecompressibility-thechangeinvolumeoftherockgrains.Thisis
very small and usually not of interest in sandstones since it is a purely mechanical
change in volume of the very stiff grains.
(ii)Bulkvolumecompressibility-thechangeintheunitvolumeoftherock.This
is of interest in reservoirs near the surface because of the problem of subsidence;
Changes in volume of the reservoir around faults which may cause the fault to slip
and alter the conductivity both through the fault and across it;

Reservoirs composed of unconsolidated or very weakly consolidated material where
thechangesinporositycanbesignifcant.Thechangesinthevolumeofthereservoir
both in a vertical sense leading to subsidence and in a horizontal sense leading to
shearing of the wellbore and the associated loss in integrity.
(iii)Porevolumecompressibility-changeinporevolume.Thisisofgreatestinterest
since the pore volume affects the porosity which affects reservoir performance.

Forcompleteness,allaspectsofthereservoircompressibilityshouldbeconsidered,
however, in many problems only specifc aspects of the compressibility may be
required such as in a well cemented sandstone reservoir where the bulk volume
changeisverysmallandthesubsidenceisnegligible,buttheporecompressibility
is an important feature of the drive mechanism.
5.4 Measurement Of Pore Volume Compressibility
The measurement of pore compressibility is usually conducted in a coreholder which
appliesanequalcompactingpressurearoundthecore.Aninnerlinerensuresthepower
fuid(usuallyhydraulicoil)doesnotcontaminatetheporesofthesample.Thepore
pressureisusuallykeptatambient,i.e.thecompactingpressuremimicstheneteffect
oftheoverburdenandtheporepressureinthereservoir.Thismakesthetestsimpler,
however,theremaybeconditionswherethecompressibilityofthegrainsthemselves
plays a signifcant role in the system and the test may require to be conducted at
true overburden and pore pressure conditions. For the test at ambient pore pressure
conditions,anoutletisconnectedtothecoreholderandthisisleadtoapipetteora
balancetomeasuretheamountofporefuidexpelled.Thepressureofthehydraulic
oilisincreasedinstagesandforeachstagetheamountoffuidexpelledismeasured
aftertherockhascometoequilibrium.Thedatacanthenbeanalysedtoindicatethe
change in porosity or pore compressibility. Figure 23 shows the concept.

Sealed core
Pump
Pipette
Pressure vessel
Figure 23 Measurementofthereductioninporevolumeastheexternalstress(orcompact-
ing pressure) is increased
The results show the change in pore volume relative to the original pore volume,
for a given change in the compacting pressure (this assumes that changes in the
compacting pressure have the same effects as changes in the pore pressure) which
can be substituted in to the isothermal compressibility as

C = -
p
1
v
dv
dP
p
p
c
where:
C
p
= pore volume compressibility
v
p
= initial pore volume
dv
p
=changeinporevolume(amountoffuidexpelled)
dP
c
= change in compacting pressure
Institute of Petroleum Engineering, Heriot-Watt University

fundamental Properties of Reservoir Rocks


Typicalvaluesofporecompressibilityareintherange3x10
-6
psi
-1
to10\x10-6

psi
-1
,
however,softsedimentscanhavecompressibilitiesintherange10\x10-6psi
-1
to20x10
-6

psi
-1
or 30 *10
-6
psi
-1
. Figure 24 illustrates the values determined for some limestones
and sandstones.

Sandstone
Limestone
Porosity %
P
o
r
e

c
o
m
p
r
e
s
s
i
b
i
l
i
t
y

1
0
-
6

p
s
i
-
1
10
9
8
7
6
5
4
3
0
10 20
Figure 24 Compressibility of Sandstones and Limestones
5.5 Effect of stress on permeability
Astheeffectofastressontherockmatrixaffectstheporevolume,italsoaffectsthe
porethroatradiiandthepermeabilityoftherock.Ingeneral,anincreaseinstress
reducestheporethroatradiiandthepermeabilitydeclines.Formostrockssubjected
toanhydrostaticstress,thisisthecaseasthestressisequalinalldirections.Figure
25 shows typical permeability declines for increase in stress for sandstone.


Permeability stress sensitivity for various sandstones
1000
100
10
1
0 20 40 60 80
Hydrostatic stress (MPa)
P
e
r
m
e
a
b
i
l
i
t
y

(
m
D
)
Figure 25 Thereductioninpermeabilityforarangeofsandstonesamples(theporosityis
intherange15%to22%)
Unconsolidated material has larger absolute changes in permeability as the total
strain is greater.
Intruetriaxialstressregimes,thestressesarenotidenticalandthestrain(andtherefore
pore throat radii) may cause the sample to dilate in one direction and increase the
pore throat radii therefore enhancing the permeability. This can be illustrated better
byconsideringafracturedcore(fgure26).

Institute of Petroleum Engineering, Heriot-Watt University

fundamental Properties of Reservoir Rocks


h maximum
Permeability
Fracture
Core

h maximum

h minimum

h maximum perpendicular to fracture


Fracture
Core

h minimum

h maximum

h maximum parallel to fracture

h maximum
Permeability
Fracture closing under stress
Fracture opening under stress
Figure 26 Triaxialstressesappliedtoafracturedcore

Ifthelargesthorizontalstressactsacrossthefracture(i.e.perpendiculartothefacesof
the fracture) then it will be clamped shut; if the largest horizontal stress acts parallel
tothefracture,thenitmaysplitopen.Inthiswaytheanisotropy(ordifferenceinthe
properties) may lead to different permeabilities and porosities from the same sample
if the stresses are applied in different ways around the core.
6. POROSITy-PERMEABILITy RELATIONShIPS
Whereasforporositythereareanumberofdownholeindirectmeasurementmethods,
the same is not the case for permeability. The downhole determination of permeability
ismoreillusive.Downholepermeabilityismainlyobtainedbyfowandpressure
determination and requires other characteristics for example the fowing interval.
There has been a continued interest in porosity-permeability correlations, on the
basis if one has a good correlation of laboratory measured porosity and permeability
then down hole measurements of porosity could unlock permeability values for
those formations where recovered core has not been practical. Although porosity is
anabsolutepropertyanddimensionless,permeabilityisnotandisanexpressionof
fowwhichisinfuencedbyarangeofpropertiesoftheporousmedia,includingthe
shape and dimensions of the grains and the porosity. Since porosity is an important
parameter in permeability it is not surprising for those rocks which have similar particle
characteristicsthatarelationshipexistsbetweenporosityandpermeability.Figure
27belowgivesexamplesofpermeabilitycorrelationsfordifferentrocktypes.


1000
100
10
1.0
0 5 10 15 20 25 30 35
Porosity: Percent
P
e
r
m
e
a
b
i
l
i
t
y
:

M
i
l
l
i
d
a
r
c
i
e
s
Reef Limestone
Sucrosic Dolomite
?Oolitic Limestone
Chalky
Limestone
Intercrystalline
Limestone and
Dolomite
Fine Grained
Friable Sand
Well Cemented
Hard Sand
Figure 27 Permeability and Porosity Trends for Various Rock Types
(CoreLaboratoriesInc)
7 SuRfACE kINETICS
Ifcoreforaparticularsectioncannotberecovered,orforexampleisformedasapile
ofsandontherigfoor,thencorrelationsliketheseinfgure27areused.Porosity
measurements obtained indirectly from wireline methods can be used to obtain the
laboratory porosity vs down hole porosity cross plot. Using this laboration porosity
value the associated permeability value can be determined from an appropriate
correlationasinfgure27.
Thesimultaneousexistenceoftwoormorephasesinaporousmediumneedsterms
such as the capillary pressure, relative permeability and wettability to be defned.
Withonefuidonlyonesetofforcesneedstobeconsidered:theattractionbetween
thefuidandtherock.Whenmorethanonefuidispresenttherearethreesetsof
active forces affecting capillary pressure and wettability.
Surface free energy exists on all surfaces between states of matter and between
immiscibleliquids.Thisenergyistheresultofelectricalforces.Theseforcescause
molecularattractionbetweenmoleculesofthesamesubstance(cohesion)andbetween
moleculesofunlikesubstances(adhesion).
Institute of Petroleum Engineering, Heriot-Watt University

fundamental Properties of Reservoir Rocks


Surfacetension(orinterfacialtension)resultsfrommolecularforcesthatcausethe
surfaceofaliquidtoassumethesmallestpossiblesizeandtoactlikeamembrane
under tension.
7.1 Capillary pressure theory
Theriseordepressionoffuidsinfneboretubesisaresultofthesurfacetensionand
wettingpreferenceandiscalledcapillarity.Capillarypressureexistswhenevertwo
immisciblephasesarepresent,forexample,inafneboretubeandisdefnedasthe
pressuredropacrossthecurvedliquidinterface.Theequilibriuminforcebetweenthe
moleculesofasinglephaseisdisruptedataninterfacebetweentwodissimilarfuids.
The difference in masses and the difference in the distances between the molecules
of the different phases produces an initially unbalanced force across the interface. Figure
28 shows the interface between oil and water molecules.

Different mass.
Different space
between molecules.
W
W W W
O
O O O
W: water molecule
O: oil molecule
distance between molecules
Figure 28 Representation of an oil water boundary
Interfacialtensiondeformstheoutersurfaceofimmiscibleliquidstoproducedroplets.
Ifthetwoliquidsarepresentonasurface,theinterfacialtensiondeformstheliquids
to produce a characteristic contact angle as shown in Figure 29.
A wetting phase is one which spreads over the solid surface and preferentially wets
thesolid.Thecontactangleapproacheszero(andwillalwaysbelessthan90).
A non-wetting phasehaslittleornoaffnityforasolidandthecontactanglewill
begreaterthan90
0

Oil
Water
Contact angle,

so
sw
Solid

wo

Interfacial tension, s, defined as force / unit length


Interfacial tension between the water and oil
Interfacial tension between the solid and water
Interfacial tension between the solid and oil

wo

so

sw
Figure 29 Interfacialtensionbetweenoil,waterandasolid
Thecontactangledescribesthenatureoftheinteractionofthefuidsonthesurface:
fortheoil-watersystemshownabove:ananglelessthan90indicatesthatthesurface
iswaterwet.Iftheangleweregreaterthan90thenthesurfacewouldbeoilwet.
The composition of the surface also affects the interfacial tension. Figure 30 shows the
effect of octane and napthenic acid on a water droplet on silica and calcite surfaces.
Thewaterisnotaffectedbythechangeinsurfaceinthewater/octanesystem,however,
thenapthenicacidcausesthewatertowetthesilicasurface,buttobenon-wetting
on the calcite surface.

Octane Napthenic acid
30
30
106
35
Silica
Calcite
Octane Napthenic acid
Figure 30 The effect of a change in the surface on wetting properties
TheAdhesiontension,Atisdefnedasthedifferencebetweenthesolidwaterand
solidoilinterfacialtension.Thisisequaltotheinterfacialtensionbetweenthewater
andoilmultipliedbythecosineofthecontactangle,
A
t
=
sw
-
so
=
wo
Cos
wo
Institute of Petroleum Engineering, Heriot-Watt University
1
fundamental Properties of Reservoir Rocks
Ifa containerofoiland wateris consideredasinfgure 31, thedenserwaterlies
below the oil.

cos
h

radius, r
OIL
.c
Water
Figure 31 Capillary rise in an oil/water system
Ifaglasscapillarytubeofradius,risinsertedsuchthatitpiercestheinterfacebetween
the oil and water, the geometry of the tube and the imbalance in forces produced
betweentheglass,oilandwatercausetheinterfacetobepulledupwardsintothetube.
Ifnonwettingfuidswereused,theinterfaceinthetubemaybepusheddownwards.
Underequilibriumconditions,i.e.afterthetubehaspiercedtheoriginalinterface,
theadhesiontensionaroundtheperiphery(2r) of the tube can be summed to give
thetotalforceupwards.Sincetheinterfaceisstatic,thisforcemustbebalancedby
theforcesinthecolumnofwaterdrawnupthetubeandtheequivalentcolumnofoil
outsidethetube,i.e.atpointC,theforce(orpressure)mustbethesameinthetube
asoutside,thereforetheexcessforceproducedbythecolumnofwaterisbalanced
by the adhesion tension.
net force upwards = 2r
wo
Cos (16)
netforcedownwards=(
w
gh -
o
gh)r
2
=gh(
w
-
o
)r
2
(17)
theinterfaceisatequilibrium,therefore
2r
wo
Cos=gh(
w
-
o
)r
2
(18)
The capillary pressureisthedifferenceinpressureacrossaninterface,thereforein
termsofpressure(theP
c
,forceactingonareapr
2
)


gh( ) r
r
2 r Cos
r
P
gh( ) =
2 Cos
r
w o
2
2
wo
2 c
w o
wo

Itcanbeseenfromtheequations,capillarypressurecanbedefnedbothintermsof
curvatureandintermsofinterfacialtension,asexpressedbythehydrostatichead.

P
2 Cos
r
c
c


gh p p
w o
( )
(19)
where
P
c
= capillary pressure
= surface tension
= contact angle
r
c
= radius of the tube
h = height of interface

w
= the density of water

o
= the density of oil.
Foradistributionofcapillaries,therefore,thecapillarypressurewillgiverisetoa
distributionofingressofwettingfuidintothecapillaries.Therelativepositionofthe
capillary rise is given with respect to the free water level, FWL, i.e. the point of zero
capillary pressure. Figure 32 illustrates the effect of three different capillary radii on
the rise of water. Figure 33 shows the behaviour for a full assembly of capillaries
andalongsidetheassociatedcapillarypressurecurve.Inthisfgureitisimportant
tonotefveaspects.
The free water level-the position of zero capillary pressure
The oil -water contact
The100%watersaturationatadistanceabovethefree-waterleveldueto
the capillary action of the largest tube.
The irreducible level representing the limit if mobile water saturation
The different radii segregate the capillary pressure and therefore the height to
which the water is drawn into the oil zone.
The zone of varying water saturation with height above the 100% free water oil
contact is called the transition zone.
The formation containing irreducible water will produce only hydrocarbons whereas the
transition zone of varying water saturation will produce water and hydrocarbons.
The shape of the capillary pressure curves in the transition zone will depend on the
nature of the rock.
Institute of Petroleum Engineering, Heriot-Watt University

fundamental Properties of Reservoir Rocks



oil
oil
oil
oil

h
FREE WATER
LEVEL
WATER WATER
Figure 32 Capillary Rise in Distribution of Capillaries

Water
0%
100%
Sw
So
100%
0%
Oil water contact
Oil
Water
OWC
Pc
Free water level
0
FWL
I
r
r
e
d
u
c
i
b
l
e

W
a
t
e
r
T
r
a
n
s
i
t
i
o
n

Z
o
n
e
Figure 33 Capillary Pressure Curve
It must be remembered that although concepts of capillary pressure were formulated
intermsoffneboretubes,applicationinpracticedealswithacomplexnetworkof
interconnectedporesinamatrixcarryingsurfacechemicalpropertiesasillustrated
infgure1oftheporecastoftheporespace.
Theheightatwhichawettingliquidwillstandaboveafreelevelisdirectlyproportional
to capillary pressure which is related to the size and size distribution of the pores.
It is also proportional to interfacial tension and the cosine of the contact angle and

inverselyproportionaltothetuberadiusanddifferenceinfuiddensity.Thesmaller
theporesie.thelowerthepermeability,thehigherthecapillarypressure.
7.2 Fluid distribution in reservoir rocks
Water is retained by capillary forces as hydrocarbons accumulate in productive
reservoirs. The water is referred to as connate or interstitial water and in water wet
rocksitcoatstherocksurfacesandoccupiesthesmallestpores,whereashydrocarbons
occupy the centre of the larger pores. The magnitude of the water saturation retained is
proportionaltothecapillarypressurewhichiscontrolledbytherockfuidsystem.

~
S
w
P
c =
2

Cos
r
e
Rock Fluid
Property
Wettability
Rock / Fluid Property
Rock Property
(Permeability and Porosity)
_
Waterwet,coarsegrainedsandandooliticandvuggycarbonateswithlargepores
have low capillary pressure and low interstitial water contents. Silty, fne grained
sands have high capillary pressures and high water contents.
Reservoir saturation reduces with increased height above the hydrocarbon-water
contact.At the base of the reservoir there will usually be a zone of 100% water
saturated rock. The upper limit of this is referred to as the water table or water
oilcontact(WOC).However,thereisanonidentifablelevel,thefreewaterlevel
representing the position of zero capillary pressure.
Figure 34 shows the capillary pressure curve for a reservoir where the water saturation
reducesabovetheaquifer.The100%watersaturationcontinuessomedistanceabove
thefreewaterlevelcorrespondingtothelargestporesoftherock,h
D
. Above this level
both the oil and water are present and the reservoir water saturation decreases with
increasedheightabovethehydrocarbonwatercontact,sincethelargerporescanno
longer support the water by capillary action and the water saturation falls. Between
the100%WOCandtheirreduciblesaturationlevelistermedthetransition zone.
Institute of Petroleum Engineering, Heriot-Watt University

fundamental Properties of Reservoir Rocks



Oil
Sand
Grain
Pc
Water
WOC
FWL
0% Water Saturation 100%
Transition Zone
h
p
h
Figure 34 Capillary Pressure Curve for Porous media
Considerthecapillarypressurecurvesforthetworocksinfgure35.Thefrstsample
(case1)hasasmallrangeofconnectingporesizes.Thesecondsample(case2)hasa
muchlargerrangeofconnectingporesizes,althoughthelargestporesareofsimilar
sizeinbothcases.Also,incase2,theirreduciblewatersaturationisreachedatlow
capillarypressure,butwiththegradedsystem,amuchlargercapillarypressureis
needed.


h
Case 1
Case 2
High Pc needed to reach limiting
water saturation.
Irreducible (or non - communicating)
water approach at low Pc
Largest connecting pores
about the same size.
Therefore simular h
D
h
D
Water saturation
Irreducible
water saturation
100%
h
I
P
c

=

(
P
w

-

P
o
)

g
h
X
Figure 35 Capillary Pressure Curves for Different Rocks
Inadditiontowatertransitionzones,therecanalsobeanoil/gastransitionzone,but
thisisusuallylesswelldefned.
Rockwettabilityinfuencesthecapillarypressureandhencetheretentiveproperties
oftheformation.Oilwetrockshaveareducedornegligibletransitionzone,andmay
containlowerirreduciblesaturations.Lowfuidinterfacialtensionreducesthetransition
zone,whilehighinterfacialtensionextendsit.Figure36illustratesthiseffect.

0 100
A
High Interfacial Tension
Low Interfacial Tension H
e
i
g
h
t

A
b
o
v
e

W
a
t
e
r

L
e
v
e
l
Water Saturation: Percent Pore Space
Interfacial Tension Effect
Figure 36 Interfacial Tension Effect
Saturation history infuences the capillary pressure water saturation relationship
and therefore the size of the transition zone. Drainage saturation results from the
drainageofthewettingphase(water)fromtherockasthehydrocarbonsaccumulate.
Itrepresentsthesaturationdistributionwhichexistsbeforefuidproduction.The
level of saturation is dictated by the capillary pressure associated with the narrow
pore and is able to maintain water saturation in the large pore below. Imbibition
Institute of Petroleum Engineering, Heriot-Watt University

fundamental Properties of Reservoir Rocks


saturationresultsfromtheincreaseinthewettingphase(water)andtheexpulsionof
the hydrocarbons. In this case the saturation is determined by the large pore reducing
the capillary pressure effect and preventing water entering the larger pore. This is
the situation which occurs both when natural water drive imbibes into the formation
raisingthewatertablelevelandinwaterinjectionprocesses.Clearlythetwosaturation
historiesgeneratedifferentsaturationheightprofles.Figure37showsthedrainage
and imbibition effects on capillary rise.

0 100
A
Drainage
Imbibition H
e
i
g
h
t

A
b
o
v
e

W
a
t
e
r

L
e
v
e
l
Water Saturation: Percent Pore Space Drainage Imbibition
Figure 37 Saturation History Effect
A large density differencebetweenwaterandhydrocarbons(water-gas)suppresses
thetransitionzone.Conversely,asmalldensitydifference(water-heavyoil)increases
the transition zone. Figure 38 shows the differences in density for water/heavy oil and
water/gasoncapillaryrise.Transitionzonesbetweenoilandgasarenotsignifcant
because of the large density difference between oil and gas.

0 100
A
Small Density Difference
(Water-Heavy Oil)
Large Density Difference
(Water Gas)
H
e
i
g
h
t

A
b
o
v
e

W
a
t
e
r

L
e
v
e
l
Water Saturation: Percent Pore Space Fluid Density Difference Effect
Figure 38 Fluid Density Effect
7.3. Impact of Layered Reservoirs
A characteristic of reservoirs is the various rock types making up the reservoir section.
Each rock type has its own capillary pressure characteristics. Wells penetrating such
formationswillshowawatersaturationdistributionrefectingthespecifccapillary
effectsofeachformationtype.Insomecasesa100%watersaturationwillbeabove
a lower water saturation associated with a lower elevation material with a higher
permeability,Figure39.

ForexamplewellAwouldonlyindicate100%water.WellBwouldpenetratethe
transitionzoneofthetoplayerthenaregionof100%watersaturation.Thesaturation
proflesforwellBandCareillustratedinfgure39.Thetransitionzoneofthenext
layer2,followedbyaninterfacialof100%saturationassociatedwithlayers2,3and
4theninto100%forthenexttwolayers.WellDpenetratesthroughthetopandnext
layerattheirreduciblesaturationlevel,intothetransitionzoneforlayerthree,then
into irreducible saturation for the 4th layer.

1
2
3
4
Transition
zone
Water saturation
profile well C only Water saturation
profile
Well B only
0
=
1
5
K
=
4
0
m
d



0
=
2
5
K
=
1
9
0
m
d

0
=
1
0
K
=
5
m
d
0
=
3
0
K
=
2
0
0
m
d
A B C D
SHALE
SHALE
S
A
N
D
S
T
O
N
E

R
E
S
.

Free Water Level
100% Water Level
FWL FWL
H
e
i
g
h
t
0% Sw 100%
0 100%
Figure 39 CapillaryEffectsinStratifedFormations
8 EffECTIVE PERMEABILITy
8.1Defnition
The idea of relative permeability provides an extension to Darcys Law to the
presenceandfowofmorethanasinglefuidwithintheporespace.Whentwoor
moreimmisciblefuidsarepresentintheporespacetheirfowsinterfere.Specifc
or absolute permeabilityreferstopermeabilitywhenonefuidispresentat100%
saturation. Effective permeabilityrefectstheabilityofaporousmediumtopermit
thepassageofafuidunderapotentialgradientwhentwoorthreefuidsarepresent
intheporespace.Theeffectivepermeabilityforeachfuidislessthantheabsolute
permeability. For a given rock the effective permeability is the conductivity of each
phaseataspecifcsaturation.Aswellastheindividualeffectivepermeabilitiesbeing
lessthanthespecifcpermeability,theirsumisalsolower.
Institute of Petroleum Engineering, Heriot-Watt University

fundamental Properties of Reservoir Rocks


If measurements are made on two cores having different absolute permeabilities
k
1
and k
2
,thereisnodirectwayofcomparingtheeffectivepermeabilityk
w
and k
o

curves since for the two cores they start at different points k
1
and k
2
.Thisdiffculty
is resolved by plotting the relative permeability k
rw
and k
ro
where
Relative Permeability =

permeability to one phase when one or more phases are present
permeability to one phase alone
k
k
k
r
e

Relative permeability is dimensionless and is reported as a fraction or percentage.


On relative permeability plots the curves start from unity in each case, so direct
comparisons can be made.
A typical set of effective permeability curves for an oil water system is shown in
fgure40andforagasoilsysteminfgure41.

1.0
0.9
0.8
0.7
0.6
0.5
0.4
0.3
0.2
0.1
0
1.0 0.9 0.8 0.7 0.6 0.5 0.4 0.3 0.2 0.1 0
S , Water Saturation, Fraction W
R
e
l
a
t
i
v
e

P
e
r
m
e
a
b
i
l
i
t
y
k
ro
k
rw
Figure 40 Relative permeability curves for water-oil sysrem
0

1.0
0.9
0.8
0.7
0.6
0.5
0.4
0.3
0.2
0.1
0
1.0 0.9 0.8 0.7 0.6 0.5 0.4 0.3 0.2 0.1 0
Liquid Saturation = S + S , %
WO O
R
e
l
a
t
i
v
e

P
e
r
m
e
a
b
i
l
i
t
y
,

F
r
a
c
t
i
o
n

o
f

A
b
s
o
l
u
t
e
C
o
n
n
a
t
e

W
a
t
e
r

p
l
u
s

R
e
s
i
d
u
a
l

O
i
l

S
a
t
u
r
a
t
i
o
n
k
rg
k
ro
Figure 41 Relative permeability curves for gas-oil sysrem
Thefollowingpointsaretobenoted:
Theintroductionofasecondphasedecreasestherelativepermeabilityofthefrst
phase:forexample,

k
or
drops as

S
w
increasesfromzero.Secondly,atthepointwhere
the relative permeability of a phase becomes zero there is still a considerable saturation
of the phase remaining in the rock. The value of

So at

k
ro
= 0 is called the residual oil
saturation and the value of

S
w
at

k
rw
= 0 is called the irreducible water saturation.
The shapes of the relative permeability curves are also characteristic of the wetting
qualitiesofthetwofuids(fgure42).Whenawaterandoilareconsideredtogether,
waterisalmostalwaysthewettingphase.Thismeansthatthewater,orwettingphase,
wouldoccupythesmallestporeswhilethenon-wettingphase,oroilphase,would
occupy the largest pores. This causes the shape of the relative permeability curves
for the wetting and non-wetting phase to be different.
Institute of Petroleum Engineering, Heriot-Watt University
1
fundamental Properties of Reservoir Rocks

K
rw
100
90
80
70
60
50
40
30
20
10
0
100 90 80 70 60 50 40 30 20 10 0
Water Saturation, S
W
R
e
l
a
t
i
v
e

P
e
r
m
e
a
b
i
l
i
t
y
,

%
K
ro
Water-Wet Drainage
Water-Wet Imbibition
Oil-Wet Drainage
(Decreasing Sw )
(Increasing Sw )
(Increasing Sw )
Figure 42 Oil and Water Relative Permeability Curves for Water-Wet and Oil-Wet
Systems(CoreLaboratoriesInc)
This is illustrated by looking at the relative permeability to one phase at the irreducible
saturation of the other phase. The relative permeability to water at an irreducible oil
saturationof10%(90%water)isabout0.6,fgure40,whereastherelativepermeability
tothenon-wettingphase,oil,attheirreduciblewatersaturationof0.3approaches1.0.
In this case it is 0.95. One practical effect of this observation is that it is normally
assumed that the effective permeability of the non-wetting phase in the presence of
anirreduciblesaturationofthewettingphaseisequaltotheabsolutepermeability.
Consequently,oilfowinginthepresenceofconnatewateroranirreduciblewater
saturation is assumed to have a permeability equal to the absolute permeability.
Similarly,gasfowinginareservoirinthepresenceofirreduciblewatersaturation
isassumedtohaveapermeabilityequaltotheabsolutepermeability.
Relative permeability characteristics are important in the displacement of hydrocarbons
bywater,andinthedisplacementofoilandwaterbygas.Suchdisplacementsoccur
duringprimaryandsecondaryrecoveryoperations,aswellasduringcoringandcore
recovery.
Relative permeability data when presented in graphical form are often referred to as
drainage or imbibition curves.(fgure42)
Imbibition relative permeability is displacement where the wetting phase saturation
isincreasing.Forexample,inawaterfoodofawaterwetrock,orcoringwitha
water base mud.

Drainage relative permeability is where the non-wetting phase saturation is increasing.


Forexample,gasexpulsionofoilduringprimarydepletionorgasexpansionoffuids
duringcorerecovery,andtheconditionexistinginthetransitionzoneatdiscovery.
Water displacement of oil differs from gas displacement of oil since water normally
wets the rock and gas does not. The wetting difference results in different relative
permeability curves for the two displacements.
8.2 Water displacement of oil
Priortowaterdisplacementfromanoilproductivesandinterstitialwaterexistsas
a thin flm around each sand grain with oil flling the remaining pore space. The
presence of water as previously stated has little effect on the fow of oil, and oil
relativepermeabilityapproaches100%.Waterrelativepermeabilityiszero.
Waterinvasionresultsinwaterfowthroughbothlargeandsmallporesasthewater
saturation increases. Imbibition relative permeability characteristics infuence the
displacement. Oil saturation decreases with a corresponding decrease in oil relative
permeability. Water relative permeability increases as water saturation increases.
Oilremainingafterfood-outexistsastrappedglobulesandisreferredtoasresidual
oil. This residual oil is immobile and the relative permeability to oil is zero. Relative
permeabilitytowaterreachesamaximumvalue,butislessthanthespecifcpermeability
becausetheresidualoilisinthecentreoftheporesandimpedeswaterfow.
8.2.1 Water-oil relative permeability
Accumulation of hydrocarbons is represented by drainage relative permeability curves
asthewatersaturationdecreasesfrom100%toirreducible.Waterrelativepermeability
reduceslikewisefrom100%tozerowhileoilrelativepermeabilityincreases.
Subsequentintroductionofwaterduringcoringorwaterfoodingresultsinadifferent
set of relative permeability curves - these are the imbibition curves. The water curve
is essentially the same in strongly water wet rock for both drainage and imbibition.
The oil phase relative permeability is less during imbibition than during drainage.
The oil remaining immobile after a waterfood is infuenced signifcantly by the
capillary pressure and interfacial tension effects of the system. It is of note that a
high residual oil saturation is a result of the oil ganglia being retained in the large
pores as a result of capillary forces. Figure 43 illustrates the pore doublet model
illustrating how oil can be trapped in a large pore. The forces to displace this droplet
have to overcome capillary forces and are too great to use pressure through pumping.
Theforcerequiredcanbereducedbyreducingtheinterfacialtensionwhichisthe
basisformanyenhancedoilrecoverymethods;forexample,surfactantandmiscible
fooding.
Institute of Petroleum Engineering, Heriot-Watt University

fundamental Properties of Reservoir Rocks



Trapped oil
Water penetrating
smaller pores due to
capillary forces
Advancing water
Water In Oil
Water In Oil
Water In
Water
Figure 43 Pore Doublet Model
An important perspective in a displacement process is the concept of mobility ratio.
Thisrelatesthemobilityofthedisplacingfuidrelativetothatofthedisplacedfuid.
ItisthereforearatioofDarcysLawforeachrespectivefuidattheresidualsaturation
oftheotherfuid.Inthecontextofwaterdisplacingoil.

M = mobility ratio =
k
k
rw w
ro o
/
/

(20)
where k
rw
is the relative permeability at residual oil saturation


k
ro
is the relative permeability at the irreducible water saturation.
These relative permeabilities are sometimes referred to as end point relative
permeabilities. When M is less than 1 this gives a stable displacement whereas when
M is greater then 1 unstable displacement occurs.

Thistopiciscoveredextensivelyinthechapteronimmiscibledisplacement
8.3 Gas displacement of oil and gas-oil relative permeability
Gas is a non-wetting phase and it initially follows the path of least resistance through
thelargestpores.Gaspermeabilityiszerountilacriticalorequilibriumsaturation
isreached(fgure41).
Gassaturationlessthanthecriticalvalueisnotmobilebutitimpedesthefowof
oil and reduces oil relative permeability. Successively smaller pore channels are
invadedbygasandjoinedtoformothercontinuouschannels.Thepreferenceofgas
for larger pores causes a more rapid decrease of oil relative permeability than when
water displaces oil from a water wet system. Figure 44 shows the alteration of relative
permeabilityasgascomesoutofsolutionandfowsatincreasingsaturationthrough
theoilreservoir.These gas/oil relative permeabilitycurves arevery signifcantin
relation to the drive mechanism of solution gas drive, which we will discuss in a
subsequentchapter.
Institute of Petroleum Engineering, Heriot-Watt University

fundamental Properties of Reservoir Rocks


C
h
a
r
a
c
t
e
r
i
s
t
i
c

S
a
n
d

D
u
r
i
n
g

O
i
l

D
i
s
p
l
a
c
e
m
e
n
t
b
y

G
a
s

@

5
%

G
a
s

s
a
t
u
r
a
t
i
o
n
C
h
a
r
a
c
t
e
r
i
s
t
i
c

S
a
n
d

D
u
r
i
n
g

O
i
l

D
i
s
p
l
a
c
e
m
e
n
t
b
y

G
a
s

@

2
0
%

G
a
s

s
a
t
u
r
a
t
i
o
n
C
h
a
r
a
c
t
e
r
i
s
t
i
c

S
a
n
d

D
u
r
i
n
g

O
i
l

D
i
s
p
l
a
c
e
m
e
n
t
b
y

G
a
s

@

4
5
%

G
a
s

s
a
t
u
r
a
t
i
o
n
1
0
0
1
0
0
8
0
8
0
6
0
6
0
4
0
4
0
2
0
2
0
0
0
R e l a t i v e P e r m e a b i l i t y : P e r c e n t
G
a
s

S
a
t
u
r
a
t
i
o
n
:

P
e
r
c
e
n
t

P
o
r
e

S
p
a
c
e
K
r
o
K
r
g
G
a
s

S
a
t
u
r
a
t
i
o
n
:

5
%

o
f

P
o
r
e

S
p
a
c
e
S
p
e
c
i
f
i
c


P
e
r
m
e
a
b
i
l
i
t
y


(
K
s
)
:


2
5
0

m
d
.
E
f
f
e
c
t
i
v
e

P
e
r
m
e
a
b
i
l
i
t
y

t
o

O
i
l


(
K
o
)
:

1
8
3

m
d
.
E
f
f
e
c
t
i
v
e

P
e
r
m
e
a
b
i
l
i
t
y

t
o

G
a
s

(
K
g
)
:


0
.
0
m
d
.
R
e
l
a
t
i
v
e


P
e
r
m
e
a
b
i
l
i
t
y

t
o

O
i
l


(
K
r
o
)

=

1
8
3
/
2
5
0

=

0
.
7
3
R
e
l
a
t
i
v
e


P
e
r
m
e
a
b
i
l
i
t
y

t
o

G
a
s

(
K
r
g
)

=

0
.
0
/
2
5
0


=

0
.
0
K
r
o
K
r
g
G
a
s

S
a
t
u
r
a
t
i
o
n
:

2
0
%

o
f

P
o
r
e

S
p
a
c
e
S
p
e
c
i
f
i
c


P
e
r
m
e
a
b
i
l
i
t
y


(
K
s
)
:


2
5
0

m
d
.
E
f
f
e
c
t
i
v
e

P
e
r
m
e
a
b
i
l
i
t
y

t
o

O
i
l


(
K
o
)
:

5
2

m
d
.
E
f
f
e
c
t
i
v
e

P
e
r
m
e
a
b
i
l
i
t
y

t
o

G
a
s

(
K
g
)
:


1
0
m
d
.
R
e
l
a
t
i
v
e


P
e
r
m
e
a
b
i
l
i
t
y

t
o

O
i
l


(
K
r
o
)

=

5
2
/
2
5
0


=

0
.
2
1
R
e
l
a
t
i
v
e


P
e
r
m
e
a
b
i
l
i
t
y

t
o

G
a
s

(
K
r
g
)

=

1
0
/
2
5
0


=

0
.
0
4
1
0
0
1
0
0
8
0
8
0
6
0
6
0
4
0
4
0
2
0
2
0
0
0
R e l a t i v e P e r m e a b i l i t y : P e r c e n t
G
a
s

S
a
t
u
r
a
t
i
o
n
:

P
e
r
c
e
n
t

P
o
r
e

S
p
a
c
e
O
i
l
W
a
t
e
r
G
a
s
G
a
s

S
a
t
u
r
a
t
i
o
n
:

4
5
%

o
f

P
o
r
e

S
p
a
c
e
S
p
e
c
i
f
i
c


P
e
r
m
e
a
b
i
l
i
t
y


(
K
s
)
:


2
5
0

m
d
.
E
f
f
e
c
t
i
v
e

P
e
r
m
e
a
b
i
l
i
t
y

t
o

O
i
l


(
K
o
)
:

6
.
2

m
d
.
E
f
f
e
c
t
i
v
e

P
e
r
m
e
a
b
i
l
i
t
y

t
o

G
a
s

(
K
g
)
:


7
0
m
d
.
R
e
l
a
t
i
v
e


P
e
r
m
e
a
b
i
l
i
t
y

t
o

O
i
l


(
K
r
o
)

=

6
.
2
/
2
5
0


=

0
.
0
2
5
R
e
l
a
t
i
v
e


P
e
r
m
e
a
b
i
l
i
t
y

t
o

G
a
s

(
K
r
g
)

=

7
0
/
2
5
0


=

0
.
2
8
1
0
0
1
0
0
8
0
8
0
6
0
6
0
4
0
4
0
2
0
2
0
0
0
R e l a t i v e P e r m e a b i l i t y : P e r c e n t
G
a
s

S
a
t
u
r
a
t
i
o
n
:

P
e
r
c
e
n
t

P
o
r
e

S
p
a
c
e
K
r
o
K
r
g
Figure 44 GasOilRelativePermeabilities(CoreLab)
CONTENTS
1. INTRODUCTION
1.1 Core Analysis
1.2 CoreDefnitions
2. SAMPLE PREPARATION
2.1 Whole Core Scanning
2.2 Core Cleaning
3. POROSITY MEASUREMENTS
3.1 Methods
3.2 Wholecoreversusconventionalversus
sidewallsamples
4. PERMEABILITY
4.1 Introduction
4.2 ImpactofStress
4.3 SteadyStatePermeabilityMethods
4.4 UnsteadyStatePermeabilityMeasurements
5. FLUID SATURATION
5.1 Gassaturation
5.2 Oilsaturationbyretort
5.3 Watersaturation
6. CAPILLARY PRESSURE
6.1 Introduction
6.2 CapillaryPressureMeasurementTechniques
6.2.1 PorousDiaphragm (fgure22)
6.2.2 Centrifugemethod(Figure23)
6.2.3 Dynamicmethod(Figure24)
6.2.4 MercuryInjection(Figure25)
6.3 UseofLaboratoryCapillaryPressureData
forReservoir
6.4 Averagingcapillarypressuredata
7. EFFECTIVE PERMEABILITY
Rock Properties Measurement

LEARNING OBJECTIVES
Having worked through this chapter the Student will be able to:
Listthevarioustypesofrecoveredcore.
Describebriefythevariousmethodsofmeasuringporosityandpermeability.
Briefy describe the various stress conditions that can be imposed on a rock
sample.
Understandhowtoconvertlaboratorybasedcapillarypressuremeasurement
datatofeldrelatedvaluesofcapillarypressure.
Beabletodeterminethesaturationdistributioninawellmadeupofdifferent
rocktypesgivencapillarypressuredata.
DerivetheLeverettJfunctionandbeawareofthemajortortuosityrelatedassumption
initsderivation.
Institute of Petroleum Engineering, Heriot-Watt University

Rock Properties Measurement


1. INTRODUCTION
1.1 Core Analysis
In this chapter we will focus on the laboratory based methods used to determine
someoftheparametersoutlinedinthepreviouschapter.Thetopicisalsocovered
inothermodulesoftheoverallPetroleumEngineeringprogrammeinthecontextof
thespecifcmodule.Corerecoveryiscoveredindrillingandrockpropertiesarealso
coveredinthePetrophysicsmodule.
Cores obtained from the reservoir formation contain a considerable amount of
informationaboutthenatureoftherocksthemselvesandvariousproperties.They
arealsoasourceofmaterialforinvestigatingrockbehaviourwithrespecttofuid
displacementanditsreactiontovariousfuidtypes.
Coresarerecoveredfromtheformationofinterestusinganannularshapedcoring
bit.Theintegrityoftherecoveredcoredependsonthenatureoftherockandcan
varyfromrockwhichiswellformedtothatwhichisfriableincharacterorevenis
sounconsolidatedthatitwouldformapileofsandontherigfoorwhenrecovered
fromthecorebarrel.Thecorefromthecorebarrelprovidesarecord,overthewell
sectionrecovered,ofthepropertiesoftheformation.Figure1illustratesthewide
rangeofmeasurementsandprocedurescarriedoutoncoresamples
1
.
A comprehensive document on the procedures for generating some of the rock
propertiesthroughlaboratorymeasurementistheAPIRecommendedPracticesfor
Core Analysis
2
.APRRP40whichwasrevisedin1998.ThisAPIdocumentgoes
intodetailbeyondthatcoveredinthisoverviewchapter

Government or
Regulatory Board
Sampling
Curation
Slabbed Core
Photograph
Sedimentology
Lithology
Samples
Thin Sections
Detail Pore Structure
Diagenesis
Porosity Type
Environmental
Evidence
Small Samples
Grain Size Distribution
Mineral Analysis
X-Ray and SEM
Analysis
Bio-Dating and
Association
Routine Core Plug
Analysis
Porosity
Permeability
Grain Density
As-Received
Saturations
Special Core Analysis
Preserved /Restored State
Capillary Pressure
Relative Permeabilty
Electrical Properties
Acoustic Properties
Compressive Properties
Clay Chemistry Effects
Specific Tests
Calbration of Wireline Logs
Figure 1 DataObtainedFromCoredWells
1
.

Ascoveredinthepreviouschapterthereareanumberofpropertiesinrelationto
measurementspossibleonthecoresasshowninthefgure1.Incoreanalysisthe
measurements can be divided into two parts; routine measurements which cover;
fuid saturations, porosity and permeability; special core analysis which covers a
widerangeofmeasurementsandspecialtestsofspecialinteresttotheorganisation
commissioningthetesting.Inthischapterwewillfocusonroutinecoreanalysis
andalsocoverbriefycapillarypressuremeasurements.
1.2 Core defnitions
Beforeexaminingsomeofthemethodsitisimportanttodefnethevariouscoretypes
usedinexaminingrockpropertiesandtheirreactiontothetransmissionoffuids.
ThesedefnitionscomefromtheAPIrecommendedRP40
2
.
Fresh Core
Anynewlyrecoveredcorematerialpreservedasquicklyaspossibleatthewellsite
topreventevaporativelossesandexposuretooxygen.Thefuidtypeusedforcoring
shouldbenoted,e.g.,freshstate(oil-baseddrillingfuid),freshstate(water-based
drillingfuid).
Preserved Core.
Similartofreshcorebutsomeperiodofstorageisimplied.Preservedcoreisprotected
fromalterationbyanumberoftechniques,fromsimplemechanicalstabilisationusing
bubblewraporsimilar,freezingthecoretolockinfuidswhichwouldotherwise
evaporate(inthiscasethefreezingmayaltersomeoftherockproperties),enclosure
inheat-sealableplasticlaminates,anddipsandcoatings.
Cleaned Core.
Corefromwhichthefuidshavebeenremovedbysolvents.Thecleaningprocess
(thespecifcationandsequenceofsolvents,temperatures,etc)shouldbespecifed.
Somesolventscoulddamagethefabricoftherockandspecialcleaningprocedures
likecriticalpointdryingmightberequiredforexamplewithrockscontainingfriable
clays(fgure2).

Figure 2 Sandstonecontainsillite.
Institute of Petroleum Engineering, Heriot-Watt University

Rock Properties Measurement


Restored - State Core
Thisiscorethathasbeencleanedandthenreexposedtoreservoirfuidswiththe
intention of reestablishing the reservoir wettability condition. The conditions of
exposuretothecrudeoil,especiallyinitialwatersaturation,temperatureandtime,
canallaffecttheultimatewettablity.
Pressure - Retained Core
Thisismaterialthathasbeenkept,sofaraspossible,atthepressureofthereservoir
inordertoavoidchangeinthefuidsaturationsduringtherecoveryprocess.
2. SAMPLE PREPARATION
2.1 Whole Core Scanning
Priortosubdivisionofthewholecoreforthevarioustypesofanalysisanumberof
procedurescantakeplacetorecordthecharacteristicsofthewholecoreandtorelate
ittoindirectdownholemeasurements.Thepurposeofthiscoreexaminationand
descriptionistorecogniselithological,depositional,structuralanddiageneticfeatures
of the whole core or slabbed core. Qualitative and quantitative core descriptions
provide the basis for routine core analysis sampling, facies analysis, and further
reservoirstudiessuchasreservoirqualityandsupplementarycoreanalysis.Besides
visualexaminationandgeneratingaphotographicrecord,thesetechniquesprovide
ameansofrelatingtodownholemeasurementsandtoidentifyfeaturesofthecore
whichmightotherwiseifundetectedgenerateunrepresentativedatainsubsequent
analysis.
Thefollowinganalysismightbecarriedoutonwholecore.Acoregammalog,anx-
rayanalysis,acomputertomographyCTscanandoranNuclearMagneticResonance
NMR Scan.
Withinarockarenaturallyoccurringgamma-rayemitterswhichcangiveameasurable
gamma-rayresponsethatcanberecordedwithdepth.Ifsuchameasurementcanbe
madeonthewholecoreinthelaboratorythiswholecorelaboratorybasedmeasurement
canbeusedasdepthchecktorelatetoopenholemeasurements.Figure3.


Conveyor Belt
Core
Lead Shield
Scintillometer
Recorder
Figure 3 Naturalgammascanonwholecore.(Corelab).
AnumberofX-Raytechniquescanbeusedwhichinclude,fuoroscopy,x-radiography
andcomputerisedtomography(CT)scanning.Inonemethodacontinuousanalysisis
whereanattenuatedx-raybeamdirectedthroughthecoreimpingesonafuorescent
screenandthecapturedimageisrecordedbyvideocamera.Inx-radiographythe
attenuationofthebeamiscapturedandrecordedonsensitiveflm.Inthisprocedure
thecoreisstationary.TheadvancesinCTscanninginmedicalapplicationshave
beenusedinCTscanningwheretheattenuatedbeamdirectedinmultipledirections
byarotatingbeamenablesareconstructionofdensityvariationswithinthecore.The
resolutionoftheimagedependsonthethicknessofthebeamandthesizeofpixel
usedtoconstructtheimage.AsketchofCTscanningandtheprincipalonwhichit
isbasedisshowninfgure4

Shield
Particle
or energy
detector.
Attenuated
beam
Narrow incident
beam
h
I
o
I
I = I
o
e
-h
is a function of bulk density and atomic number
Sample for
measurement
Figure 4(a) Computeraidtomographyonwholecore.Principalofattenuation.
Institute of Petroleum Engineering, Heriot-Watt University

Rock Properties Measurement



Reconstruction algorithm
in computer.
Intensity profiles
Rotating energy
source and detector
Figure 4(b) Reconstructedcrosssection.
ThemainbeneftofNuclearMagneticResonance,(NMR)imagingisthatitisused
toprovideareconstructionofthefuidswithinacore,basedonthefrequencyofthe
excitationenergyassociatedwithanudei.Thisexcitationenergyissuppliedbyan
oscillatingmagneticfeld.ThehighenergyattenuationassociatedwithCTscanning
doesnotenablethedistinctivedensityvariationsaspossiblewiththosefromNMR
scanning.
Thesescansareabletoidentifylocalisedvariationsinacorewhichifcapturedin
subsequentcoreanalysismeasurementscouldgiverisetoanomalousresults.
2.2 Core Cleaning
Samplepreparationisanimportantconsiderationincoreanalysis.Priortosamples
orplugsbeingusedforthedeterminationofporosityorpermeabilitytheymustbe
thoroughlycleanedtoextractalloftheoilandbrineandthenbeproperlydried,with
theexceptionofsaturationmeasurementsforthedeterminationofporosity.Thisis
generally carried through fushing, fowing or contacting with various solvents to
extracthydrocarbons,waterandbrine.
Solventextractionusingcentrifuge,SoxletandDeanStarkrefuxingsolventextractors
arecommonlyusedtoremovebothoilandbrine.Nostandardsolventsareusedand
organisationsusetheirownpreferences(fgure5).


Core plug
Measurement of
collected water
Figure 5 Porousdiaphragmcapillary-pressuresystem.
Careneedstobetakentodrythesamplesparticularlywhenhydrateablemineralsare
presentinthesamplethatbreakdownathightemperatures.Thedryingprocedure
iscriticalinthattheinterstitialwatermustberemovedwithnomineralalteration.
Humidity-controlledovensareusedwhendryingclaybearingsamplestomaintain
theproperstateofhydration.Criticalpointcandryingbeusedtoclearcorecontinuing
delicateclayslikeillite(seePhaseBehaviourchapter-section8.1).
3. POROSITY MEASUREMENTS
3.1 Methods
Figure6illustratesthemethodsusedforroutinedeterminationofporosity.
Institute of Petroleum Engineering, Heriot-Watt University

Rock Properties Measurement



Water Oil Gas
Pore Volume Determination
Grain Volume Determination
Porosity
Vacuum Gauge
Valve
Displacement Pump
Pressure Gauge
Gas Inlet Valve Outlet Valve
Mercury
Sight Glass
Core Sample
Micrometer Scale
Plunger
Sample in Place,
Stopcock Open
Washburn Porosimeter Kobe Porosimeter
Boyles Law Porosimeter
Sample
Chamber
Reference
Volume
Valve Valve
Pressure
Gauge
Resaturation
Figure 6 Porositymeasurementmethods(Corelab)
(a) Bulk Volume
Inallporositymethodsabulkcoresamplevolumehastobedeterminedandthismay
becarriedouteitherbydisplacementofliquidorbycalliperingashapedsampleand
computationbytheappropriateformula.Figure7showsthedisplacementmethod,
andfgure8showsamercurydisplacementpump.
10

Thermometer
Core plug
Adjustable fork
Reference mark
Mercury vessel
Weighted
base
Single pan
balance
+ 0.01 gm _
Figure 7 Archimedesmercuryimmersionapparatus(API)
2

Volume
read-out
Sample
chamber
Pressure
read-out
Displacement
plunger
Figure 8 Volumetricmercurydisplacementpump(API)
2
(b) Summation of fuids
This method involves the independent determination of oil, gas and pure water
volumesofafreshcoresample.Theoilandwatercanbeobtainedbyretort(Figure
9)andthegasbymercuryinjection.Theporevolumeisdeterminedbysummingthe
threeindependentvolumes.
Institute of Petroleum Engineering, Heriot-Watt University
11
Rock Properties Measurement

Temperature
Controller
Thermocouple
Insulated Oven
Water Bath
Water Inlet
Sample Cup
Condensing Tube
Receiving Tube
Screen
Heating Elements
Figure 9 Ovenretort(API)
2
(c) Gas transfer
(i)TheBoylesLawbasedporositydeterminationmethodinvolvesthecompression
ofagasintotheporespaceortheexpansionofgasfromtheporesofaprepared
sample.Dependingontheinstrumentationandtheprocedure,eitherporevolume
orgrainvolumecanbedetermined.Figure10showsatypicalsetupforthisand
isthemostcommonmethodformeasuringthegrainvolume.Itinvolvessettingup
apressureinaknownreferencevolumeandthenexpandingthepressureintothe
spacecontainingthesample.Withsuitablecalibrationthegrainvolumeisdetermined
usingtheidealgasrelationthatPV=constant.

Sample
chamber
Reference
volume
P P1
Pressure
regulator
Gas in
Figure 10 Boyle'slawporosimeter.
1
(ii)TheWashburn-Buntingmethodinvolvesthevacuumextractionandcollection
ofthegascontainedintheporesofapreparedsample.Themethodmeasurespore
volume.
(d) Liquid resaturation
The pores of a prepared sample are flled with a liquid of a known density. The
increase in weight of the sample divided by the fuid density is a measure of the
porevolume.
(e) Grain density
Totalporosityisdeterminedbythismethodascomparedwitheffectiveporosity.The
sampleisreducedtograinsizeafterthedryweightandbulkvolumearedetermined.
Grainvolumeisdeterminedandsubtractedfromthebulkvolumetoyieldthetotal
porevolume.
3.2 Whole core versus conventional versus sidewall samples
Aswellascoringusingacoringbitandcorebarrel,itisalsopossibletorecoversamples
oftheformationusingwirelinetools,thesearetermedsidewallcoring.Therearetwo
typesofsidewallcoringdevices.Oneisbasedonexplodingacoreplugshapedpiece
intotheformation.Clearlysamplesrecoveredbythistechniquemaybesuitablefor
mineraldescriptionbutarenotsosuitedtoporosityandpermeabilityanalysisasa
resultofthedamagegeneratedbytheexplosiveforceofthesamplingdevice.Sidewall
corerswhichcutintotheformationdonotsufferfromsuchmechanicaldamage.
Wholecoreporositiestendtobeslightlylowerthansmallplugsamplesincertain
rocktypes.Thewholecoreislikelytoincludetightermaterialthanwouldbeincluded
inamorecarefullysampledplug.
Forsampleswithmediumtohighporosity,sidewallandconventionalsamplesagree
withinoneortwopercent.Duringsidewallsamplinglowporosityhighlycemented
materialstendtoshatterandyieldvaluesgreaterthanthetrueporosity.
4. PERMEABILITY
4.1 Introduction
TheAPIrecommendedpracticeforthedeterminationofpermeabilityisalsodetailed
inAPIRP40whichisaconsiderableimprovementonAPIRP27.
Thereareessentiallytwoapproachestomeasuringthepermeability,the steady state
method wherethepressuredropforafxedfowrateismeasured,generallyagas,or
the unsteady state methodwherethefowinthetransientregemeismeasured.
Inthelattertherearetwotypesoftest,thepulse-decaymethodwheretwopressures
aresetupanddownstreamofthecontainedsample.Aslightincreaseintheupstream
pressureisimposedandthedecayofthispressurethroughthesampleismonitored.
The advent of very high speed data acquisition systems and accurate pressure
Institute of Petroleum Engineering, Heriot-Watt University
1
Rock Properties Measurement
transducershasmadeitpossibletomonitorthesetransientfowconditions.Theother
approachisthepressurefalloffmethodwherearelativelylowupstreampressureis
setandthedecayofthispressureismonitoredasitisreleasedthroughthecoreto
thedownstreamopentoatmosphere.
4.2 Impact of Stress
Overrecentyearstheimpactofreservoirstressesonrockpropertiesandthereforethe
interestinmeasuringrockpropertiesunderrealisticstresseshasgrowninparticular
inrelationtopermeability.Stresseffectsalsohaveanimpactonotherproperties
includedporosity.Indescribingthevariousapproachestopermeabilitymeasurement
wewillalsolookatvariousproceduresforimposingstressonthesamples.
In fgure 19 of the previous chapter we identifed the various stress directions in
the context of permeability measurement. Figure 11 illustrates the core recovered
fromaverticalwellandthenaturalstressesimposed.Itisimportanttodistinguish
thedifferentpossiblestressloadingsthatcanbeappliedtocoreplugsandalsothe
confgurationofthestressesinthenaturalstate.Inthenaturalstatethestressescan
beconsideredtoberesolvedinthreeprincipaldirections.Theverticaldirectionbeing
themajorprincipalstressandthetwohorizontaldirectionsthetwominorprincipal
stresses.Figure11a

Whole core
Core plug
for horizontal
k measurement
Inch
Formation
Core plug
for vertical
k measurement
Major
principal stress
Minor principal stresses
Figure 11 (a) Corerecoveredfromverticalwellandstressorientationinthereservoir.
Ifacoreplugisrecoveredfromawholecorerecoveredfromaverticalwellthenthe
stressorientationsinapermeabilitytestwouldbeasshowninthesketchbelow.Figure
11band11c.Thesefguresdemonstratethatforacylindricalhorizontalcoreplug
itisdiffculttoimposeadistinctivemajorprincipalstressonthecoreplugdifferent
fromoneoftheminorprincipalstresseswhereasforaverticalorientatedcoreplug
suchdistinctivestressescanbeapplied.
1

Major principal stress
Minor principal stress
Major principal stress
Minor principal stress
Minor
principal
stress
Major principal stress
Figure 11 (b) Stressorientationforhorizontalcoreplug.

Major principal stress
Minor principal stress
Major principal stress
Minor principal stress
Minor
principal
stress
Major principal stress
Figure 11 (c) Stressorientationfromverticalcoreplug
Inrequestingreservoirstressestobeappliedtocoreplugmeasurementsitisimportant
toexaminethatthestressesappliedactuallyrepresentthosewhichtherockwould
besubjectedtointheformation.Thevariousmodesofstressingarockareshown
infgure12a-d
Isostatic Stress.Figure12a.Underisostaticstressloading,equalstressisapplied
tothesampleinalldirections,andsamplestraincanoccuronallaxes.Excessive
porosityreductiontypicallyoccurswhentheimposedisostaticstressisequaltothe
verticalreservoirstress(i.e.,theoverburdenstress).
Institute of Petroleum Engineering, Heriot-Watt University
1
Rock Properties Measurement

1
A
L
D
D
L
Sample
Isostatic stress
Figure 12 (a) IsostaticStress
Triaxial Stress.Figure12b.Underthetruetriaxialstressconditions,unequalstress
isappliedtothethreemajoraxesofthesample.Inthegeneralcase,strainswillbe
differentoneachaxis.Typicallyacubeorrectangularprism-shapedsamplewill
beused.

L
1
L

2
Triaxial stress
Figure 12 (b) TriaxialStress
Biaxial Stress.Figure12c.Biaxialstressloadingconditionsareaspecialcaseoftriaxial
stressloading.Inthebiaxialstressloadingofacylinder,thestressparalleltothe
cylindersaxisisdifferentfromthestressappliedaroundthecylinderscircumference.
Strainscanoccurparalleltoboththeaxisanddiameterofthecylinder.

Sample
L
D
D
L

1
C
Biaxial stress
Figure 12 (c) BiaxialStress
1
Uniaxial Strain.Figure12d.Uniaxialstraincompressionisaspecialcaseofbiaxial
stressloading;thestressappliedtothecircumferenceisjustsuffcienttomaintain
thediameterconstantasthestressparalleltothecylinderaxisisincreased.Strain
occursonlyparalleltotheaxisofthecylinder.

D
L

L
Sample
Uniaxial stress
Figure 12 (d) UniaxialStress
4.3 Steady State Permeability Methods
The most conventional permeability measurement approach has been to use the
measurementofthepressuredropassociatedwithafxedfowrate.Todetermine
specifcpermeabilitynitrogenorairisusuallycausedtofowthroughapreparedsample
ofmeasureddimensions.Thepressuredifferentialandfowratesaremeasuredand
thepermeabilitycalculatedfromtheDarcyequation.Aschematicsetupisshown
inthesketchbelow.Figure13

+
_
Differential
Pressure.
Pressure
transducer
Pressure
regulator
End view showing
radial stress
q
r
@ P
r
, T
r
Sample holder
Flow meter
P
1
P

P
a
p
D
L
Figure 13 Schematicofsteadystatepermeabilitymeasurement
2
TheconfningofthecoreinthiscaseshowsaHasslertypecoreholderwheretheradial
stressislowandisappliedtoensurethatfowofgasdoesnotby-passthecore.
Institute of Petroleum Engineering, Heriot-Watt University
1
Rock Properties Measurement
Figure14showsahighpressurecoreholderdesignedtoimposereservoirstresses.
Theslideableinlettubeenablesthestrainofthestresscoretobetakenup.Thestress
loadingforthisarrangementisisostatic.

Outlet
Flow Tube
Rubber
Sleeve
Cylindrical
Core Plug
Slidable Inlet
Tube
End Plug End Plug
Cavity for
Hydraulic Oil
to Produce Confining
Stresses
Retaining
Ring
Inlet Port for
Confining Oil
Figure 14 Highpressurecoreholderforstresscondition,isostatic
2
Figure 15 shows a sophisticated core holder where a different axial stress can be
appliedcomparedtotheradialstress.Inthisarrangementtheendfacesofthecore
plugneedtobemachinedaccuratelytoensurethattheloadingoftheaxialstress
isdistributedoverthewholeface.Ifnotthecoreisliabletofragment.Thestress
loadingforthiscoreplugisbiaxial.
1

Port for Oil to Produce
Radial Confining Stress,
or Vacuum to Dilate Sleeve
Inlet Flow Port
Outlet
Ports
Port for Oil to Produce
Radial Confining Stress,
or Vacuum to Dilate Sleeve
Reach Rod, X
Core Plug
Rubber Sleeve
Cavity for
High Pressure
Nitrogen for
Axial Stress
Large Piston of Axial Stress
Intensifier
N

Figure 15 Highpressurecoreholderorbiaxialloading
1
.
Usingacoreplugremovedfromahorizontalwellcoreitispossibleusingbiaxial
stressloadingtosomewhatsimulatethestressconditions,byconsideringthetwominor
principalstressesasequal.Howeverusingbiaxialstressconditionsforaconventional
plugfromaverticalwellrecoveredcore,thenthestressconditionsimposeddonot
refectthoseintheformation.Theradialstressisacombinationofthemajorprincipal
stressandoneoftheminorprincipalstressesandintheequipmenttheseareequal.
Ifhowever,oneisinterestedinmeasuringtheverticalpermeabilityfromasample
extractedfromthewholecorethenbiaxialstressconditionswillrefectmorereadily
thereservoirstresscondition.
Arecentinnovationhasbeenthetruetriaxialcell
2
(Figure16).Inthisarrangement
aseriesofaxialtubesarehydraulicallypressuredbetweentheconfningrubbersleeve
ofthecoreandthecoreholderbody.Thisenablesastresspatterntobeestablished
torepresentamorerealisticstressconditionreservoirstressconditions.
Institute of Petroleum Engineering, Heriot-Watt University
1
Rock Properties Measurement

Platen
Threaded
end cap
Trapped tube
Core
Rubber
sleeve
Aluminium
cell body
Maximum principal stress

1
1
1

A A
Section AA
Hydraulically
pressured tubes
Face of
core plus
Figure 16 Truetrixialcell.
Althoughliquidscouldbeusedinpermeabilitymeasurementsitiscommontousea
gas.GaspermeabilitiesneedtobecorrectedfortheKlinkenbergeffectandreported
asequivalentliquidpermeabilities.
Thesamplesforanalysismaybeeithertheconsolidatedpieceusedfortheporosity
determination or another sample but clearly it must be extracted and cleaned to
ensurethatnowateroroilarepresent.Ifinterstitialwaterisverysalinethenitmay
benecessarytoremovesalt.
Anotherrecentinnovationhasbeentheprobepermeameter.Thesedeviceswere
initiallyinventedtomeettheneedforadevicetogiveindicationsofpermeabilityof
anoutcrop.Theapplicationofrockoutcropsasanaloguesofsubsurfaceformations
hasbeenveryvaluableindevelopinggeological/reservoirmodellingprocedures.The
0
examinationofthevariouslevelsofpermeabilitymeasurement,(upscaling),have
demonstratedthevalueofbeingabletomeasurethepermeabilityoverasmallarea
whichtheprobepermeameteraffords.Figure17showsanarrangementofatypical
probepermeameter.Aswellasbackpackmountedversionforuseinoutcropstudies
they can also be laboratory mounted and can automatically scan the permeability
variationsinaslabofrock.

Flow
meter
Pressure
transducer Pressure
regulators
r
i
r
o
Rock being
examined
Figure 17 Schematicofsteadystateprobepermeameter.
TheAPIRP40documentalsodescribesaradialsteady-stateapparatus,fgure18,
wherefowisfromtheoutertotheinnerradius.Inthissetupthepreparationisnot
easyandaxialstressesarenotbalancedbyradialstresses.
Institute of Petroleum Engineering, Heriot-Watt University
1
Rock Properties Measurement
r
e
P

L
P
1
r
w
Calibrated
Gas Burette
Rubber Gaskets
Springs
Regulators
Air Supply
M
e
r
c
u
r
y

M
a
n
o
m
e
t
e
r
P
i
s
t
o
n
Pivot Ball
Figure 18 Radialfowsteadystatepermeameter
2
.
4.4 Unsteady State Permeability Measurements
Theadventofhighspeedcomputersanddataacquisitionsystemshasenabledthe
applicationofunsteadystatepermeabilitymeasurements.Theprinciplesaresimilar
tothebehaviourofawellduringawelltestandtheanalysisofthepressuresduringthe
unsteadystatedrawdownorbuild-upperiod.Figure19givesaschematicofapressure-
falloffsystem.Anupstreamgasreservoirofdifferentvolumes,toaccommodatea
widerangeofpermeabilities,ispressuredandthenreleasedtoatmosphereviafow
throughthecore.Thepressurejustupstreamofthecoreisaccuratelymonitored.Full
detailsofthecalculationprocedurepresentedbyJonesaregivenintheAPIRP40
practisedocument
2
.


Fill Vent
Hydrostatic
confining
pressure
V
T
V
P
P
1
P
c
Figure 19 Schematicofpressure-fallofgaspermeameter
2
.
Inthepulsedecaymethodforpermeabilitymeasurementaconfgurationofequipment
isasshowninfgure20.Itconsistsofanupstreamanddownstreamreservoir.The
twogasreservoirsareflledtoapressure.Whenequilibriumisreachedwithallvalves
open,thejoiningvalvesareclosedandthepressureintheupstreamgasreservoiris
increasedby2-3%ofthepressuresetinthevessels.Thevalve1isthenopenedandthe
pressuretimebehaviourofthetransientfowbehaviourismonitored.Thisprocedure
lendsitselftoverylowpermeabilityvalues,0.1-millidarciesto0.01microdarcies.
CalculationproceduresarealsogivenintheAPIpractisedocument.

V
P
Valve 1
Valve
Fill/vac.
V

V
1
+
_
P

P
c

p
Figure 20 Pulsedecayapporatusaxialfowofgas.
5. FLUID SATURATION
Core analysis is sometimes used to measure the fuid saturations associated with
the core. Because of the large pressure variations between the reservoir and the
surface these saturations are not too representative of the values that would exist
intheformation,unlessprecautionshavebeentakentopreventevaporationduring
pressuredecline.Suchprecautionscouldbetheapplicationofpressurecoringwhere
thedownholepressureisheldinthecorebarrelasitisrecoveredtosurface.Atthe
Institute of Petroleum Engineering, Heriot-Watt University

Rock Properties Measurement


surfacepriortoreleasingthepressurethecoreinitscontainerisfrozen.Itisthen
slippedandstoredinafrozenstate.Duringcontrolledthawingofthecorethefuids
producedandretainedenabledownholesaturationtobeobtained.
5.1 Gas saturation
Conventionalandsidewallcoresampleshavegassaturationmeasuredbyinjecting
mercuryintothegasflledportionsofthepores.Thegasiscompressedintoasmall
volume or forced into solution in the liquids in the pores using a mercury pump.
Measurementofthevolumeofmercurypenetratedisameasureofthegascontent
ofthesample.
5.2 Oil saturation by retort
Oildistilledatatmosphericpressuregivesameasureoftheoilcontentoftheplug.
Thedistillateiscollectedinacalibratedreceiver.Temperaturesupto650
0
Careused
(Figure9).
5.3 Water saturation
Samples can have their water content determined by atmospheric distillation
concurrentlywiththeoilcontentdetermination.Adistinctionshouldbemadebetween
theporewaterandthewaterofhydrationorcrystallisation.
Watersaturationcanalsobemeasuredbyasolventrefuxingmethod(Dean-Stark)
(fgure20).Tolueneisthemostcommonlyusedsolvent.Theoilcontentofthesample
isobtainedbydifferenceoftheweightofthesamplebeforeandafterextractionand
dryinglesstheweightofthewaterremovedduringsolventextraction.

Core plug
Measurement of
collected water
Figure 21 DeanStarkApparatus

6. CAPILLARY PRESSURE
6.1 Introduction
Thegenerallaboratoryprocedureforcapillarypressurestosaturateacoresample
withawettingphaseandmeasurehowmuchwettingmeasurementphaseisdisplaced
fromthesamplewhenitissubjectedtosomegivenpressureofnon-wettingphase.
Displacementtakesplacewhentheoilornon-wettingphasejustexceedsthecapillary
pressurecorrespondingtothelargestpore.Inotherwordsthecapillaryforcewill
holdthewaterinthelargestporeuntiltheoilpressureislargerthanthecapillary
pressureofthelargestpore.
Thevolumeofthefuiddisplacedataparticularpressurealsorepresentsthepore
volumeofallporesofthatparticularsize.Oncethisporevolumehasbeendisplacedat
aparticularpressurethepressureisincreasedandthenewporevolumemeasured.
A plot of water volume displaced versus the displacement pressure will represent
aplotofthecapillarypressureversusthepercentageoftheporeswithacapillary
pressuregreaterthanthesubjectcapillarypressure.
Clearlyarockwhichcontainsavarietyofporesizeswillhaveacapillarypressure
curvewhichisnotdiscontinuousbutisasmoothcurve.
Sincecapillarypressure,

P
2 Cos
r
c
=

thecurvecanbecalibratedtorepresentporesizeversuspercentageofporeslessthan
thesubjectporesize.
6.2 Capillary Pressure Measurement Techniques
Therearefourmainmethodsforcapillarypressuremeasurement
(i) Desaturationordisplacementthroughaporousdiaphragm.
(ii) Centrifugeorcentrifugalmethod.
(iii) Dynamiccapillarypressuremethod.
(iv) Mercuryinjectionmethod.
6.2.1 Porous Diaphragm (fgure 22)
Intheporousdiaphragmmethodthereisapermeablemembraneofuniformpore
sizedistributioncontainingporesofsuchasizethattheselecteddisplacingfuidwill
notpenetratethediaphragmwhenthepressuresappliedtothedisplacingphaseare
below some selected maximum pressure of investigation. Pressure applied to the
assemblyisincreasedbysmallincrements.Thecoreisallowedtoapproachastate
ofstaticequilibriumateachpressurelevel.Thesaturationofthecoreiscalculated
ateachpointdefningthecapillarypressurecurve.Anycombinationoffuidscanbe
used:gas,oiland/orwater.
Thisprocedureisclosesttotheactualsaturationinthereservoirbutthemethodis
timeconsumingvaryingfrom10to40daysforasinglesample.
Institute of Petroleum Engineering, Heriot-Watt University

Rock Properties Measurement



Nitrogen Pressure
Neoprene Stopper
Saran Tube
Nickel-Plated
Spring
Core
Seal of
Red Oil
Scale of
Squared Paper
Kleenex Paper
Ultra-Fine
Fritted Glass
Disk
Crude Oil
Brine
Figure 22 Porousdiaphragmcapillary-pressuresystem.
6.2.2 Centrifuge method ( Figure 23)
Thehighaccelerationsinacentrifugeincreasethefeldofforceonasamplesubjecting
ittoanincreasedgravitationalforce.Thecoreplugismountedinamodifedcentrifuge
tubeasshownandthedesaturationofthesampleismonitoredwithastrobelight.
Whenthesampleisrotatedatvariousconstantspeedsacompletecapillarypressure
curvecanbeobtained.Theadvantageofthemethodistheincreasedspeedofobtaining
thedatainthatthecompletecurvecanbeestablishedinafewhours.

Window Core Holder Body Seal Cap
O-Ring Tube Body Support Disk Core
Figure 23 Centrifugefordeterminationofcapillarypressurecurves
5
.
6.2.3 Dynamic method ( Figure 24)
A dynamic method has been used where a simultaneous steady-state fow of two
fuidsisestablishedinthecore.Thesaturationisvariedbyregulatingthequantity
ofeachfuid enteringthe coreand thepressuredifferencebetweenthe twofuids
givesthecapillarypressure.


Core
p
g
p
o
p
c
Gas
outlet
Gas
inlet
Oil inlet
Porcelain
plate
To atmosphere
Oil burette
Figure 24 Dynamiccapillarypressureequipment
5
.
6.2.4 Mercury Injection ( Figure 25)
Themostcommonprocedurefordeterminationofcapillarypressureisusingmercury
injection.Theprocedurewasdevelopedtoacceleratethedeterminationofthecapillary
pressure-saturationrelationship.Mercuryisthenon-wettingfuid.Thecoresample
isinsertedintothemercurychamberofamercurypumporamercuryporosimeter
andevacuated.Mercuryistheninjectedintothecoreunderpressure.Thevolumeof
mercuryinjectedateachpressuredeterminesthenon-wettingphasesaturation.This
procedureiscontinueduntilthecoresampleisflledwithmercuryortheinjection
pressurereachessomepredeterminedvalue.Theprocedureisusedinanumberof
industriestodeterminetheporesizecharacteristicsoftheporousmedia.
The main advantages are that the test takes considerably less than the diaphragm
method,amatterofoneortwohours.Thedisadvantagesarethedifferenceinwetting
properties and permanent loss of the core sample. Also there is concern on the
poresizetopressurerelationshipsincethedesaturationofsomelargeporesmaybe
determinedbyaccessviasmallerpores.
Institute of Petroleum Engineering, Heriot-Watt University

Rock Properties Measurement



0-00 psi Pressure Guage
0-,000 psi Pressure Guage
Regulating Valve
Cylinder
Lucite Window
Lucite Window
U-Tube
Manometer
To
Atmosphere
Figure 25 Mercuryinjectionporosimeter
5
.
6.3 Use of Laboratory Capillary Pressure Data for Reservoir
Saturation Distribution.
As we have noted above, laboratory capillary pressure tests can be made with a
varietyoffuidsthatdifferfromreservoirfuids.Itisnecessarythereforetoconvert
laboratorybasedresultstobeapplicabletothefeldwherethefuidsmightbedifferent.
Wewillexaminetheprocedureforconvertingair-mercurydatatowater-oildatafor
applicationinfelddeterminationsofsaturationprofles.
Asshownpreviously,capillarypressuresaturationdatacanbeconvertedtoheight
saturationdata:

h
P
g
c
w o
=
( )
(1)
Air/mercurycapillarypressurecurvesarecomparableinshapetoair/brineoroil/brine
capillarypressurecurves.
Whenconvertingcapillarypressurecurvestoanequivalentheight,thedifferencein
interfacialtensionandcontactanglebetweenthelaboratoryandreservoirsystems
mustbeaccountedfor.Forexample
surfacetension()ofwater=70dynes/cm
surfacetension()ofmercury=480dynes/cm
contactangle()water/solid=0degrees
contactangle()mercury/solid=140degrees


P
2 Cos
r
c
=

(2)
Atcorrespondingsaturationstherefore

Pc
Pc
480Cos140
70Cos0
5
air / mercury
air / water
=
P
c
air/mercury=5P
c
air/water (3)
Theinterfacialtensionandcontactanglevalueswilldependonthecharacteristicsof
thefuids.TherelationshipbetweenPcmercury/airandPcoil/waterisoftentaken
as 10:1 but these interfacial tension and contact angle values should be checked
beforeconvertingdata.
P
c
air/mercury=10P
c
water/oil (4)
Theequationsbelowgivetheprocedureforgeneratingaheightsaturationproflefor
thereservoirfromalaboratorybasedPcvssaturationcapillarypressuredata.

h
P Cos
Cos
g
P
g
c
w h
c
w h
=
( )
( )
( )
=
( )
L R
L R



(5)
where:
h = height in feet above the free water level corresponding to zero capillary
pressure
P
cR
=capillarypressureatinitialreservoirconditions(psi)
P
cL
=capillarypressureinthelaboratory(psi)
(Cos)
R
= interfacial tension cosine of the contact angle (initial reservoir
conditions)
(Cos)
L
=interfacialtensioncosineofthecontactangle(laboratoryconditions)

w
=densityofwateratinitialreservoirconditions

h
=densityofhydrocarbonatinitialreservoirconditions
Itshouldbenotedthattheinterfacialtensionofanoil/watersystemisapproximately
10timesgreaterthanthatforanoil/gassystemandthatconsequentlycapillaryforces
aremoreimportantfortheformersystem.
Institute of Petroleum Engineering, Heriot-Watt University

Rock Properties Measurement


EXERCISE 1 Calculation of water saturation distribution in a layered reservoir.
The purpose of this exercise is to show that in a well, the water saturation not only
varies with the height above the free water level, but also due to variations in rock
properties.
A well penetrates a reservoir which from cuttings is known to consist of rock types
A and B from which a set of air-mercury measured capillary pressure curves are
available, taken in a nearby well. Figure E1. During logging the lowest 100% S
w

was found at the bottom of the well in rock type B as indicated in the fgure E.
The porosity at this level is 1%.
Specifc gravities of the water and oil are 1.03 and 0.80 respectively at reservoir
conditions. The density of water is . lb
m
/ft

.
Questions
1. Determine the Free Water level and locate it on fgure E2.
2. Construct the water saturation profle.
. Estimate permeabilities
. Which intervals would you recommend for completion based on the criteria
S
w
<0% and k<0.1mD.
What is the net pay (cumulative thickness having S
w
<0%).
0

0
00
10
100
0
0
0 0 100%
type A rock
type B rock
1 . .0
1 10 10
(mD)
(%)
Pore space unoccupied by mercury
Pc.
(psi)
h
(lt)
Figure E1 Capillarypressurecurvesfromnearbywell.
Institute of Petroleum Engineering, Heriot-Watt University
1
Rock Properties Measurement

100 S
w
in B
type rock
found at
this level
A
B
A
B
A
B
A
B
Rock
type
h
(ft)
(1 cm for
10 ft)
Porosity
1% 10
%
1%
10%
1%
%
%
%
%
1%
10%
10%
1%
1%
100 Water 0
0 Oil 100%
Unit No.
k
(mD)
Saturations
Figure E2 Opposite

6.4 Averaging capillary pressure data


Capillary pressure measurements are not part of routine core analysis and a
comprehensivesetofcapillarypressuredataisnotalwaysavailable.Leverett
4
in1941
generatedafunctionwhichrelatedcapillarypressuretoporosityandpermeability,
whichiscommonlytermedtheLeverett J Function.Theapplicationofthisfunction
wastobeabletogeneratecapillarypressureinformationwhenlaboratorydatawas
notavailable.Capillarypressuredataareobtainedfromcoresampleswhichrepresent
anextremelysmallpartofthereservoir.TheJfunctionisusedtocombineallthe
capillarydatatoclassifyaparticularreservoir.
ThetheorybehindtheJFunctionisoutlinedbelowandisbasedonfgure26considering
fowthroughacore,whichisassumedtobeabundleofcapillarytubes.

L
cap
L
core
Figure 26 ModeloffowforLeverettJFunction.
ThelaminarfowoffuidthroughapipeisgivenbyPoiseuillesequation:

q
r P
8 L
4
cap
=

(6)
Forntubes

q
r P
8 L
n
4
cap
=
n

(7)
Theporosityofthebundleoftubesis


=
n r
A
2
(8)
andthepermeabilityis

k
q L
A P
core
=

(9)
Institute of Petroleum Engineering, Heriot-Watt University

Rock Properties Measurement


IfAissubstitutedfornrandthen

r
8K
L
L
2 cap
core
=

(10)
L
L
cap
core
isthetortuosityofthebundleoftubes.
On the assumption that the reservoir rock has the same tortuosity at all points,
then

r constant
K
1
2
=

(11)
andsubstitutingforrinthedefnitionofcapillarypressuregives:,

P
2 Cos
constant
K
c 1
2
=

(12)
or

1
constant
P
K
Cos
J
c
1
2
=


(13)
SometimestheJfunctioniswrittenwithouttheCosterm.
Thecapillarypressuremeasurementscanthereforebenormalisedfordifferencesin
permeabilities,porositiesandfuidsandusedtomeasurethecapillarypressure,i.e.
theJfunctionisobtainedindependentofk,,and.
Asetofcapillarypressuredatafromasetof9coreplugstakenfromdifferentdepths
inawellisshowninfgure27andshowsthewidevariationinshapeofthesecurves
refectingthedifferentporecharacteristicsasgiveninthetablebelow.

1

0.
.10
.
0.
.
1,100.00
.00
.00
.10
1.
.
0.
0.
.0
.
.
.
1.
Sample
No.
Permeability
mD
Porosity
%
CAPILLIARY PRESSURE vs WATER SATURATION (Sw)

10
11
1
1
1
1
1
1
10 0 0 0 0 0 0 0 0 100
0
1
Sw %
P
c

(
P
S
I
G
)
Figure 27 Setofcapillarypressurecurves.
AplotoftheJfunctionforasetofcapillarypressurecurvesisgiveninfgure28and
showstheimpactofbringingtogetherdifferentrocksunderonecurve
Institute of Petroleum Engineering, Heriot-Watt University

Rock Properties Measurement



10
10
0 0 0 0 0 0 0 0 100
100
00
00
00
00
00
00
00
00
1000
1100
100
Sw %
P
c
(
k_
)
1 _
Figure 28 LeverettJFunction
Thedataforfgure27howeverwouldnotgeneratesuchagoodfunction.Thebig
assumption in Leverett's model is that of constant tortuosity. Clearly different
rocktypeswillhavedifferenttortuositiesasaresultoftheporecharacteristicsand
compositionoftherock.HoweverwithinarocktypetheJfunctioncouldbeauseful
routetoobtaincapillarypressuredataifporosity,permeabilityandsaturationdata
isavailable.
ExaminationoffelddatahasshownthatbyplottingJversusabettercorrelation

S S
S
w wc
wc
( )
( ) 1
isobtainedsuggestingthattheS
wc
refectsthetortuosityvariationswithin
thevariousrocks.Figure29


0 0.1

10
1
1
1
1
0

0. 0. 0. 0. 0. 0. 0. 0. 1.0
Normalised Wetting Phase Saturation Sw*
( (
Sw-Swc
1-Swc
=
D
i
m
e
n
s
i
o
n
l
e
s
s

C
a
p
i
l
l
a
r
y

P
r
e
s
s
u
r
e
P
c
K

LEGEND
Different reservoir sand sequence
in a formation
Figure 29 ModifedLeverettJFunctionCurves.
7. EFFECTIVE PERMEABILITY
It is not the intention of these notes to review in detail the various approaches
to measuring effective permeabilities to multiphase systems. There has been
considerableactivityinthisareaforgas-oil,oil-water,andthreephasegas-oil
-watersystems.
Therearetwoapproachestomeasuringrelativepermeability,usinganunsteadystate
methodorasteadystatemethod.
Institute of Petroleum Engineering, Heriot-Watt University

Rock Properties Measurement


Intheunsteadystatemethod,adisplacementprocessissetupwhereonefuiddisplaces
anotherandthefowratesandpressuredropsaremonitoredasafunctionoftime
forafxedrateprocess.Thesaturationsareobtainedbycalculationtheremaining
volumesoftherespectivefuids.Itismorediffculttogeneraterelativepermeabilities
asafunctionofsaturationinthiswayandsomewouldconsiderthemethodismore
suitedtogenerateend-pointeffectivepermeabilityvalues.
Inthesteadystatemethodarangeofconstantratetestsaresetupandthepressure
dropnotedwhenequilibriumhasbeenachieved.Figure30givesasketchofatypical
steadystatesetup.

Oil recycle system
Differential
pressure
transducer
Oil
Brine
Oil - water
separator and
production monitor
Differential
pressure
transducer
Composite core
Brine recycle
system
Pressure
control
system
P
P
Figure 30 Steadystaterelativepermeability.
Thefocusisagainonthreephaserelativepermeabilitywhichhasbeenthesubject
ofmanypapersandcorrelations.ItishoweverofgreatinterestnowthatlargeWAG,
water-alternatinggasinjectionprocessesarebeingusedtoimproverecovery.

Solution to Exercise
EXERCISE 1Calculationofwatersaturationdistributioninalayeredreservoir.
Thepurposeofthisexerciseistoshowthatinawell,thewatersaturationnotonly
varieswiththeheightabovethefreewaterlevel,butalsoduetovariationsinrod
properties.
Awellpenetratesareservoirwhichfromcuttingsisknowntoconsistofrocktypes
A and B from which a set of air-mercury measured capillary pressure curves are
available,takeninanearbywell.FigureE1.Duringloggingthelowest100%S
w

wasfoundatthebottomofthewellinrocktypeBasindicatedinthefgureE2. The
porosityatthislevelis15%.
Specifc gravities of the water and oil are 1.03 and 0.80 respectively at reservoir
conditions.Thedensityofwateris62.4lb
m
/ft
3
.
QUESTIONS
1.DeterminetheFreeWaterlevelandlocateitonfgureE2.
2.Constructthewatersaturationprofle.
3.Estimatepermeabilities
4.WhichintervalswouldyourecommendforcompletionbasedonthecriteriaS
w
<50%
andk<0.1mD.
Whatisthenetpay(cumulativethicknesshavingS
w
<50%).
SOLUTION
1.Thefrststepistoconverttheair-mercurycapillarypressuredatatooil-water.
P
c
air/mercury=10P
c
water/oil(equation4,page26)
P
cR
= h (
w
-
o
)g(equation5,page27)
Conversionvalues:

P
cair
/hg=10P
c
wateroil-

lb
in
f
2

P
lb
in
in
ft
h ftx x
lbm
ft
xg
P oil water
lb
n
in
ft
h ft x
lb
ft
xg
c
f
c
f
m
2
2
2 3
2
2
2 3
144
1 03 0 8 62 4
144
1 03 0 8 62 4
( . . ) .
/ ( ) ( . . ) .
=

=
Institute of Petroleum Engineering, Heriot-Watt University

Rock Properties Measurement


1lb
f
=1lb
m
xg
P
c
oil/waterpsi=0.1ftoil/water
Pcair/mercury=1ftoil/water
Thecapillarypressurecurvescannowberescaled.FigureE3.
Plottingh
ft
= P
c
air/mercury(psi)versus0-100%watersaturation.
2.Freewaterlevel
ThisoccursinrocktypeB.=15%.Fromcapillarypressurecurve100%water
saturationat15psii.e.15ft.
Freewaterlevelis15ftbelowthisposition,asindicatedonFigureE4.
The free water level now provides the basis for the water saturation profle
determination.
3.WaterSaturationProfle
Thewatersaturationvalueisdeterminedateachlevelwheretherockproperties
changebutnotingwherethe100%watersaturationvalueoccursforeachrocktype.
Atthefrstchange,theheightis20ftfromrocktypeB,15%totypeB10%
Fromthecapillarypressurecurvestherespectivesaturationsare75%and100%
FigureE4.ForrocktypeB10%,the100%watersaturationlevelisat27ftwhenthe
saturationdecreases.Thenextrockchangeisat41ftabovetheFreeWaterLevel,
fromrocktypeB10%totypeB14%withawatersaturationvalueof73%and
44%.The44%isbasedonanestimateofthecapillarypressurecurveforavalue
ofporosityof14%betweenthe15%and10%curves.Thisprocessiscontinued
throughallthedepthsoftherockpropertychangesandthetotalsaturationprofle
generated.
4.Theestimatesofpermeabilityarebasedonporositypermeabilitytrendsfromthe
limiteddatagivenforthevariousrocktypesofthecapillarypressurecurves.In
unit1rocktypeB15%thepermeabilityis35mDUnit2,B10%thepermeability
is 15mD Unit 3 B 14%, interpolation suggests a value around 32mD and so on
throughtheunits.
5. CompletionintervalsaccordingtothecriteriaS
w
<50%andk>0.1mDare
shadedonthefgureE4.
6. Netpayaddsuptoaround125ft.
0

0
00
10
100
0
0'
0'
0
0'
00'
10'
100'
0'
0'
0 0 100%
type A rock
type B rock
1 . .0
1 10 10
(mD)
(%)
Water saturation
Pc.
(psi)
h
(lt)
0'
1 psi
Figure E3 Capillarypressurecurvesfromnearbywell
Institute of Petroleum Engineering, Heriot-Watt University
1
Rock Properties Measurement
100 Water 0
A
B
A
B
A
B
A
B
0 Oil 100%
Unit No.
k
(mD)
Rock
type
Porosity
1% 10
Saturations
1
1
1
1
1
1
11
10

1
0.0
0.0
1
1
0.

1
0.1
0.0
0.0
10

1
0.

FWL
0
00
10
100
0
10 mm
100% WL
h
(ft)
1'
Figure E4

REFERENCES
1.Archer.S.,Wall.C.,PetroleumEngineeringPrinciplesandPractice,Graham
andTrotman1986
2.RecommendedPracticesforCoreAnalysis.AmericanPetroleumInstitute.
RecommendedPractise40.SecondEdition,Feb1998.
3.Smart.B,
4. Leverett.M,C.,CapillaryBehaviourinPorousSolids.TransAIME1941
5. AmyxetalPetroleumReservoirEngineeringMcCranhill1960

Você também pode gostar